You are on page 1of 83

https://t.me/UpscMaterials https://t.me/FreeUpscMaterials https://t.

me/MaterialForExams

GKTODAY

Current Affairs [PDF] -November 1-15, 2018

Manjul Tripathi | tripathimanjul18@gmail.com |

This is a dynamic PDF E-book by GKToday. The latest version of this book can be downloaded from this link
Published by: GKTODAY.IN
GKToday © 2018 | All Rights Reserved

This document is a property of GKToday. Unauthorized Duplication is not allowed.

https://t.me/APPSC
https://t.me/UpscMaterials https://t.me/FreeUpscMaterials https://t.me/MaterialForExams
Current Affairs [PDF] -November 1-15, 2018

Contents
......................... ......... ....... ....... ........ .................................................
News Headlines: November 1-15, 2018 ...................................................................................................................................................................... 3
November 1, 2018 .......................................................................................................................................................................................................... 13
November 2, 2018 ......................................................................................................................................................................................................... 16
November 3, 2018 ......................................................................................................................................................................................................... 19
November 4-5, 2018 .................................................................................................................................................................................................... 22
November 6, 2018 ........................................................................................................................................................................................................ 25
November 7-8, 2018 .................................................................................................................................................................................................... 29
November 9, 2018 ......................................................................................................................................................................................................... 31
November 10, 2018 ...................................................................................................................................................................................................... 34
November 11-12, 2018 .................................................................................................................................................................................................. 38
November 13, 2018 ....................................................................................................................................................................................................... 41
November 14, 2018 ...................................................................................................................................................................................................... 44
November 15, 2018 ...................................................................................................................................................................................................... 48
Multiple Choice Questions ............................................................................................................................................................................................ 51

Manjul Tripathi | tripathimanjul18@gmail.com |

© 2018 GKToday | Allhttps://t.me/APPSC


Rights Reserved | https://www.gktoday.in 2
https://t.me/UpscMaterials https://t.me/FreeUpscMaterials https://t.me/MaterialForExams
Current Affairs [PDF] -November 1-15, 2018

News Headlines: November 1-15, 2018


Current Affairs – NEWS HEADLINES: November 1, 2018
India
SC asks Centre for pricing details of the 36 Rafale fighter jets India is buying from France
182-metre tall ‘Statue of Unity’ of Sardar Vallabhbhai Patel costing Rs. 2990 crore inaugurated in
Sadhu-Bet Island, Gujarat
143rd birth anniversary of Sardar Patel celebrated as Rashtriya Ekta Diwas (National Unity Day)
Delhi HC sentences 16 policemen to life imprisonment for killing 42 Muslims in 1987 Hashimpura
massacre case in UP
Vice President departs for a visit to the African nations of Botswana, Zimbabwe and Malawi
Professor Ajit Prasad, Director of the Indian Institute of Management (IIM)-Lucknow, dies at 61
SSB DG S.S. Deswal appointed Director General of the Indo-Tibetan Border Police (ITBP)
Anupam Kher resigns as chairperson of the Film and Television Institute of India, Pune
J&K: Governor Satya Pal Malik flags off ‘Saubhagya Raths’ in Jammu for 20 districts to identify
households without electricity
First ever India-US Dialogue on Intellectual Property held in New Delhi by FICCI in partnership
USIBC
Economy & Corporate Manjul Tripathi | tripathimanjul18@gmail.com |

Growth of eight infrastructure sectors at 4.3% in September


Amazon Pay and ICICI Bank launch Amazon Pay ICICI Bank credit card in partnership with Visa
SBI cuts daily cash withdrawal limit for Classic and Maestro Debit Cards holders from Rs 40,000 to
Rs 20,000
World
New Zealand tops World Bank’s ‘Ease of Doing Business’ ranking of 190 countries, India 77th (100th
in 2017)
World Cities Day observed on October 31 with its theme as: ‘Building Sustainable and Resilient
Cities’
External Affairs Minister Sushma Swaraj meets Kuwait’s Amir Sheikh Sabah Al-Ahmad Al-Jaber Al-
Sabah in Kuwait City
EAM Sushma Swaraj meets Emir of Qatar Sheikh Tamim Bin Hamad Al-Thani in Doha
NASA’s 2009 Kepler space telescope retires after running out of fuel
Harvard University selects Nobel Prize Winner Malala Yousafzai for its 2018 Gleitsman Award for
promoting girls’ education
Pioneering coral reef scientist Ruth Gates dies in Honolulu (USA) at 56
Louis Cha, a Hong Kong journalist and best-selling Chinese martial arts novelist, dies at 94
Pakistan’s SC overturns conviction of Asia Bibi facing execution for blasphemy
Sports
Sri Lanka’s bowling coach Nuwan Zoysa ICC and charged with match-fixing
Pankaj Advani wins second leg of the Asian Snooker Tour in Jinan, China
Current Affairs – NEWS HEADLINES: November 2, 2018
India
SC allows use of 5 more documents by claimants for the draft Assam National Register of Citizens
(NRC), extends deadline to Dec 15
Foundation Day of Madhya Pradesh, Chattisgarh, Haryana, Karnataka and Kerala celebrated on
November 1
Lieutenant General P.S. Rajeshwar takes over as the 12th Chief of Integrated Defence Staff
Odisha: Jharsuguda airport renamed after freedom fighter Veer Surendra Sai
Kerala: 96-year-old Karthyayani Amma scores 98/100 marks in literacy exam under state’s
“Aksharalaksham” programme
Railway passengers can now book unreserved general tickets online

© 2018 GKToday | All Rights Reserved | https://www.gktoday.in 3


https://t.me/APPSC
https://t.me/UpscMaterials https://t.me/FreeUpscMaterials https://t.me/MaterialForExams
Current Affairs [PDF] -November 1-15, 2018

Union Cabinet approves opening up of International Solar Alliance (ISA) membership to all UN
countries
Cabinet approves MoU between India and South Korea in the field of tourism
India-Morocco agreement on legal assistance in criminal matters approved by Centre
2-week long India-Japan military exercise ‘Dharma Guardian-2018’ begins in Vairengte, Mizoram
Fourth Dialogue of NITI Aayog and Development Research Centre (DRC) of the State Council, China
held in Mumbai
Economy & Corporate
GN Bajpai resigns from the newly-appointed board of debt-ridden Infrastructure Leasing &
Financial Services (IL&FS)
TCS acquires London-based W12 Studios to boost creative design capabilities
US withdraws duty-free concessions on import of 50 Indian products, mostly from the agriculture
and handloom sectors
World
Indian Vice-President M. Venkaiah inaugurates 13th Global Expo Botswana in capital Gaborone
Maldives: Former President Mohamed Nasheed, who was in self-imposed exile in London for two
years, returns Manjul Tripathi | tripathimanjul18@gmail.com |
Sri Lanka: President Maithripala Sirisena lifts suspension of Parliament
Norwegian diplomat Geir Pedersen named UN’s next special envoy to Syria
Venezuelan politician and journalist Teodoro Petkoff dies in Caracas at 86
Satyajit Ray’s ‘Pather Panchali’ (1955) is the only Indian film included in BBC’s 100 best foreign
language films list
Qatar to host 23rd edition of the World Corporate Games in November 2019
Sports
Rahul Dravid becomes 5th Indian cricketer to be inducted into ICC Hall of Fame after Bishan Bedi,
Sunil Gavaskar, Kapil Dev and Anil Kumble
India (105/1 in 14.5) beat West Indies (103/10 in 31.5) by 9 wickets in final ODI at
Thiruvananthapuram to win 5-match series 3-1
Current Affairs – NEWS HEADLINES: November 3, 2018
India
Justices Hemant Gupta, R. Subhash Reddy, Mukesh Kumar Rasikbhai Shah and Ajay Rastogi sworn
in as judges of the Supreme Court
Maharashtra govt. to convert decommissioned aircraft carrier INS Viraat into a floating museum
Economy & Corporate
Foreign exchange reserves decline by $1.444 billion to $392.078 billion in the week to October 26
PM announces sops for MSME sector including approval for loans up to Rs 1 crore within 59 minutes
with 2% interest subvention and mandatory 25% procurement from MSMEs by CPSEs
Bank of Maharashtra restores powers of R. P. Marathe as MD and CEO. He was divested of
responsibilities in June following arrest by Pune Police in an alleged cheating case
promulgates an ordinance to amend the Companies Act, 2013
World
International Day to End Impunity for Crimes against Journalists observed on November 2
189 members of UN General Assembly vote to urge the US to end the economic embargo against
Cuba
NASA’s 11-year old Dawn mission to asteroid belt comes to end
Sports
‘Ekamra Sports Literary Festival’ organised by Odisha govt. in Bhubaneswar on November 2-3
Current Affairs – NEWS HEADLINES: November 4, 2018
India
Centre appoints journalist Arnab Goswami, former Foreign Secretary S Jaishankar, BJP MP Vinay

© 2018 GKToday | All Rights Reserved | https://www.gktoday.in 4


https://t.me/APPSC
https://t.me/UpscMaterials https://t.me/FreeUpscMaterials https://t.me/MaterialForExams
Current Affairs [PDF] -November 1-15, 2018

Sahasrabuddhe and IGNCA chairman Ram Bahadur Rai as members of Nehru Memorial Museum
and Library (NMML) Society. Former Union Minister MJ Akbar continues to be the vice chairman of
the NMML executive council. Shakti Sinha is Director of NMML.
Assam Police awarded by Centre for “good practices” under the Crime and Criminal Tracking
Network & Systems (CCTNS)
‘The Week’ journalist Namrata Biji Ahuja wins IPI (International Press Institute)–India Award for
Excellence in Journalism
Maharashtra: 5-year old alleged man-eater tigress Avni shot dead in Yavatmal district
Uttarakhand: ‘Gyan Kumbh’ being held at Patanjali Yogpeeth, Haridwar on November 3-4
Gujarat Govt. to provide “Namo E-Tab” tablet computers to 1st-year college students in state for Rs
1,000
Jharkhand: CM Raghubar Das announces to provide mobile phones to 28 lakh farmers of the state
Punjab: Aam Aadmi Party (AAP) suspends MLAs Sukhpal Singh Khaira and Kanwar Sandhu from the
party
Zika Virus strain that causes microcephaly not found in Rajasthan: Indian Council of Medical
Research
Ministry of Drinking Water and Sanitation announces Swachh Bharat World Toilet Day Contest for
Manjul Tripathi | tripathimanjul18@gmail.com |
districts and States
Vice-President M. Venkaiah Naidu meets Botswana’s President Mokgweetsi Masisi in Gaborone and
Zimbabwe’s President Emmerson Mnangagwa in Harare
Chief Commissioner of Railway Safety (CCRS), Lucknow to conduct an inquiry about the cause of
Amritsar accident
Economy & Corporate
Oil Marketing Companies, CSCs sign MoU to provide LPG-related services to people
World
8 nations can continue to import oil from Iran without attracting US sanctions but at lower levels
after November 5
Sports
Bangladesh beat Pakistan 3-2 in penalty shoot-out to win SAFF Under-15 Championship in Lalitpur,
Nepal; India win bronze
Uzbek businessman Gafur Rakhimov elected President of the International Boxing Association
(AIBA)
VVS Laxman launches cover of his autobiography ‘281 and Beyond’
Current Affairs – NEWS HEADLINES: November 4, 2018
India
National Seminar on Entrepreneurship and Business Development in Ayurveda being organised by
the Ministry of AYUSH in New Delhi on November 4-5
Economy & Corporate
RBI initiates steps to set up digital Public Credit Registry (PCR) to capture details of all borrowers
Central Information Commission (CIC) issues show-cause notice to RBI Governor for non-disclosure
of wilful defaulters’ list
Maharashtra government imposes electricity surcharge of 10 paise per unit on industrial and
commercial consumers to finance its scheme to provide 25,000 solar pumps to farmers
New Delhi Municipal Corporation (NDMC) launches quick response-enabled fridge magnets for
digital payment of utility bills by customers
CM Naveen Patnaik launches ‘Odisha Aerospace and Defence Manufacturing Policy 2018’
World
Indian actor Anupam Kher conferred ‘Distinguished Fellow’ title at Cambridge MIT Sloan School of
Management in Boston
Zimbabwe: Vice President M Venkaiah Naidu lays foundation stone of new Indian Embassy building

© 2018 GKToday | All Rights Reserved | https://www.gktoday.in 5


https://t.me/APPSC
https://t.me/UpscMaterials https://t.me/FreeUpscMaterials https://t.me/MaterialForExams
Current Affairs [PDF] -November 1-15, 2018

in Harare
Bangladesh: PM Sheikh Hasina announces construction of 560 model mosques and an Islamic
university with assistance from Saudi Arabia
The Pacific Ocean islands of New Caledonia opt to remain part of France in referendum
Sports
Australia’s Ashleigh Barty wins Women’s Singles title at WTA Elite Trophy in China
Current Affairs – NEWS HEADLINES: November 6, 2018
India
Ayurveda Day observed on Dhanawantari Jayanti (November 5)
Nuclear submarine INS Arihant successfully completes its first deterrence patrol
Hindu publishing group Chairman N. Ram wins Raja Ram Mohan Roy Award presented by the Press
Council of India
Mathematician and Super 30 founder Anand Kumar wins ‘Global Education Award’ of ‘Malabar Gold
& Diamonds’
Mizoram: Assembly Speaker Hiphei resigned from his post as well as the Congress and joins the BJP
The First Lady of South Korea Kim Jung-sook meets External Affairs Minister Sushma Swaraj in
New Delhi
Manjul Tripathi | tripathimanjul18@gmail.com |
India and South Korea sign MoUs to enhance cooperation in tourism and sports
Economy & Corporate
launches Operation Greens to ensure availability of tomato, onion and potato round the year
without price volatility.
Patanjali Paridhan launches apparel under brands Livefit, Aastha and Sanskar
World
10 Indian workers died every day on an average in gulf countries: Commonwealth Human Rights
Initiative
Vice President Venkaiah Naidu meets Malawian President Arthur Peter Mutharika in Lilongwe
2018 Europe Music Awards (EMA) conferred in Bilbao (Spain): Shillong-based Big-Ri & Meba Ofilia
win ‘Best India Act’ for their song “Done Talking”
UK opens up armed forces recruitment to Indians and nationals from the Commonwealth
S. grants temporary Iran oil waivers to eight countries including China, India
NASA’s Hubble Space telescope spots formation of galaxies that resembles a smiling face in the sky
Gaddam Dharmendra appointed India’s Ambassador to Iran
World Tsunami Awareness Day observed on November 5
Sports
Newly-built Ekana International Stadium in Lucknow renamed as ‘Bharat Ratna Atal Bihari Vajpayee
International Cricket Stadium’
New York Marathon: Kenya’s Mary Keitany wins women’s race in 2:22:48, Ethiopia’s Lelisa Desisa
wins men’s race in 2:05:59
Odisha CM Naveen Patnaik wins International Hockey Federation (FIH) President’s Award
Karen Khachanov defeats Novak Djokovic in final to win Men’s Singles title at Paris Masters tennis
2018 Jakarta Asian Games gold medallist Swapna Barman to receive seven pairs of customized shoes
from Adidas
Current Affairs – NEWS HEADLINES: November 7, 2018
Karnataka By-Election Results
Y. Raghavendra of BJP wins Shivamogga Lok Sabha seat
S. Ugrappa of Congress wins Ballari Lok Sabha seat
Shivarame Gowda of Janata Dal (Secular) wins Mandya Lok Sabha seat
Anitha Kumaraswamy of JD (S) wins Ramanagara assembly seat
Anand Nyamgouda of Congress wins Jamkhandi assembly seat

© 2018 GKToday | All Rights Reserved | https://www.gktoday.in 6


https://t.me/APPSC
https://t.me/UpscMaterials https://t.me/FreeUpscMaterials https://t.me/MaterialForExams
Current Affairs [PDF] -November 1-15, 2018

India
UP CM Yogi Adityanath and South Korean first lady Kim-Jung Sook inaugurate Queen Huh
Memorial at Queen Huh Park in Ayodhya
Uttar Pradesh: Faizabad district renamed as Ayodhya
Meghalaya: Justice Pranoy Kumar Musahary sworn in as Chairperson of state’s 1st Lokayukta
Bhajan singer Vinod Agrawal dies in New Delhi at 63
49th International Film Festival of India (IFFI) 2018 in Goa from Nov 20 to 28; Israel-focus country,
Jharkhand-focus state
Guinness world record set in Ayodhya with 3 lakh earthen lamps lit up simultaneously on the banks
of Saryu River
India declares its nuclear triad operational after nuclear submarine INS Arihant conducts 1st
deterrence patrol
Andhra Pradesh to get separate High Court in capital Amaravati from January 1, 2019: SC
Economy & Corporate
WCD Ministry sets up online marketing portal ‘Mahila-E-Haat’ for organic products of women
entrepreneurs
Women of India National Organic Festival 2018 organised by WCD Ministry in New Delhi from Oct
Manjul Tripathi | tripathimanjul18@gmail.com |
26 to Nov 4
World
International Day for Preventing the Exploitation of the Environment in War & Armed Conflict
observed on November 6
India elected as a member of International Telecommunications Union Council (ITU) for another 4-
year term from 2019 to 2022
Pakistan-China luxury bus service passing through PoK launched from Lahore to Kashgar
WTO dispute settlement panel upholds Japan’s complaint against a safeguard duty imposed by India
on steel imports
Sports
Bharat Ratan Atal Bihari Vajpayee International Cricket Stadium inaugurated in Lucknow
India (195/2 in 20) beat West Indies (121/9 in 20) by 71 runs 2nd T20 International in Lucknow to
clinch 3-match series
Maharashtra Cricket Association (MCA) stadium in Pune taken over by banks for non-payment of Rs
69.53 crore loan
2018 Jakarta Asian Games heptathlon gold medallist Swapna Barman to get 7 pairs of customized
shoes from Adidas
Current Affairs – NEWS HEADLINES: November 8, 2018
India
PM celebrates Diwali with jawans of the Indian Army and ITBP at Harsil in Uttarakhand
The US exempts India from imposition of certain sanctions for the development of the strategically-
located Chabahar Port in Iran
Economy & Corporate
RBI raises minimum average maturity requirement for ECBs (external commercial borrowings) in
the infrastructure sector raised by eligible borrowers to 3 years from earlier 5 years
World
United Nations Postal System issues special postal stamps to mark Diwali celebration
US Midterm elections: Opposition Democrats win majority in House of Representatives; President
Donald Trump’s Republican Party retains its majority in the Senate
UN World Food Programme and Chinese e-commerce giant Alibaba Group form strategic
partnership to support efforts eliminate hunger globally by 2030
Sports
8th Asian Shotgun Championship in Kuwait: Angad Vir Singh Bajwa wins gold in Men’s Skeet

© 2018 GKToday | All Rights Reserved | https://www.gktoday.in 7


https://t.me/APPSC
https://t.me/UpscMaterials https://t.me/FreeUpscMaterials https://t.me/MaterialForExams
Current Affairs [PDF] -November 1-15, 2018

Current Affairs – NEWS HEADLINES: November 9, 2018


India
Union Cabinet approves approved setting up of a Central Tribal University in Vizianagaram district
of Andhra Pradesh
J&K: Former bureaucrat Kewal Krishan Sharma assumes office as 4th advisor to Governor Satya Pal
Malik
Lottery falls within the purview of betting and gambling: Bombay High Court
Economy & Corporate
Govt approves mechanism for sale of enemy shares worth Rs 3,000 crore; these shares being held by
Custodian of Enemy Property of India (CEPI)
Cabinet approves leasing out six airports of AAI (Ahmedabad, Jaipur, Lucknow, Guwahati,
Thiruvananthapuram and Mangaluru) for operation, management and development under Public
Private Partnership (PPP)
Cabinet approves strategic disinvestment of 100% Govt. of India’s equity in the Dredging
Corporation of India Ltd. (DCIL)
Cabinet approves filling of Padur Strategic Petroleum Reserves (SPR) at Padur, Karnataka by
overseas National Oil Companies
Manjul Tripathi | tripathimanjul18@gmail.com |
Defence Ministry enhances financial powers of Vice Chiefs of the three Services in order to expedite
the decision making
Cabinet approves signing of a pact with Italy for facilitating customised training programmes on
labour and employment
Indian economy to grow 7.4% in 2018 and 7.3% in 2019: Moody’s Investors Service
India overtakes the US to become 2nd largest smartphone market behind China in July-September
2018 quarter: Canalys
World
USA: Attorney General Jeff Sessions resigns after being asked by President
US grants Iraq sanctions waiver to keep importing electricity from neighbouring Iran
Australia to offer Pacific countries up to $2.18 billion in grants and cheap loans to build
infrastructure to counter China’s rising influence in the region
Taiwan commissions two Perry-class guided missile frigates acquired from US
Samsung launches foldable phone Infinity Flex in US
Sports
Manpreet Singh to lead India’s 18-member hockey squad for the FIH World Cup beginning in
Bhubaneswar on Nov 28
Asian Airgun Championship in Kuwait City: Saurabh Chaudhary wins gold in 10m Air Pistol Junior
Men’s event
Current Affairs – NEWS HEADLINES: November 10, 2018
India
M777 A2 Ultra Light Howitzers, K-9 Vajra-Tracked Self Propelled Guns and 6×6 Filed Artillery
Tractors inducted into the Army
SIMBEX (Singapore-India Maritime Bilateral Exercise) to be held off Andaman Sea and Bay of Bengal
from Nov 10 to 21
Ladakh project wins UNESCO Asia-Pacific Award for Cultural Heritage Conservation
EC bans exit polls between Nov 12 and Dec 7 in poll-bound Chhattisgarh, MP, Rajasthan, Mizoram
and Telangana
French scholar Jawahar Lal Sarin presented Chevalier de la Légion d’Honneur, France’s highest
civilian honour, in Delhi
Abhay Kumar appointed as India’s next Ambassador to Madagascar
Statehood Day of Uttarakhand celebrated on November 9
Legal Services Day observed on November 9

© 2018 GKToday | All Rights Reserved | https://www.gktoday.in 8


https://t.me/APPSC
https://t.me/UpscMaterials https://t.me/FreeUpscMaterials https://t.me/MaterialForExams
Current Affairs [PDF] -November 1-15, 2018

Economy & Corporate


Former IAS officer Ashok Kumar Gupta appointed as Chairperson of the Competition Commission of
India (CCI)
IRS officers P. K. Dash, Akhilesh Ranjan and Neena Kumar inducted as members of CBDT
‘The Big Reverse: How Demonetization Knocked India Out’, written by former banker Meera H.
Sanyal, released
Lucknow-based Central Institute of Medicinal and Aromatic Plants (CIMAP) signs agreement with
US-based Research Institute for Fragrant Materials (RIFM) for improving the quality of fragrant oils
World
Swati Chaturvedi, author of ‘I am a Troll: Inside the Secret World of the BJP’s Digital Army’, wins
2018 London Press Freedom Award for Courage
Poor diet is a greater public health threat than malaria, tuberculosis or measles: FAO
Google renames its memory-decluttering app ‘Files Go’ to ‘Files by Google’
China’s Xinhua unveils world’s first virtual news anchors based on AI technology
‘Single-use’ named Word of the Year by Collins Dictionary
Sports
Asian Airgun Championships in Kuwait City: Manu Bhaker and Saurabh Chaudhary win gold in 10m
Manjul Tripathi | tripathimanjul18@gmail.com |
air pistol mixed team junior events
Current Affairs – NEWS HEADLINES: November 11, 2018
India
24th Kolkata International Film Festival (KIFF) being held at the Netaji Indoor Stadium from Nov 10
to 17
Karnataka: 269th birth anniversary of erstwhile Mysore (Mysuru) ruler Tipu Sultan celebrated on
November 9
J&K Govt. declares heavy snowfall in the Valley in November as state-specific ‘Special Natural
Calamity’
‘Healed: How cancer gave me a new life’, written by actress Manisha Koirala with Neelam Kumar,
released
Economy & Corporate
Demonetisation and GST held back India’s economic growth last year: Ex RBI Governor Raghuram
Rajan
World
Sri Lanka: President Maithripala Sirisena dissolves Parliament, calls for a snap general election on
January 5, 2019
World Science Day for Peace and Development celebrated on November 10 with its theme as
‘Science, a Human Right’
World Internet Conference organised in Wuzhen, China from Nov 7 to 9 with its theme as ‘creating a
digital world for mutual trust and collective governance — towards a community with a shared
future in cyberspace’
South Africa: A triple monument honouring former President Nelson Mandela, Zulu king Goodwill
Zwelithini and spiritual leader Swami Sivananda unveiled in a public park in Durban
France: War Memorial for Indian soldiers who died in World War I inaugurated at Villers Guislain by
Indian Vice President
Facebook launches video app ‘Lasso’ that enables users to make and share short-format videos
United States: 9 killed California wildfires
Sports
Kerala: 66th edition of Nehru Trophy Boat Race held at Punnamada Lake in Alappuzha Payyipadan
chundan (Snake Boat) wins
Bajrang Punia ranked first in Men’s 65 kg freestyle by United World Wrestling, is India’s first to get
top tank

© 2018 GKToday | All Rights Reserved | https://www.gktoday.in 9


https://t.me/APPSC
https://t.me/UpscMaterials https://t.me/FreeUpscMaterials https://t.me/MaterialForExams
Current Affairs [PDF] -November 1-15, 2018

Current Affairs – NEWS HEADLINES: November 12, 2018


India
130th birth anniversary of Maulana Abul Kalam Azad observed as National Education Day on
November 11
Jharkhand: CRPF launches language, culture course for better intelligence against Naxals
Fisheries ministers of South Indian states decide to implement the Minimum Legal Size (MLS)
measure recommended by the CMFRI to curb juvenile fishing
Veteran sports journalist Roshan Lal Sethi dies in New Delhi at 81
Economy & Corporate
International Symposium to Promote Innovation & Research in Energy Efficiency (INSPIRE 2018)
being organised in New Delhi by Energy Efficiency Services Limited (EESL) and World Bank from
November 11 to 13
Global IT Challenge for Youth with Disabilities organised by the Department of Empowerment of
Persons with Disabilities (DEPwD) in New Delhi from November 9 to 11
ADB to provide USD 13 million loan to EESL to promote efficient energy usage in India
Shareholders of Usha Martin approve sale of company’s steel business to Tata Steel
World
Manjul Tripathi | tripathimanjul18@gmail.com |
Sri Lanka: PM Mahinda Rajapaksa leaves President Sirisena’s Sri Lanka Freedom Party (SLFP), joins
the newly-formed Sri Lanka People’s Party (SLPP)
USA: 7 people including late singer Elvis Presley (1935-77) awarded Presidential Medal of Freedom
World leaders observe centenary of the end of First World War in Paris on November 11
Sports
Fuzhou China Open Badminton: Japan’s Kento Momota (Men’s) and Chen Yufei (Women’s) win
Singles titles
Hikaru Nakamura of US wins Tata Steel India Rapid Chess tournament in Kolkata
Saurav Ghosal defeats Zahed Salem to win men’s singles title at Kolkata International $30,000 PSA
squash tournament
Istanbul Marathon: Kenya’s Felix Kimutai wins men’s race in 2:09:57, compatriot Ruth Chepngetich
wins women’s race in 2:18:35
Current Affairs – NEWS HEADLINES: November 13, 2018
India
Public Service Broadcasting Day celebrated on November 12 to commemorate the only visit of
Mahatma Gandhi to the studio of All India Radio, Delhi in 1947
Chhattisgarh: Voting held in 18 of the 90 Assembly seats in first phase
Navies of India and Indonesia holding exercise ‘Samudra Shakti’ from November 12 to 18 in
Indonesia
India and Morocco sign an agreement on mutual legal assistance in criminal matters
Twitter launches #PowerOf18 campaign to boost youth engagement for 2019 polls
Former BBC journalist Mark Tully and Indian poet Jayanta Mahapatra win 2018 Tata Literature
Live! Lifetime Achievement Award and Poet Laureate Award respectively
‘A Great Association-Glimpses into the Life of Swami Bhoomananda Tirtha’ book authored by Swami
Nirviseshananda Tirtha released
Union Minister of Chemicals & Fertilizers and Parliamentary Affairs Ananth Kumar dies in
Bengaluru at 59
Economy & Corporate
Industrial production, measured in terms of Index of Industrial Production (IIP), grows 4.5% in
September
Retail inflation, based on Consumer Price Index (CPI), falls to one-year low of 3.31% in October
4 innovators selected for early detection of TB, malaria by Tata Trust and The Global Fund backed
Indian Health Fund

© 2018 GKToday | All Rights Reserved | https://www.gktoday.in 10


https://t.me/APPSC
https://t.me/UpscMaterials https://t.me/FreeUpscMaterials https://t.me/MaterialForExams
Current Affairs [PDF] -November 1-15, 2018

PSU construction firm NBCC bags Rs 260 crore contract to construct the All India Institute of
Ayurveda, Yoga and Naturopathy at Goa
ISPRL signs MoU with ADNOC (Abu Dhabi National Oil Company) to explore storage of crude oil at
Padur underground facility in Karnataka
Department of Biotechnology (DBT), India and Wellcome Trust, UK-(India Alliance), completes 10
year of partnership
Global Cooling Innovation Summit being organised in New Delhi on November 12-13
PM inaugurates country’s first multi-modal terminal constructed on river Ganga in Varanasi and
receives the first container cargo belonging to PepsiCo
PepsiCo becomes 1st company to use the country’s inland waterways for container movement by
transporting products from its plant in Kolkata to Varanasi
Economist Thirukodikaval Nilakanta Srinivasan dies in Chennai
Mahendra Singh Dhoni named brand ambassador of BharatMatrimony
World
World Pneumonia Day observed on November 12 with its theme as ‘Stop Pneumonia: Invest in Child
Health’
Meeting of Trade Ministers of RCEP member countries held in Singapore on November 12-13; Indian
Manjul Tripathi | tripathimanjul18@gmail.com |
delegate led by Union Minister for Commerce & Industry and Civil Aviation, Suresh Prabhu
Nilambar Acharya named Nepal’s next ambassador to India
Sports
Mercedes driver Lewis Hamilton wins Formula 1 Brazilian Grand Prix in Sao Paulo
Current Affairs – NEWS HEADLINES: November 14, 2018
India
Minister for Statistics and Programme Implementation D. V. Sadananda Gowda assigned additional
charge of the Ministry of Chemicals & Fertilizers
Rural Development, Panchayati Raj and Mines Minister Narendra Singh Tomar assigned additional
charge of the Ministry of Parliamentary Affairs
New envoys present credential to President: Pham Sanh Chau-Ambassador of Vietnam, Artis
Bertulis: Ambassador of Latvia, Mohamed Salam Jameel A. F. El-Kayed, Ambassador of Jordan &
Kalupage Austin Fernando: High Commissioner of Sri Lanka
Union HRD Ministry launches two new initiatives — Leadership for Academicians Programme
(LEAP) and Annual Refresher Programme In Teaching (ARPIT)
Armies of India and Russia holding “Exercise Indra 2018” in Jhansi (UP) from Nov 18 to Dec 2
6th Indian Social Work Congress in New Delhi: Dr. Martha Farrell posthumously honoured with
Lifetime Achievement Award
Punjab: Gobind Singh Longowal unanimously re-elected as President of Shiromani Gurdwara
Parbandhak Committee (SGPC)
Infosys Prize 2018
Each awardee receives a pure gold medal, a citation and a prize purse worth USD 100,000
Engineering and Computer Science: Navakanta Bhat of IISc, Bangalore
Humanities: Kavita Singh of JNU, New Delhi
Life Sciences: Roop Mallik of TIFR, Mumbai
Mathematical Sciences: Nalini Anantharaman of University of Strasbourg, France
Physical Sciences: S. K. Satheesh of IISc, Bangalore
Social Sciences: Sendhil Mullainathan of University of Chicago
Economy & Corporate
to release Rs 75 commemorative coin to mark 75th anniversary of hosting of Tricolour for the first
time by Netaji Subhash Chandra Bose at Port Blair on December 30, 1943
38th India International Trade Fair (IITF) at Pragati Maidan in New Delhi from Nov 14 to 27; theme:
Rural Enterprises in India, Afghanistan: Partner Country, Nepal: Focus Country, Jharkhand: Partner

© 2018 GKToday | All Rights Reserved | https://www.gktoday.in 11


https://t.me/APPSC
https://t.me/UpscMaterials https://t.me/FreeUpscMaterials https://t.me/MaterialForExams
Current Affairs [PDF] -November 1-15, 2018

State
Twitter CEO Jack Dorsey meets PM Modi in New Delhi
Flipkart Group CEO Binny Bansal quits over ‘serious personal misconduct’ charges
Ashok Leyland CEO and MD Vinod Dasari resigns
World
Comics legend and Spider-Man creator Stan Lee dies at 95
Amnesty International revokes Ambassador of Conscience Award given to Myanmar leader Aung
San Suu Kyi in 2009
33rd ASEAN Summit being held in Singapore from Nov 11 to 15 with its theme as ‘Resilient and
Innovative’
Sri Lanka: Supreme Court stays until December 7 the dissolution of Parliament by President
Maithripala Sirisena
Sports
Virat Kohli and Jasprit Bumrah remain top batsman and bowler respectively in the latest ICC ODI
rankings
Current Affairs – NEWS HEADLINES: November 15, 2018
India
Manjul Tripathi | tripathimanjul18@gmail.com |
Winter Session of Parliament from December 11 to January 8
Meghalaya: India International Cherry Blossom festival in Shillong from Nov 14 to 17
GSAT-29 communication satellite launched by GSLV MkIII-D2 from Satish Dhawan Space Centre at
Sriharikota
NITI Aayog’s Atal Innovation Mission and UNICEF present Young Champions Awards on occasion of
Children’s Day
India and UK sign MoU on ‘India-UK Cancer Research Initiative for Affordable Approaches to
Cancer’ in New Delhi
56th National Metallurgists’ Day celebrated in Kolkata on November 14
129th birth anniversary of 1st PM Pandit Jawaharlal Nehru celebrated as Children’s Day
Regional meeting of the World Customs Organisation being held in Jaipur from November 14 to 17
West Bengal govt. celebrating November 15 as “Rosogolla Dibas” to commemorate the first
anniversary of the sweetmeat getting Geographical Indication (GI) tag as ‘West Bengal’s Rosogolla’.
Tamil Nadu: Ruling AIADMK launches Tamil TV channel ‘News J’
Rajasthan: BJP’s Dausa MP Harish Chandra Meena and Nagaur MLA Habibur Rahman join Congress
Delhi Police launches e-learning portal ‘NIPUN’ for training its personnel
Economy & Corporate
Inflation, based on Wholesale Price Index (WPI), at 5.28% for October 2018
Govt launches credit scheme for cooperative startups titled ‘Cooperative Enterprise Support and
Innovation Scheme’
Ashok Chawla quits as non-executive chairman of YES Bank
IndusInd Bank launches IndusInd Bank Nexxt Credit Card – the first interactive Credit Card in India
with buttons
Royal Enfield launches Continental GT 650 Twin (Rs. 2.5 lakh) and Interceptor INT 650 Twin (Rs.
2.65 lakh)
World
Sri Lanka: Parliament passes no-confidence motion against PM Mahinda Rajapaksa
PM Modi launches APIX (Application Programming Interface Exchange), a banking technology
platform, at the Fintech Festival in Singapore
Israel: Defence Minister Avigdor Lieberman resigns
30 African countries launch joint initiative to save 4 carnivore species — wild dog, cheetah, leopard
and the lion
World Diabetes Day observed on November 14 with its theme as ‘The Family and Diabetes’

© 2018 GKToday | All Rights Reserved | https://www.gktoday.in 12


https://t.me/APPSC
https://t.me/UpscMaterials https://t.me/FreeUpscMaterials https://t.me/MaterialForExams
Current Affairs [PDF] -November 1-15, 2018

Photo-messaging app Snapchat introduces 2 new features – ‘Friendship Profiles’ and ‘Bitmoji
Stories’
Memoir ‘Becoming’, written by former U.S. first lady Michelle Obama, released
Sports
Athlete Hima Das appointed 1st Youth Ambassador of UNICEF India.
November 1, 2018
World Cities Day: October 31
The World Cities Day is observed every year across the world on 31st October to work together for
planned and sustainable urban life in the wake of rapid growth of population and problems.
Key Facts
The observance of this day aims to promote international community’s interest in global urbanization,
push forward cooperation among countries in meeting opportunities and addressing challenges of
urbanization, and contributing to sustainable urban development around the world. The global
observation of 2018 World Cities Day was celebrated in Liverpool, United Kingdom.
2018 Sub-theme: ‘Building Sustainable and Resilient Cities’. It aims to highlight importance to support cities
to become resilient and develop their capacity to absorb impact of hazards, protect and preserve human
life and limit damage to and destruction of public and private assets while continuing to provide
infrastructure and services after crisis. Manjul Tripathi | tripathimanjul18@gmail.com |

Background
The World Cities Day was established by United Nations General Assembly (UNGA) by passing resolution
68/239 in December 2013. This day was constituted in recognition of significance of urban basic services
as foundation for overall social and economic development. The general theme of World Cities Day is
“Better City, Better Life”, to promote successes of urbanization and address specific challenges resulting
from urbanization, while each year different sub-theme is selected to either promote successes of
urbanization or to address specific challenges resulting from urbanization. The first World Cities Day is
celebrated in Shanghai, China on 31 October 2014 under sub-theme “Leading Urban Transformations”.
Growth of eight infrastructure sectors at 4.3% in September 2018
As per data released by Union Ministry of Commerce and Industry, index of eight core industries slowed
down to 4.3% in September 2018. It is lowest growth recorded by core sectors in the last four months, as
production of crude oil and natural gas declined by 4.2% and 1.8%, respectively. Previously, lowest growth
rate was in May 2018 when core sectors expanded at 4.1%. In September 2018, Fertiliser, cement and
electricity sectors output grew by 2.5%, 11.8% and 8.2%, respectively. However, growth of coal, refinery
products, and steel sectors declined to 6.4%, 2.5% and 3.2%, respectively.
Core industries
Core industries are main or key industries of the economy. In most countries, these particular industries
serve as backbone of all other industries. In India, there are eight core sectors comprising of coal, crude oil,
natural gas, petroleum refinery products, fertilisers, steel, cement and electricity. The eight infrastructure
sectors, constitute 40.27% of the total index of industrial production (IIP).
Revised weightage in core sectors: Petroleum Refinery production (weight: 28.04%), Electricity
generation (19.85%), Steel production (17.92%), Coal production (10.33%), Crude Oil production (8.98%),
Natural Gas production (6.88%), Cement production (5.37%), Fertilizers production (2.63%).
NASA retires Kepler space telescope
NASA has retired Kepler space telescope after it ran out of fuel needed for further science operations. This
brings end of nine-and-a-half year mission of Kepler space telescope in which it had discovered over 2,600
intriguing exoplanets from outside our solar system some of which may harbour life.
Kepler space telescope
The unmanned space telescope was launched in 2009 on 3.5-year mission (from 2009 until November
2012), but operated for 9 years. It was NASA’s first planet-hunting mission. It was named after German
mathematician and astronomer Johannes Kepler. During its over nine years life, Kepler had observed
530,506 stars and detected 2,662 planets. It used transit photometry detection method for searching for
exoplanet, which looked for periodic, repetitive dips in visible light of stars caused by planets passing or

© 2018 GKToday | All Rights Reserved | https://www.gktoday.in 13


https://t.me/APPSC
https://t.me/UpscMaterials https://t.me/FreeUpscMaterials https://t.me/MaterialForExams
Current Affairs [PDF] -November 1-15, 2018

transiting in front in front of its host star. The telescope had suffered mechanical failure in 2013. But it
was made functional by changing its field of view periodically. This had paved way for K2 mission.
Anupam Kher resigns as Chairman of FTII
Veteran actor Anupam Kher has resigned as Chairman of the Film and Television Institute of India (FTII).
He was appointed as Chairman of FTII in October 2017 and had replaced Gajendra Chauhan. With this,
Kher became first chairperson of FTII to resign before completion of his full term. He had served as
chairman of FTII for 11 months, which makes his tenure shortest since FTII’s formation.
Anupam Kher
In span of his 35 years acting career, he has appeared in over 500 films and many plays. He is best known
for his roles in Saaransh, Daddy, Lamhe, Khel, Ram Lakhan, Dilwale Dulhania Le Jayenge, Maine Gandhi Ko
Nahin Mara and A Wednesday among others. He has also acted in international films like Bend It Like
Beckham, Bride and Prejudice, Silver Linings Playbook and The Big Sick. He is recipient of Padma Shri
(2004) and Padma Bhushan (2016).
Film and Television Institute of India (FTII)
FTII is premier institute in country that provides training for acting, film making, video editing, direction
and production. It is autonomous body under Union Ministry of Information and Broadcasting. It was
established in 1960 and is registered under Societies’ Registration Act of 1860. FTII is situated in Pune
(Maharashtra) on premises of erstwhile Prabhat Film Company.
Manjul Tripathi | tripathimanjul18@gmail.com | Since its inception, FTII has become
India’s premier film and television institute. Its alumni have become technicians, actors and directors in
the film and television industry.
SS Deswal appointed Director General of Indo-Tibetan Border Police
The Appointments Committee of the Cabinet (ACC) has appointed senior IPS officer S S Deswal as
Director General of Indo-Tibetan Border Police (ITBP). He will succeed R K Pachnanda, who
superannuated from office and shall be in office for period up to his superannuation on August 31, 2021.
Deswal is 1984 batch IPS officer of Haryana cadre. Prior to this appointment, he was Director General of
Sashastra Seema Bal (SSB).
Indo-Tibetan Border Police (ITBP)
ITBP (भारत-तिब्बत सीमा पुलिस बल) is one of five Central Armed Police Forces of India. It was
raised on 24 October 1962, under CRPF Act, in wake of 1962 Sino-Indian War. It was provided full
autonomy after Parliament enacted Indo-Tibetan Border Police Force Act, 1992. It works under command
of Union Home Ministry. It is headquartered in New Delhi.
Its motto is Shaurya – Dridhata – Karm Nishtha (English: Valour – Steadfastness and Commitment). At
present, ITBP is strong Central Armed Police Force of 90000 personnel. It primarily guards the 3,488 km
long Indo-China border along Tibet Autonomous Region from Karakoram Pass in Ladakh to Jachep La in
Arunachal Pradesh.
It is specialized mountain force and mans border outposts on altitudes ranging from 9000’ to 18700’ in
Western, Middle and Eastern sectors of Indo-China Border. It is also engaged in variety of internal
security tasks including anti-naxal operations, disaster management, civil Medical Camp, nuclear,
biological and chemical disasters and UN peacekeeping missions around the world.
First India-US Dialogue on Intellectual Property held in New Delhi
The first ever India-US Dialogue on Intellectual Property (IP) was launched in New Delhi to deepen
bilateral strategic cooperation on IP policy. It was launched by US Chamber of Commerce’s Global
Innovation Policy Center (GIPC) and Industry body Federation of Indian Chambers of Commerce &
Industry (FICCI) in partnership with US-India Business Council (USIBC). It will be convened annually,
alternating between New Delhi and Washington, DC.
India-US Dialogue on Intellectual Property
The dialogue aims to identify solutions and also share technical knowledge and expertise between both
countries in IP domain. It will provide platform to experts from both countries to develop stronger
understanding of uniform IP frameworks and create solutions in collaborative way. The discussions under
this dialogue included focus on joint opportunities and challenges related to whole spectrum of IP,
including patent filing, regulatory landscape, copyright and infringement, technology transfer, and

© 2018 GKToday | All Rights Reserved | https://www.gktoday.in 14


https://t.me/APPSC
https://t.me/UpscMaterials https://t.me/FreeUpscMaterials https://t.me/MaterialForExams
Current Affairs [PDF] -November 1-15, 2018

enforcement. Its recommendations will be shared with both governments to deepen strategic cooperation
on IP policy.
Harvard University selects Malala Yousafzai for 2018 Gleitsman Award for promoting girls’ education
Harvard University (US) has selected Nobel Prize Winner Malala Yousafzai (20) for its 2018 Gleitsman
Award in recognition of her work promoting girls’ education. She will be presented with this award at
December 6, 2018 ceremony. The Gleitsman Award provides US $125,000 for activism that has improved
quality of life around the world.
Malala Yousafzai
Malala was born on 12 July 1997 in Mingora city in Swat province of Pakistan. At the age of 11 she had
caught international attention as female education activist after writing for the BBC about life under the
Taliban in Pakistan under pseudonym Gul Makai. Her campaign had led to Taliban assassination attempt
near that left her severely wounded after she was shot in head in 2012. Later she had founded nonprofit
Malala Fund to support her work. At present she is student at Oxford University in England.
Awards and Honours: In 2014, at the age of 17 she was awarded Nobel peace prize, becoming youngest
Nobel Laureate in the history of Nobel award. She had shared this award with Indian children’s rights
activist Kailash Satyarthi. She also has been honoured with National Youth Peace Prize (2011), Mother
Teresa Awards for Social Justice (2012), Simon de Beauvoir Prize (2013), Sakharov Prize for Freedom of
Thought (2013), Anna Politkovskaya Award (2013). United Nations in her honour has declared 12 July as
Manjul Tripathi | tripathimanjul18@gmail.com |

‘Malala Day’.
India successfully conducted night trial of Agni-I ballistic missile
India has successfully carried out night user trial of Agni-I short-range nuclear-capable ballistic missile.
The test flight was conducted by Indian Army’s Strategic Forces Command off Abdul Kalam Island in Bay
of Bengal, off the coast of the Indian state of Odisha. The test was second known nighttime trial of Agni-I
since its first such successful test in April 2014.
Agni-I missile
Agni-I is short range nuclear capable surface-to-surface ballistic missile. It is first missile of the Agni
series launched in 1983. It was developed by premier missile development laboratory of DRDO in
collaboration with Defence Research Development Laboratory and Research Centre Imarat and integrated
by Bharat Dynamics Limited, Hyderabad.
It weighs 12 tonnes and is 15-metre-long. It is designed to carry payload of more than one tonne (both
conventional and nuclear warhead). It is single stage missile powered by solid propellants. It can hit a
target 700 km away. Its strike range can be extended by reducing the payload. It can be fired from road
and rail mobile launchers.
It is equipped with sophisticated navigation system which ensures it reaches the target with a high degree
of accuracy and precision. The missile already has been inducted into armed forces. Since its induction it
has proved its excellent performance in terms of range, accuracy and lethality. It is also claimed to be a
part of India’s minimum credible deterrence under No first to use policy.
India ranked 77th World Bank’s Ease of Doing Business Index
India ranked 77th among 190 countries by leapfrogging 23 ranks in World Bank’s Ease of Doing Business
(EODB) 2018 rankings. In the 2017 rankings, India ranked 100th and had jumped 30 places. In 2016, 2015
and 2014, India had not improved much and was placed at 130, 131 and 134, respectively, in ranking.
EODB 2018 was topped by New Zealand (consecutive for third time), followed by Singapore, Denmark, and
Hong Kong. US was placed at eighth and China was ranked 46th while Pakistan is placed at 136th.
Ease of Doing Business Index
The index ranks 190 countries based on 10 indicators across the life-cycle of a business, from “starting a
business” to “resolving insolvency.” These 10 indicators are: Starting business, Dealing with construction
permits, Getting electricity, Registering property, Getting credit, Protecting minority investors, Paying
taxes, Trading across borders, Enforcing contracts and Resolving insolvency. Each one of these indicators
carry equal weightage. In case of India, Delhi and Mumbai are only two cities surveyed by World Bank for
this reankings.
India’s performance in EODB 2018

© 2018 GKToday | All Rights Reserved | https://www.gktoday.in 15


https://t.me/APPSC
https://t.me/UpscMaterials https://t.me/FreeUpscMaterials https://t.me/MaterialForExams
Current Affairs [PDF] -November 1-15, 2018

Its jump in ranking in EODB 2019 was aided largely by improvement in areas such as “trading across
borders”, “dealing with construction permits”, “getting electricity”, “getting credit” and “starting a
business”.
It has improved in rank in six out of ten indicators. It has moved closer to international best
practice. India’s score also improved to 67.23, a gain of 6.63 from last year.
The most dramatic improvements by India was registered in indicators related to ‘construction
permits’ (improved to 52 from 181) and ‘trading across borders’ (rose to 80 from 146).
However, compared to last year, India saw its ranking dip on various other parameters, such as
“registering property”, “protecting minority investors”, “paying taxes” and “resolving insolvency”.
It is among top ten economies improving most across three or more areas alongside countries such
as Afghanistan, Djibouti, China, Azerbaijan, Togo, Kenya, Côte d’Ivoire, Turkey and Rwanda.
It is only other country, alongside Djibouti, to be in top ten for second year in row. Two economies
with largest populations — China and India — demonstrated “impressive reform agendas”.
It made starting business easier by integrating multiple application forms into general
incorporation form. India also replaced value-added tax (VAT) with GST for which registration
process is faster.
Manjul Tripathi | tripathimanjul18@gmail.com |

November 2, 2018
Rahul Dravid inducted into ICC Hall of Fame
Former India captain Rahul Dravid was officially inducted into International Cricket Council’s (ICC) Hall of
Fame. He presented memento by the legendary opener Sunil Gavaskar at Greenfield International
Stadium in Thiruvananthapuram, Kerala. With this, Dravid became fifth Indian cricketer after Bishan
Singh Bedi (inducted in 2009), Sunil Gavaskar (2009), Kapil Dev (2010) and Anil Kumble (2015) to be named
on the elite list. In July 2018, ICC had announced Rahul Dravid, Australian legend Ricky Ponting and
former England Women’s wicket-keeper Claire Taylor’s induction into Hall of Fame during ceremony in
Dublin. Ponting is 25th from Australia to receive the honour. Taylor is third female player from England to
be named on the list and seventh overall.
Rahul Dravid
Dravid has scored 23, 208 international runs for India across formats, including 48 hundreds and 146
fifties. He has scored the runs at an average of 45.41. He has scored 13,288 runs in 164 Tests, which
comprises 36 centuries, and has scored 10,889 runs in 344 One Day Internationals (ODIs) with 12
centuries. He has just played one Twenty20 International. He was named ICC Cricketer of the Year and
ICC Test Player of the Year in 2004. He holds world record for being excellent slip fielder and finished his

© 2018 GKToday | All Rights Reserved | https://www.gktoday.in 16


https://t.me/APPSC
https://t.me/UpscMaterials https://t.me/FreeUpscMaterials https://t.me/MaterialForExams
Current Affairs [PDF] -November 1-15, 2018

Test career in 2012 with 210 catches.


ICC Cricket Hall of Fame
It recognises achievements of legends of game from cricket’s long and illustrious history. It was launched
by the International Cricket Council (ICC) of Dubai on January 2, 2009 as part of its centenary celebrations
in association with Federation of International Cricketers’ Associations (FICA). Members of Hall of Fame
assist in selection of future inductees. The inaugural inductees were W. G. Grace and Graham Gooch.
Former England women’s cricket team captain Rachael Heyhoe-Flint was first woman cricketer to be
inducted in the Hall of Fame in 2010.
Union Cabinet approves India-Morocco agreement on legal assistance in criminal matters
The Union Cabinet has approved agreement between India and Morocco on mutual legal assistance in
criminal matters. The Union Cabinet was chaired by Prime Minister Narendra Modi.
Key Facts
The agreement will provide broad legal framework for bilateral cooperation between India and Morocco in
investigation and prosecution of crime, restraint, tracing, forfeiture or confiscation or proceeds and
instruments of crime. It will also increase effectiveness in investigation and prosecution of crime and
provide necessary peaceful ambience which is pre-requisite for development of society as a whole. It will
further be also instrumental in gaining better inputs and insights in modus operandi of terrorists and
organised criminals, which in turn can be Manjulused to fine-tune| policy decisions in the field of internal
Tripathi | tripathimanjul18@gmail.com

security.
Cabinet approves renaming of Jharsuguda Airport in Odisha as Veer Surendra Sai Airport, Jharsuguda
The Union Cabinet has approved renaming of Jharsuguda Airport in Odisha as “Veer Surendra Sai Airport,
Jharsuguda” after Veer Surendra Sai who is well-known freedom fighter of Odisha. The renaming of
airport fulfills long-pending demand of Odisha Government and reflects sentiments of local public of the
respective area. It will is befitting tribute to contribution of revered personality associated with State.
The airport has been developed by Airports Authority of India (AAI) in collaboration with Odisha
government at estimated cost of Rs 210 crore, with Rs 75 crore contribution from the state. It is spread
across over 1,027.5 acres of land and has 2,390-metre long runway. The area of its terminal building is
4,000 square metres.
Surendra Sai
He was freedom fighter and tribal leader, born in 1809 in small town Khinda in Sambalpur (now in
Odisha). He was direct descendant from Madhukar Sai and was legally entitled to be crowned as king of
Sambalpur after demise of king Maharaja Sai in 1827. But he was not acceptable to British power and
ignored his claim for succession.
He revolted against British raj for throne after it allow widow of Madhukar Sai Rani Mohan Kumari to
succeed him and then followed by succession of Narayan Singh, a descendant of royal family but born of
low caste as king of Sambalpur. The aim of Surendra Sai’s revolt was to drive the British out of Sambalpur.
His revolution against the British commenced from 1827 when he was only 18 years of age and continued
till 1862 when he surrendered and even after that, until he was finally arrested in 1864 – a total period of
37 years. He had suffered imprisonment in Hazaribagh Jail for 17 years in course of his revolutionary
career and after his final arrest for another term of 20 years including his detention of 19 years in remote
Asirgarh hill fort till he breathed his last there.
He was not only great revolutionary throughout his life but also inspiring leader of the people. He had
espoused cause of the down trodden tribal people who were being exploitated by higher castes people and
who became tools in hands of the British for establishment of their political power in Sambalpur. He died
in Asirgarh Jail on 23 May 1884.
Lt General Rajeshwar takes over as 12th Chief of Integrated Defence Staff
Lt Gen P S Rajeshwar took over as 12th Chief of Integrated Defence Staff to Chairman, COSC. Prior to this
appointment, he was serving as Director General (Perspective Planning) at Integrated Headquarters,
Ministry of Defence.
Lt Gen PS Rajeshwar
He was commissioned into Regiment of Artillery in December 1980. He is alumnus of Indian Military

© 2018 GKToday | All Rights Reserved | https://www.gktoday.in 17


https://t.me/APPSC
https://t.me/UpscMaterials https://t.me/FreeUpscMaterials https://t.me/MaterialForExams
Current Affairs [PDF] -November 1-15, 2018

Academy (IMA), Dehradun, National Defence College (NDC), Delhi. He holds Masters in National Security
Administration from NDC, Philippines. He has rich operational and staff experience with distinction of
serving in operational environments such as Counter Insurgency area in the North East & Counter
Terrorist Operations in Jammu and Kashmir, High Altitude Area, apart from United Nations
Peacekeeping Operations. During 38 years of his illustrious career, he has commanded Artillery Regiment
during Operation Parakram, Infantry Brigade at Line of Control (LoC), Counter Insurgency Force in J&K
and Corps in the desert sector.
Integrated Defence Staff
Government had established Integrated Defence Staff, headed by Chief of Integrated Staff as Chairman
(who enjoys status equivalent to that of Vice Chief of Service.) to ensure high degree of synergy between
Armed forces. It acts as point organisation for integration of policy, doctrine, war fighting and
procurement by employing best management practices. It was created on 1 October 2001 based on
recommendations of Group of Ministers which was set up in 2000 (post-Kargil) to review India’s defence
management. It has representation from all three services, Ministry of External Affairs (MEA), Defence
Research and Development Organisation (DRDO) and Department of Defence (DoD) and Def (Finance).
Cabinet approves MoU between India and South Korea for strengthening cooperation in Tourism
The Union Cabinet Narendra Modi has approved signing of Memorandum of Understanding (MoU)
between India and South Korea for strengthening cooperation in the field of Tourism.
Manjul Tripathi | tripathimanjul18@gmail.com |

Objectives of MoU
Expand bilateral cooperation in tourism sector
Increase exchange of information and data related to tourism
Encourage cooperation between tourism stakeholders including hotels and tour operators
Establish exchange programmes for cooperation in Human Resources Development (HRD)
Encourage Investment in Tourism and Hospitality sectors
Exchange visits of tour Operators, decision or opinion makers, media for promotion of two-way
tourism.
Exchange experience in areas of promotion, marketing destination development and management.
Encourage participation in travel fairs/exhibitions in each other’s country
Promote safe, honorable and sustainable tourism.
Background
India and South Korea have enjoyed strong diplomatic and long economic relationship. Both countries are
now desiring to strengthen and further develop established relationship in field of tourism. South Korea is
one of leading tourist generating markets for India from East Asia region. Signing of this MoU will be
instrumental in increasing arrival in India from this source market.
Dharma Guardian-2018: First India-Japan military exercise begins in Vairengte, Mizoram
The first joint military exercise named “Dharma Guardian” between India and Japan has begun at Indian
Army’s Counter-Insurgency Warfare School at Vairengte (Mizoram). The exercise is aimed at developing
mutual understanding and respect between militaries of both countries, as also facilitate in tracking
worldwide phenomenon of terrorism.
Dharma Guardian
In this exercise, Indian Army will be represented by 6/1 Gorkha Rifles, while Japanese Ground Self-Defence
Force will field its 32 Infantry Regiment. The emphasis of this exercise will be on building interoperability.
Armies of both countries will jointly train, plan and execute series of well-developed tactical drills for
neutralization of likely threats that may be encountered in urban warfare scenario. Moreover, experts
from both sides will also hold detailed discussions to share their expertise on varied operational aspects.
Union Cabinet approves opening up of International Solar Alliance membership to all UN countries
The Union Cabinet chaired by Prime Minister Narendra Modi has given ex-post facto approval for opening
up International Solar Alliance (ISA) membership to all countries that are members of the United Nations.
In this regard, Cabinet has approved to move Resolution in first Assembly of ISA for amending the
Framework Agreement of ISA.
Significance
It will put solar energy in global agenda with universal appeal for developing and deploying solar energy.
© 2018 GKToday | All Rights Reserved | https://www.gktoday.in 18
https://t.me/APPSC
https://t.me/UpscMaterials https://t.me/FreeUpscMaterials https://t.me/MaterialForExams
Current Affairs [PDF] -November 1-15, 2018

It will make ISA inclusive, whereby all member countries of UN could become member. Thus, expanding
membership will lead to ISA initiative benefitting the world at large.
International Solar Alliance (ISA)
ISA is an alliance dedicated to the promotion of solar energy among its member countries. The ISA
Framework Agreement had entered into force in December 2017 and formally became de-jure treaty based
International Intergovernmental Organization. It is headquartered at Gurugram, India. The main objective
of ISA is to include global deployment of over 1,000GW of solar generation capacity and mobilisation of
investment of over US$ 1000 billion into solar energy by 2030. ISA also aims to serve as an action-oriented
organization by bringing together countries with rich solar potential to aggregate global demand, thereby
reducing prices through bulk purchase. It also seeks to facilitate deployment of existing solar technologies
at scale, and promoting collaborative solar R&D and capacity building.
November 3, 2018
Justices Hemant Gupta, R. Subhash Reddy, M R Shah and Ajay Rastogi were sworn in as judges of SC
Justices Hemant Gupta, R Subhash Reddy, Mukesh Kumar Rasikbhai Shah and Ajay Rastogi were sworn-in
as the Judges of Supreme Court. They were administered oath of office by Chief Justice Ranjan Gogoi.
They will have tenure till they attain age of 65.
Key Facts
These four new judges of SC were appointed by President Ram Nath Kovind under Article 124 based on the
Manjul Tripathi | tripathimanjul18@gmail.com |

recommendation of SC Collegium comprising CJI Ranjan Gogoi, Justice Madan B Lokur, Justice Kurian
Joseph, Justice A K Sikri and Justice S A Bobde. With this total number of judges at the Supreme Court
rose to 28. The sanctioned strength of Supreme Court judges in 31 including Chief Justice.
Prior to this elevation, Justice Gupta served as Chief Justice of Madhya Pradesh High Court; Justice
Rastogi as Chief Justice of the Tripura High Court; Justice Shah as Chief Justice of the Patna High Court
and Justice Reddy as Chief Justice of Gujarat High Court.
Government launches Unreserved Mobile Ticketing facility
Union Ministry of Railways has launched Unreserved Mobile Ticketing facility (UTS on Mobile) to enable
seamless booking of unreserved tickets including season tickets and also platform tickets all over Indian
Railways. This will remove the need for passengers to wait in queues for purchasing the tickets.
UTS on Mobile aims to promote three C’: Cashless transactions (Digital payment), Contact less ticketing
(no need to physically visit the point of sale) and Customer convenience and experience. With this, booking
of unreserved tickets on all non-suburban sections across all Zonal Railways will be now made available on
mobile.
Unreserved Mobile Ticketing facility
The facility for booking unreserved tickets is available through ‘UTSONMOBILE’ app, available for mobile
phones with Android, IOS and windows operating systems. The process of ticket booking involves
downloading application on mobile phone and registration by furnishing the necessary details.
After successful registration, user is given user-ID and password through which user can log-in and book
ticket through this app within the ticketing area and outside Geo-fenced area. The payment can be made
by user digitally using R-Wallet, debit card, credit card, net-banking, UPI and other e-wallets.
Benefits
This environment friendly initiative will allow seamless booking of unreserved tickets all over Indian
Railways. It will obviate need for passengers to wait in queues for purchasing tickets. It will also enhance
experience for booking unreserved tickets i.e. for journey, platform tickets, season tickets etc.
12 Railway Zones free of unmanned level crossings on broad gauge routes: Indian Railways
Indian Railway has eliminated all Unmanned Level Crossings (UMLCs) on Broad Gauge routes (rails
separated by 1676 mm or 1.67m of distance) from 12 of its total 16 zones. These 12 zones are Central
Railway, Eastern Railway, East Coast Railway, East Central Railway, North Eastern Railway, Southern
Railway, North East Frontier Railway, South Central Railway, South Eastern Railway, South East Central
Railway and West Central Railway.
Key Facts
Out of total of 3,479 UMLCs on broad gauge routes still existing till April 2018, 3,402 have been eliminated
in. The balance 77 UMLCs have also been planned to be eliminated by December 2018. Most of UMLCs that
© 2018 GKToday | All Rights Reserved | https://www.gktoday.in 19
https://t.me/APPSC
https://t.me/UpscMaterials https://t.me/FreeUpscMaterials https://t.me/MaterialForExams
Current Affairs [PDF] -November 1-15, 2018

have been eliminated are either by provision of subway, road under bridges or by manning. All UMLCs on routes
having speed more than 130KMPH and on Sub-urban routes have been eliminated. By elimination of
UMLCs on war footing, accidents have reduced from 65 in 2009-10 to 3 in 2018-19.
Government Promulgates Ordinance to amend Companies Act, 2013
Union Cabinet has approved proposal for promulgation ordinance to amend the Companies Act, 2013. The
ordinance had received assent from President Ram Nath Kovind under Article 123 and has been
promulgated. The amendments to the Act are aimed to promote ease of doing business as well as ensure
better compliance levels.
Key Facts
The ordinance to change Companies Act seeks to declog National Company Law Tribunals (NCLTs) and
decriminalise minor offences by companies. The ordinance will transfer 90% of the cases to regional
directors under Ministry of Corporate Affairs from NCLTs. Moreover, it will retain status of all non-
compoundable offences since they are serious in nature.
Background
Union Government-appointed Committee (headed by Corporate Affairs Secretary Injeti Srinivas) had
suggested various changes to Act, including restructuring of corporate offences under companies law and
in-house adjudication mechanism to ensure that courts get more time to deal with serious violations.
Apart from restructuring of corporate offences to relieve special
Manjul Tripathi | tripathimanjul18@gmail.com | courts from adjudicating routine
offences, the committee had mooted re-categorisation of 16 out of 81 compoundable offences under the
Act. This move was recommended to bring down NCLT’s load as it looks at insolvency and bankruptcy
cases as well.
It also recommended disqualification of directors in case they have directorships beyond permissible
limits and capping an independent director’s remuneration. It also had suggested that remuneration any
independent director gets from company should be capped at 20% of his gross income in year to prevent
any material pecuniary relationship, which could impair their independence on the board.
India and Japan signs loan Agreement for Turga Hydel Project
India and Japan signed Loan Agreement worth Rs. 1817 crore for construction of Turga Pumped Storage
(I)’ Hydel Project. On completion, this project will contribute to Industrial Development and Living
Standard Improvement in the West Bengal.
Key Facts
This loan agreement was signed between Union Ministry of Finance, Government of India and Mr. Katsuo
Matsumoto, Chief Representative Japan International Cooperation Agency (JICA). The objective of project
is to strengthen capability to respond to fluctuation in supply and demand of power. It also seeks to
improve stability of power supply by constructing pumped storage facilities, thereby contributing to
industrial development and improvement of Living Standard of people in West Bengal.
Background
India and Japan have long and fruitful history of bilateral development cooperation since 1958. In the last
few years, the economic cooperation between India and Japan has steadily progressed, which has further
consolidated and strengthened Strategic and Global Partnership between them.
Maharashtra Government to convert decommissioned aircraft carrier INS Viraat into floating museum
Maharashtra state cabinet approved plant for conversion of Indian Navy’s longest-serving aircraft carrier
INS Viraat, into India’s first-ever moored maritime museum-cum-marine adventure centre. At present,
INS Viraat is at Mumbai’s Naval dockyard after it was decommissioned (retired) in 2017.
Key Facts
As per state government’s plan, INS Viraat’s conversion will be on publicprivate-partnership (PPP) basis. It
will be grouted (grounded, sealed to seabed with concrete and moored) seven nautical miles off Malvan
coast at Nivati rocks in Sindhudurg districct. This ship will host biodiversity centres and marine
adventure centre providing sailing and scuba-diving experiences. There will be virtual galleries, cafeterias
and even training centre for merchant navy crew. World over only seven aircraft carriers so far have been
converted into museums, theme parks and luxury hotel.
INS Viraat
It was built in 1943 during Second World War and was first commissioned as HMS Hermes into the British

© 2018 GKToday | All Rights Reserved | https://www.gktoday.in 20


https://t.me/APPSC
https://t.me/UpscMaterials https://t.me/FreeUpscMaterials https://t.me/MaterialForExams
Current Affairs [PDF] -November 1-15, 2018

Royal Navy in November 1959. During the Falklands War in 1982, the aircraft carrier had served as
flagship of the Royal Navy’s task force. British navy had decommissioned in 1985 after 27 years of service.
It has sailed nearly 11 lakh km, enough to cover the globe 27 times. It had spent 30 years in Indian Navy
and 27 years in Royal British Navy. It was commissioned into the Indian Navy on May 12, 1987.
It had played a major role in Operation Jupiter in 1989 in the Sri Lankan peace keeping operation. It had
participated in the standoff Operation Parakram in 2001-20012 when India and Pakistan were engaged in
a standoff post the Parliament terror attack. Its last operational deployment was for participation in
International Fleet Review held at Visakhapatnam, Andhra Pradesh in February 2016. INS Viraat holds
Guinness Books of record of being the longest serving warship in the world. It was also the last British-built
ship serving with the Indian Navy.
Odisha Government launches Saura Jalnidhi scheme
Odisha Government has launched Saura Jalnidhi scheme to encourage use of solar energy in irrigation by
farmers. Under this scheme, farmers will be given 90% subsidy and 5,000 solar pumps. This will provide
irrigation benefits in 2,500 acres of the state. The beneficiary of this scheme will be farmers who have
valid farmer identity cards and have minimum 0.5 acres of agricultural land.
Saura Jalnidhi scheme
The scheme was launched as part of state government’s endeavor to increase the use of solar photovoltaic
pump sets in those areas so that irrigationManjul
facilities can be provided where power system is bad. For this
Tripathi | tripathimanjul18@gmail.com |

scheme, state government has made a provision of Rs 27.18 crore in the financial year 2017-18.
In the first phase, this scheme will be available in those areas where electricity is not available for running
the pump set. This scheme will generate 1.52 lakh human day employments annually and provided
livelihood to around 5,000 families and also reduce carbon footprints.
Under this scheme, farmers will be provided with a well-equipped solar pump irrigation system in
convergence mode. It will help to reduce burden of input cost on farmers and also increase agricultural
income. It will promote the use of green energy and reduce pollution.
PM launches Support and Outreach Programme for MSME Sector
Prime Minister Narendra Modi launched Micro, Small and Medium Enterprises (MSME) Support and
Outreach Programme. As part of this programme, PM unveiled 12 key initiatives which will help the
growth, expansion and facilitation of MSMEs across the country. There are five key aspects for facilitating
the MSME sector. These include access to credit, access to market, technology upgradation, ease of doing
business, and sense of security for employees.
MSME Outreach Programme
It aims to boost MSME sector it is one of important sector for major generations of employment
opportunities. It will run for 100 days covering 100 Districts throughout the country. Various Central
Ministers will visit these districts in order to apprise entrepreneurs about various facilities being
extended to MSME Sector by Union Government and financial institutions. Under it, entrepreneurs will be
encouraged to come forward and make best use of these facilities including access to credit and market,
etc.
Under Access to Credit aspect of this porgramme, Government will launch 59 minute loan portal to enable
easy access to credit for MSMEs. Loans upto Rs. 1 crore will be granted in-principle approval through this
portal, in just 59 minutes. The link to this portal will be available at GST portal.
Government will give 2% interest subvention for all GST registered MSMEs, on fresh or incremental loans.
For exporters who receive loans in pre-shipment and post-shipment period will get increase in interest
rebate ranging from 3% to 5%.
All MSMEs or companies with turnover more than Rs. 500 crore will now be compulsorily brought on
Trade Receivables e-Discounting System (TReDS). This will enable entrepreneurs to access credit from
banks, based ontheir upcoming receivables. This will help to resolve their problems of cash cycle.
For access to markets for entrepreneurs, Union Government has taken number and now it is mandatory
for public sector companies to procure 25 %t, instead of 20% of their total purchases from MSMEs. Out of
the 25% procurement mandated from MSMEs, 3% is reserved for women entrepreneurs. It is mandatory
for all PSUs of Union Government to be part of GeM.
© 2018 GKToday | All Rights Reserved | https://www.gktoday.in 21
https://t.me/APPSC
https://t.me/UpscMaterials https://t.me/FreeUpscMaterials https://t.me/MaterialForExams
Current Affairs [PDF] -November 1-15, 2018

For Technology Upgradation, Government will form 20 hubs and 100 spokes in the form of tool rooms will
be established vital part of product design. For ease of doing business, government will establish clusters
of pharma MSMEs. 70% cost of establishing these clusters will be borne by the Union Government.
For simplification of government procedures, the return under 8 labour laws and 10 Union regulations
will be now filed only once year. Now establishments to be visited by an Inspector will be decided through
computerised random allotment.
Air pollution and water pollution laws now have been merged, and henceforth under them single consent
and will be required, besides return will be accepted through self-certification. Government has
promulgated ordinance under which, for minor violations under the Companies Act, entrepreneur will no
longer have to approach Courts, but can correct them through simple procedures. For Social Security for
MSME Sector Employees, government will launch mission to ensure that they have Jan Dhan Accounts,
provident fund and insurance.
November 4-5, 2018
Odisha Government launches Aerospace and Defence Manufacturing Policy 2018
Odisha Government launched Aerospace and Defence Manufacturing Policy 2018. The policy is aimed to
carry forward process of industrialization through promotion of aerospace and defence manufacturing
enterprises, generate employment opportunities and promote value addition.
Key Features of Polity Manjul Tripathi | tripathimanjul18@gmail.com |

It proposes to extend subsidy upto 50% of cost of land, building, plant and machinery to special purpose
vehicle (SPV) for setting up first state-of-the-art Aerospace and Defence Park in the state. This is to attract
investment in aerospace and defence manufacturing in the State, the policy
It sets ceiling of Rs. 50 crore for Common Facility Centre, Rs. 30 crore for Technology Innovation Centre
and Rs. 25 crore for Testing Centre to be established with private participation. It also provides capital
grants of 50% of infrastructure cost limited to Rs. 10 crore for subsequent aerospace and defence parks.
It also extends capital subsidy of Rs. 100 crore for first three OEMs (original equipment manufacturer) for
setting up manufacturing facilities in the state with investment of at least Rs. 1000 crore and generating
1000 domiciled employment.
In addition, it allows interest subsidy allowed up to a limit of Rs. 10 Crore per annum and Rs. 5 crore per
annum for first three OEMs setting up manufacturing units in the state based on investment in plant and
machinery for amount of more than Rs. 500 Crore and between Rs. 100 crore to Rs.500 crore, respectively.
This provision is designed to attract key players to set up units in the state. This will provide impetus for
further development of ancillary and downstream units in the state. Enterprises will come up in
industrially backward districts namely Kandhamal, Gajapati and Mayurbhanj along with KBK districts will
be extended additional incentives.
Shakti: IIT Madras develops India’s first indigenous microprocessor
The researchers from Indian Institute of Technology Madras (IIT Madras) have designed India’s first
indigenous microprocessor called ‘Shakti’. It is aimed at developing industrial-grade microprocessors and
other components of the microprocessor ecosystem. It will also help to reduce dependency on imported
microchips and risk of cyber-attacks.
Microprocessors are brain of all computing and electronic devices and are used to operate larger high-speed systems
and supercomputers.

Shakti microprocessor
It was designed, developed and booted by IIT Madras with microchip fabricated in ISRO’s Semi-Conductor
Laboratory at Chandigarh. It has been developed under project partly funded by Ministry of Electronics
and Information Technology (MeitY), as part of two-decade-old efforts to develop indigenous
microprocessors.
Its design originates from open source instruction set architecture (ISA), a set of basic instructions called
RISC V, which makes it customisable to any device. ISA is programming or machine language and
provides commands to processor instructing it on functions to be executed. RISC-V is open, free ISA
which enables new era of processor innovation through open standard collaboration. It delivers new level
© 2018 GKToday | All Rights Reserved | https://www.gktoday.in 22
https://t.me/APPSC
https://t.me/UpscMaterials https://t.me/FreeUpscMaterials https://t.me/MaterialForExams
Current Affairs [PDF] -November 1-15, 2018

of free, extensible software and hardware freedom on architecture.


Significance
It will reduce dependency on imported microchips especially in communication and defence sectors and
thus eliminate risk of cyber-attacks. It can be used in mobile computing, wireless and networking systems.
It may also provide power to mobile phones, smart meters and surveillance cameras.
Parashakti
IIT M researchers are now conducting research on ‘Parashakti’, an advanced microprocessor for
supercomputers. It will be super scaler processor that will be ready by December 2018. It will have its
applications in desktops and 32 such interconnected microprocessors may be used in supercomputers.
NDMC launches QR-enabled fridge magnets for digital payment of bills
New Delhi Municipal Corporation (NDMC) has launched quick response (QR)-enabled fridge magnets for
digital payment of bills. The magnets are personalised and each customer will have unique QR Code
mapped to their consumer account number. It is powered by SignCatch – an IT and services company,.
QR-enabled fridge magnets
There will be separate magnets provided by the NDMC to its consumers’ doorstep for both electricity and
water bills. Consumers can stick these magnets on their refrigerators at home and use their smartphones
to scan QR Code on magnet to pay their bills instantly. The QR Code will redirect consumers to their
current bill with option to pay using any ofManjul
digital payment channels active with NDMC like credit/debit
Tripathi | tripathimanjul18@gmail.com |
cards, net banking, wallet, and UPI. The move will reduce collection costs and expedite revenue realisation.
Moreover, these magnets will be extremely useful in cases where a consumer misplaces the pending paper
bills.
Quick Response (QR) code
It is a two-dimensional (matrix) machine-readable bar code made up of black and white square. This code
can be read by the camera of a smartphone. It is used for storing URLs or other information that link
directly to text, emails websites phone numbers. It is capable of 360 degrees (omni-directional), high speed
reading.
It can store up to 7089 digits as compared to conventional bar codes which can store max 20 digits. It
encodes same amount of data in one-tenth the space of a traditional bar code. It carries information both
horizontally and vertically. It has error correction capability and data stored in it can be restored even if it
is partially damaged or dirty.
AYUSH Ministry organizes National Seminar on Entrepreneurship and Business Development in Ayurveda
The National Seminar on Entrepreneurship and Business Development in Ayurveda was held in New
Delhi. It was organised by Ministry of AYUSH in association with NITI Aayog with aim of encouraging
entrepreneurs and Ayurveda stakeholders towards businesses opportunities in the sector.
Key Facts
The seminar is expected to develop awareness among stakeholders about the business opportunities,
encouraging young entrepreneurs to use of modern technologies and modern innovations in business
development in the sector and highlight opportunities at global level.
Nearly 800 participants from across country are expected to participate in main function of 3rd Ayurveda
day. It will be attended by experts in marketing, finance management, innovation, telemedicine, the policy
makers, experts in start-up as well as experienced entrepreneurs in Ayurveda pharmaceuticals and
hospital industry
The deliberations in seminar are expected to educate young entrepreneurs about the various possibilities
of business development in Ayurveda sector, usage of modern technology and ease of business being
facilitated by Government for promoting their business in the field of Ayurveda.
Bangladesh elected IORA Chairman for 2021-23
Bangladesh was as Vice Chairman of Indian Ocean Rim Association (IORA) for the period of 2019-21 and
eventually as IORA Chairman for 2021-23 at 18th IORA Council of Ministers’ Meeting held in Durban,
South Africa. The Current Chair is South Africa and Vice Chair is UAE.
Key Facts
As per practice, IORA is led by Chair and Vice Chair on behalf of member States. The stint of posts is two
years. When incumbent Vice Chair UAE will take post of Chair in 2019, Bangladesh will assume Vice

© 2018 GKToday | All Rights Reserved | https://www.gktoday.in 23


https://t.me/APPSC
https://t.me/UpscMaterials https://t.me/FreeUpscMaterials https://t.me/MaterialForExams
Current Affairs [PDF] -November 1-15, 2018

Chairmanship. After completion of 2 years of vice chairmanship, Bangladesh will become Chairman of
IORA for first time in October 2021. During 18th IORA Council of Ministers’ Meeting Maldives was
accepted as 22nd Member of IORA .Two outcome documents were adopted namely ‘Declaration on
commemoration of Nelson Mandela’ and eThekwini Communique.
Indian Ocean Rim Association (IORA)
IORA is international organisation consisting of coastal states bordering Indian Ocean, established in 1997
to promote cooperation in IOR. It is regional forum, tripartite in nature, bringing together representatives
of Government, Academia and Business for promoting co-operation and closer interaction among them.
It is based on the principles of Open Regionalism for strengthening Economic Cooperation particularly on
Trade Facilitation and Investment, Promotion as well as Social Development of the region. IORA
comprises 22 member states (including India) and 7 dialogue partners. Besides, Indian Ocean Research
Group and Indian Ocean Tourism Organisation have observer status.

Manjul Tripathi | tripathimanjul18@gmail.com |

Indian Coast Guard launches new Offshore patrol vessel ICGS Varaha
Indian Coast Guard (ICG) launches new Offshore patrol vessel (OPV) ICGS Varaha at L&T Kattupalli
shipyard near Chennai, Tamil Nadu. It is fourth in the series of 98 M OPVs designed and built indigenously
by Larsen & Toubro (L&T).
ICGS Varaha
It is fitted with advanced technology navigation and communication equipment, sensor and machinery.
Its weaponry includes one 30 mm and two 12.7 mm guns with fire control system. It has been designed to
attain maximum speed of 26 knots and has endurance of 5,000 nautical miles.
It also equipped with an Integrated Bridge System (IBS), Automated Power Management System (APMS),
Integrated Platform Management System (IPMS), and High Power External Fire Fighting (EFF) system. It
can carry one twin engine helicopter and four high speed boats, including two inflatable boats, for
boarding operation, search and rescue, law enforcement and maritime patrol. It will also be capable of
carrying pollution response equipment to contain oil spill at sea.
Background
In March 2015, Union Defence Ministry had awarded contract to L&T for construction of seven OPVs
awarded. The first and second ships of this project already have been delivered to ICG and are presently
based in Western Coast and Eastern Coast of India and third OPV will be delivered in early 2019. The
shipyard will deliver all seven ships by March 2021.
RBI initiates steps to set up digital Public Credit Registry (PCR) to capture details of all borrowers
Reserve Bank of India (RBI) has initiated steps to set up wide-based digital Public Credit Registry (PCR) to
capture loan information of individuals and corporate borrowers. In this regard, RBI has invited
expression of interest (EOI) for developing PSC from companies with turnover of over Rs 100 crore in last
three years. Setting up of PCR assumes significance amidst rising bad loans in financial system. The non-
performing assets (NPAs) in Indian banking system is about Rs 10 lakh crore.
Public Credit Registry (PCR)
PCR is digital registry of authenticated granular credit information. It will work as financial information
infrastructure providing access to various stakeholders and enrich the existing credit information

© 2018 GKToday | All Rights Reserved | https://www.gktoday.in 24


https://t.me/APPSC
https://t.me/UpscMaterials https://t.me/FreeUpscMaterials https://t.me/MaterialForExams
Current Affairs [PDF] -November 1-15, 2018

ecosystem. It seeks to serve as single point of mandatory reporting for all material events for each loan,
notwithstanding any threshold in the loan amount or type of borrower.
PCR will capture all details of borrowers, including wilful defaulters and also pending legal suits in order
to check financial delinquencies. It will also include data from entities like market regulator SEBI,
Corporate Affairs Ministry, Goods and Service Tax Network (GSTN) and Insolvency and Bankruptcy Board
of India (IBBI) to enable banks and financial institutions to get 360 degree profile of existing and
prospective borrowers on real-time basis.
Background
In June 2018, RBI had announced to set up PCR for India with view to address information asymmetry,
foster access to credit and strengthen the credit culture in the economy. This decision was taken based on
recommendation of high-level task force (HTF) i.e. YM Deosthalee committee which was constituted by RBI
to review current availability of information on credit, adequacy of existing information utilities, and
identify gaps that could be filled by PCR.
Currently, there are multiple granular credit information repositories in India, with each having
somewhat distinct objectives and coverage. Within RBI, CRILC is borrower level supervisory dataset with
threshold in aggregate exposure of Rs 5 crore. Moreover, there are four privately owned credit
information companies (CICs) operating in India. RBI has mandated all its regulated entities to submit
Manjul Tripathi | tripathimanjul18@gmail.com |
credit information individually to all four CICs.
November 6, 2018
World Tsunami Awareness Day: November 5
The World Tsunami Awareness Day is observed every year across the world on 5 November 2016 to
spread awareness among people across the world about Tsunami. This year it was third edition of World
Tsunami Awareness Day after it was instituted in 2015.
2018 World Tsunami Awareness Day is aligned with International Day for Disaster Reduction (observed
on 13 October) and “Sendai Seven Campaign”. It focused on Target “c” of Sendai Framework for Disaster
Risk Reduction which aims at reducing direct disaster economic loss in relation to GDP.
Significance of the Day
Spread awareness among people across the world in matters related to the dangers of tsunami.
Stress on importance of early warning systems in order to mitigate damage from devastating
natural calamity.
Background
The World Tsunami Awareness Day was designated by United Nations General Assembly (UNGA) by
adopting a resolution in December 2015. The proposal was first mooted by Japan after the 3rd UN
Conference on Disaster Risk Reduction held in Sendai (Japan) in March 2015.
This day coincides with the annual anniversary of the 1854 Inamura-no-hi (Fire of Inamura) event. On this
day in 1854, a villager in Wakayama Prefecture in Japan through his quick action had saved countless lives
after he had set fire to sheaves of rice on the top of a hill, thus warning people of the imminent danger of a
tsunami. This was the first documented instance of a tsunami early warning.
Tsunami
Tsunamis are large waves that approach crashing on coasts due to seafloor movement, majorly associated
with landslides or earthquakes. The word “tsunami” gets its name from the Japanese “tsu” means harbor
and “nami” means wave.
Tsunami is chain of huge waves created by disturbance created underwater. These waves are normally
associated with earthquakes taking place under or around the ocean. Other causes of Tsunami may be
submarine landslides, coastal rock falls, volcanic eruptions or extraterrestrial collision.
Like many other natural disasters, it is difficult to predict tsunamis but it can be suggested that
seismically active areas are more at risk. Tsunami waves are highly dangerous and generally look like
strong walls of water. The strong waves can attack seashore for hours, thereby destructing thousands of
lives.
India, South Korea ink MoU on sports cooperation
India and South Korea have signed Memorandum of Understanding (MoU) on sports cooperation. The

© 2018 GKToday | All Rights Reserved | https://www.gktoday.in 25


https://t.me/APPSC
https://t.me/UpscMaterials https://t.me/FreeUpscMaterials https://t.me/MaterialForExams
Current Affairs [PDF] -November 1-15, 2018

MoU was signed between Union Ministry of Youth Affairs and Sports and South Korea’s Ministry of
Culture, Sports and Tourism in New Delhi.
Key Facts
The objective of this MoU is to establish framework to facilitate and promote cooperation between two
countries on basis of reciprocity and mutual benefit. Under it two countries will encourage exchanges and
cooperative actions between sports authorities, sports federations and other sports bodies in different
areas of cooperation, as jointly decided.
Cooperation under this MoU may include areas like
Joint training for athletes and officials.
Exchange of coaches, athletes, experts.
Exchange of scientific and methodical materials.
Participation in sports event, seminars, symposia and conferences organized by either country etc.
India, South Korea sign MoU for strengthening cooperation in the field of Tourism
India and South Korea have signed a Memorandum of Understanding for strengthening cooperation in the
field of Tourism. The MoU was signed between Union Ministry of Tourism and South Korea’s Ministry of
Culture, Sports and Tourism in New Delhi. It was signed during official four-day visit of First Lady of
South Korea Kim Jung-sook (wife of incumbent South Korean President Moon Jae-in) to India. First Lady
Kim is Chief Guest at Deepotsav event organized by Uttar |Pradesh Government in Ayodhya and at
Manjul Tripathi | tripathimanjul18@gmail.com

ceremony of inaugurating new Memorial of Queen Suriratna (Heo Hwang-ok) in Ayodhya.


Ayodhya and Korea have historical connections. It is believed that lAyodhya’s egendary Princess Suriratna had
traveled to Korea in 48 CE and married Korean King Suro.

Key Facts
South Korea is one of the leading tourists generating market for India from East Asia region. Foreign
Tourist Arrivals (FTAs) from South Republic in 2017 were 1, 42, 383 and during January- September were 1,
08,901. The signing of this MoU will be instrumental in increasing tourist arrivals to India from this
source market.
It will expand bilateral cooperation in tourism sector, increase exchange of information and data related to
tourism and encourage cooperation between tourism stakeholders including Hotels and Tour Operators.
It will also encourage Investment in the Tourism and Hospitality sectors and promote safe, honorable and
sustainable tourism
Home Ministry releases Rs. 113 crore to six border states under Border Area Development Programme
Union Home Ministry has released more than Rs. 113 crore to six border states under Border Area
Development Programme (BADP) as part of its plan to ameliorate the problems of people living in isolated
locations. These six states are Assam, Nagaland, Sikkim, Gujarat, Rajasthan and Uttarakhand.
Key Facts
Till now, Home Ministry has released total of Rs. 637.98 crore during 2018-19 period to states having
International Border. The funds released are in addition to the Rs. 1,100-crore released in 2017-18 for the
all-round development of villages located along International Border in 17 states. India shares land border
with seven neighbouring countries viz. Bangladesh (4,096 km), China (3,488 km), Pakistan (3,323 km),
Nepal (1,751 km), Myanmar (1,643 km), Bhutan (699 km) and Afghanistan (106km in Pakistan Occupied
Kashmir).
Border Area Development Programme (BADP)
The umbrella BADP scheme is being implemented by Union Home Ministry in 367 border blocks of 104
border districts in 17 states. Its objective is to meet special developmental needs and well-being of people
living in remote and inaccessible areas situated near international border areas. It also seeks to saturate
border areas with entire essential infrastructure through convergence of Union, state, BADP, local
schemes and participatory approach.
Sub-schemes under it include construction of primary health centres (PHCs), connectivity, schools, supply
of drinking water, community centres, drainage to enable sustainable living in border areas. It also covers
schemes or activities relating to skill development programmes, promotion of sports activities in border

© 2018 GKToday | All Rights Reserved | https://www.gktoday.in 26


https://t.me/APPSC
https://t.me/UpscMaterials https://t.me/FreeUpscMaterials https://t.me/MaterialForExams
Current Affairs [PDF] -November 1-15, 2018

areas, promotion of rural tourism, border tourism, protection of heritage sites, Swachhta Abhiyan,
construction of helipads in remote and inaccessible hilly areas, which do not have road connectivity.
Skill development training to farmers for use of modern and scientific techniques in agriculture, organic
farming is also part of BADP now. As many as 61 model villages are being developed under BADP to
improve the quality of life for people living near border areas. A total of Rs 13,400 crore so far have been
released since BADP programme was initiated in 1986-87.
Government approves operationalisation strategy for Operation Greens
Union Ministry of Food Processing Industries (MoFPI) approved operationalisation strategy for Operation
Greens. Operation Greens was announced by Union Government in Budget 2018-19 to stabilise supply of
Tomato, Onion and Potato (TOP) crops. It is aimed at ensuring availability of TOP crops throughout the
country round the year without price volatility.
Strategy comprises series of measures
Short term Price Stabilisation Measures
National Agricultural Cooperative Marketing Federation of India (NAFED) will be nodal agency to
implement price stabilisation measures. MoFPI will provide 50% of subsidy two components of
transportation of TOP Crops from production to storage and hiring of appropriate storage facilities for
TOP Crops.
Long Term Integrated value chain development projects
Manjul Tripathi | tripathimanjul18@gmail.com |

Capacity Building of FPOs & their consortium


Quality production
Post-harvest processing facilities
Agri-Logistics
Marketing / Consumption Points
Creation and Management of e-platform for demand and supply management of TOP Crops
Assistance and eligibility
The assistance under this strategy will comprise of grants-in-aid at rate of 50% of the eligible project cost
in all areas, subject to maximum Rs 50 crores per project. Eligible organisations under it will include State
Agriculture and other Marketing Federations, cooperatives, companies, Farmer Producer Organizations
(FPO), Self-help groups (SHGs), food processors, logistic operators, supply chain operators, retail and
wholesale chains and central and state governments.
Operation Greens
It was announced in Union Budget 2018-19 on line of Operation Flood to promote FPOs, agri-logistics,
processing facilities and professional management. It aims to promote farmer producers organisations,
processing facilities, agri-logistics and professional management. It also aims to aid farmers and help
control and limit erratic fluctuations in the prices of tomatoes, onions and potatoes (TOP). It is essentially
price fixation scheme that aims to ensure farmers are given the right price for their produce. The idea
behind it is to double the income of farmers by the end of 2022.
Its major objectives are:
Price stabilisation for producers and consumers by proper production planning in TOP clusters.
Reduction in post-harvest losses by creation of farm gate infrastructure, development of suitable
agro-logistics, and creation of appropriate storage capacity linking consumption centres.
Enhance value realisation of TOP farmers by targeted interventions to strengthen TOP production
clusters and their FPOs,
Increase in food processing capacities and value addition in TOP value chain.
Set up of market intelligence network to collect and collate real time data on demand and supply and
price of TOP crops.
Nuclear submarine INS Arihant completes first deterrence patrol
India’s first indigenous nuclear submarine INS Arihant has successfully completed its first deterrence
patrol. The development signifies that underwater warship has completed its maiden long-range mission
with live nuclear-tipped missiles.

© 2018 GKToday | All Rights Reserved | https://www.gktoday.in 27


https://t.me/APPSC
https://t.me/UpscMaterials https://t.me/FreeUpscMaterials https://t.me/MaterialForExams
Current Affairs [PDF] -November 1-15, 2018

Key Facts
During deterrence patrol, a nuclear submarine carries nuclear missiles on board, where command and
control protocols for its operations are fully tested by its crew. The term deterrence patrol is meant to
deter an adversary from launching a first nuclear-strike since SSBN (Ship Submersible Ballistic Nuclear)
can launch retaliatory strike within minutes. With this, India completed its survivable nuclear triad by
adding maritime strike capability to land and air-based delivery platforms for nuclear weapons. It makes
India sixth country — after US, Russia, UK, France and China — to have fully operational nuclear triad.
INS Arihant
INS Arihant is India’s first indigenously-designed, developed and manufactured nuclear-powered ballistic
missile submarine, and three more such submarines are reportedly under various stages of construction.
It was designed in 1990s and its development project was officially acknowledged in 1998. Its design is
based on the Russian Akula-1 class submarine.
It was launched in 2009 and its nuclear reactor went critical in 2013 and it was commissioned in 2016. It is
strategic asset developed for over two decades with Russia’s help under Advanced Technology Vessel
(ATV) programme, which comes directly under Nuclear Command Authority headed by the Prime
Minister.
INS Arihant is 6,000-tonne submarine with length of 110 metres and breadth of 11 metres. It is powered by
83 MW pressurised light water nuclear reactor with
Manjul Tripathi enriched uranium
| tripathimanjul18@gmail.com | fuel. It can carry 12 Sagarika K 15
submarine launched ballistic missiles (SLBMs) having range of over 700 km.
As it powered by nuclear reactor it can function submerged for months without having to surface. This
feature allows it to travel further and with greater stealth capability. It assures second strike capability to
India i.e. capability to strike back after being hit by nuclear weapons first. In case of India, second strike
capability is important as it had committed to ‘No-First-Use’ policy as part of its nuclear doctrine.

© 2018 GKToday | All Rights Reserved | https://www.gktoday.in 28


https://t.me/APPSC
https://t.me/UpscMaterials https://t.me/FreeUpscMaterials https://t.me/MaterialForExams
Current Affairs [PDF] -November 1-15, 2018

Nuclear triad capability of India


India is sixth Nation in the world (after US, Russia, France, China and UK) to possess Nuclear Triad. It
means that India is capable of delivering nuclear weapons by aircraft, land based ballistic missiles and
submarine launched missiles. India Army has strong arsenal of land based ICBM (intercontinental ballistic
missile). It includes Agni series, Prithvi series, Prahar Missile, Shaurya Missile, supersonic Brahmos and
subsonic Nirbhay missiles. Indian Airforce (IAF) operates Jaguars aircrafts which are designed for deep
penetration strike and can carry nuclear bomb. Besides, IAF also has SU 30 MKI and Rafale aircrafts which
can also be used to deliver nuclear weapons. Indian Navy now has its own nuclear powered ballistic
missile submarine INS Arihant capable of delivering nuclear weapons.
November 7-8, 2018
Angad Vir Singh Bajwa wins gold in Men’s Skeet at 8th Asian Shotgun Championship
Angad Vir Singh Bajwa won gold medal by setting world record score in men’s skeet final of the 8th Asian
Shotgun Championship held in Kuwait. With this, he became first Indian skeet shooter to win continental
or world level event. In the final round of this event, Angad shot perfect 60 out of 60 to claim top spot
ahead of China’s Di Jin who shot 58, winning silver medal. UAE’s Saeed Al Maktoum won bronze with a
score of 46.
10m rifle mixed event
India’s pair of Elavenil Valarivan and Hriday Hazarika won gold medal in this event. In 45-shot final of this
Manjul Tripathi | tripathimanjul18@gmail.com |

event, they scored 502.1, setting world and Asian Junior record in the event. Chinese pair Shi Mengyao
and Wang Yuefeng won silver medal with 500.9. India’s another pair Mehuli Ghosh and Arjun Babuta won
bronze in the same event with a score of 436.9.
Lucknow’s Ekana cricket stadium renamed after Atal Bihari Vajpayee
Uttar Pradesh Government has decided to rename newly-built Ekana International Stadium in capital city
Lucknow after former Prime Minister Atal Bihari Vajpayee. The stadium with a seating capacity of around
50,000 will now be known as ‘Bharat Ratna Atal Bihari Vajpayee International Cricket Stadium’.
Key Facts
The stadium was renamed just ahead of first-ever international cricket match – T20 International
between India and West Indies. For renaming stadium, UP Government through Lucknow Development
Authority (LDA) used powers vested as per clause 17.5.1 of agreement signed between Ekana Sports City
Private Ltd; GC Constructions; Development Industries Pvt Ltd and LDA. The clause 17.5.1 of agreement
says that stadium will be named once project is completed. Its name was changed in memory of Atal
Bihari Vajpayee, who passed away in August 2018. The former prime minister was Member of Parliament
from Lucknow for five consecutive times between 1991 and 2009.
Note: Yogi Adityanath Government in UP earlier this year also renamed Allahabad as Prayagraj and
Mughalsarai as Pt. Deen Dayal Upadhyaya nagar.
RBI liberalises norms governing external commercial borrowings for infrastructure creation
Reserve Bank of India (RBI) has liberalised norms governing external commercial borrowings (ECBs) for
infrastructure creation. The provisions have been reviewed and decision has been taken in consultation
with Central Government.
Key Facts
RBI has reduced minimum average maturity required for the ECBs in the infrastructure space raised by
eligible borrowers to three years from earlier five years. Additionally, it also has reduced average maturity
requirement for mandatory hedging to five years from earlier ten years.
Background
The move comes amid concerns surrounding availability of funds following liquidity squeeze and the
difficulties being faced by non-bank lenders, especially those facing asset liability issues due to heavy
reliance on short-term funding for long-term assets. This, along with defaults by infra lender IL&FS, has
hurt credit markets especially infrastructure financing sector. Central Government has been unequivocal
in suggesting remedial measures which will address needs of the economy. It had suggested to include
special window for NBFCs, but RBI is not undertaking measures. However, relaxations in ECB norms
follow other moves by RBI, including earlier it permission to banks to use credit enhancement to help
NBFCs raise medium to long-term funds.
© 2018 GKToday | All Rights Reserved | https://www.gktoday.in 29
https://t.me/APPSC
https://t.me/UpscMaterials https://t.me/FreeUpscMaterials https://t.me/MaterialForExams
Current Affairs [PDF] -November 1-15, 2018

UN World Food Programme, Alibaba Group form strategic partnership to eliminate hunger
United Nations World Food Programme (WFP) and Chinese e-commerce giant Alibaba Group have formed
strategic partnership to support efforts eliminate hunger globally by 2030. The agreement was signed
between WFP Executive Director David Beasley and Alibaba Partner and Chairman of Alibaba Foundation
Sun Lijun at Alibaba’s headquarters in Hangzhou, China.
Key Facts
Under this agreement, Alibaba will lend its cutting-edge technology and resources to support digital
transformation of WFP’s operations. In particular, Alibaba Cloud, the cloud computing arm of Alibaba will
work with WFP to develop digital “World Hunger Map”. The map will help to monitor global hunger and
operations to end scourge by 2030 which is one of UN’s key Sustainable Development goals. It also aims to
boost efficiency of interventions and shorten emergency response times.
World Food Programme (WFP)
The WEF is the food-assistance branch of the United Nations. It is the world’s largest humanitarian
organization addressing hunger and promoting food security. It works to help people who cannot produce
or obtain enough food for themselves and their families. It is a member of the United Nations
Development Group (UNDG) and part of its Executive Committee. It was established in 1961 after 1960
Food and Agricultural Organization (FAO) Conference. It is headquartered in Rome (Italy) and has more
than 80 country offices around the world. It provides
Manjul food assistance
Tripathi | tripathimanjul18@gmail.com | to an average of 80 million people in
75 countries each year.
United Nations Postal System issues special postal stamps to mark Diwali celebration
United Nations Postal Administration (UNPA) has issued special stamps with Diyas lamps (special event
sheet) to commemorate India Hindu festival of Diwali. These stamps are available at UN headquarters
post office in New York, US and also online.
Key Facts
The special event sheet (stamps) issued by UNPA are in denomination of US $1.15. It contains ten stamps
and tabs featuring festive lights and symbolic lamps known as diyas. The background of sheet features
United Nations Headquarters building illuminated with message of “Happy Diwali” to celebrate the spirit
of the festival.
The description accompanying information about stamps mentioned that Diwali, also known as
Deepawali is joyous and popular festival of lights, which is celebrated in India and by followers of many
faiths across the world. It also said that during celebration clay lamps known as diyas are lit to signify the
victory of good over evil. The festival also symbolises start of new year for many communities.
Note: United States Postal Service (USPS) in October 2016 had launched commemorative stamp in honour
of festival of Diwali. It was issued after seven-years-long efforts and advocacy by Indian-American
community and several Congressional resolutions by influential American lawmakers such as
Congresswoman Carolyn Maloney.
United Nations Postal Administration (UNPA)
It is postal agency of United Nations. It issues postage stamps and postal stationery, denominated in
United States dollars for United Nations offices in New York, in Swiss francs for offices in Geneva and in
euros for the offices in Vienna. Postage rates charged are identical to those of the host nation.
US exempts India from imposition of sanctions development of Chabahar Port in Iran
United States has exempted India from imposition of certain sanctions for development of strategically-
located Chabahar port in Iran, along with construction of railway line connecting it with landlocked
Afghanistan. The decision comes after US imposed toughest ever sanctions on Iran under Iran Freedom
and Counter-Proliferation Act, 2012.
Key Facts
The exemptions to India are with respect to development of Chabahar port, construction of associated
railway and for shipment of non-sanctionable goods through port for Afghanistan’s use, as well as the
country’s continued imports of Iranian petroleum products.
This exception relates to reconstruction assistance and economic development for Afghanistan. These
activities are vital for ongoing support of Afghanistan’s growth and humanitarian relief. It is also part US
President Donald Trump’s South Asia strategy which underscores US’s ongoing support of Afghanistan’s
© 2018 GKToday | All Rights Reserved | https://www.gktoday.in 30
https://t.me/APPSC
https://t.me/UpscMaterials https://t.me/FreeUpscMaterials https://t.me/MaterialForExams
Current Affairs [PDF] -November 1-15, 2018

economic growth and development as well as close partnership with India. This strategy states that India
has major role in bringing peace and development in Afghanistan.
Background
US has imposed toughest ever sanctions on defiant Iran aimed at altering the Iranian regime’s
“behaviour”. The sanctions cover Iran’s banking and energy sectors and reinstate penalties for countries
and companies in Europe, Asia and elsewhere that do not halt Iranian oil imports. However, eight
countries — India, China, Italy, Greece, Japan, South Korea, Taiwan and Turkey — were temporarily
allowed to continue buying Iranian oil as they showed significant reduction in oil purchase from Iran.
India elected as Member of International Telecommunications Union (ITU) Council
India was elected as member of International Telecommunications Union (ITU) Council for another 4-year
term from 2019 to 2022. The elections to ITU Council were held during ongoing ITU Plenipotentiary
Conference 2018 at Dubai, UAE. India got elected to Council from Asia-Australasia region by securing 165
votes and was among 48 countries elected to Council globally. They were elected by 193 member states of
ITU.
International Telecommunication Union (ITU)
ITU is the UN specialised agency for information and communication technologies (ICTs). It has 193
countries as its members and also 800 private-sector entities and academic institutions. It is
headquartered in Geneva, Switzerland. ITUManjul
is responsible for allocating
Tripathi | tripathimanjul18@gmail.com | global radio spectrum and satellite
orbits. It also develops the technical standards that ensure networks and technologies to seamlessly
interconnect. It also strives to improve access to ICTs among the underserved communities worldwide.
November 9, 2018
Cabinet approves signing and ratifying Extradition Agreement between India and Morocco
Union Cabinet chaired by Prime Minister Narendra Modi has approved signing and ratifying Extradition
Agreement between India and Morocco. The agreement will be signed during proposed VIP visit from
Morocco in November 2018.
Benefits of Agreement
It will provide strong legal base for extradition of fugitive offenders who are accused of economic offences,
terrorism and other serious offences in one Contracting State and found in another Contracting State. It
will also strengthen bilateral relations to deal with criminal elements acting against national interest of
both India and Morocco.
Cabinet approves India and Morocco Agreement Mutual Legal Assistance in Civil and Commercial
Matters
Union Cabinet chaired by Prime Minister Narendra Modi has approved Agreement between India and
Morocco on Mutual Legal Assistance in Civil and Commercial Matters. The Agreement will be beneficial
for citizens of both the countries. It will also fulfill desire of both countries to strengthen bonds of
friendship and fruitful cooperation in Civil and Commercial matters. It will also enhance co-operation in
the service of summons, judicial documents, letters of request and the execution of judgments decrees and
arbitral awards.
Salient features of agreement
Service of summons and other judicial documents or processes;
Taking of evidence in civil matters;
Production, identification or examination of documents, recordings;
Execution of Letter of Request for taking of evidence in civil matters; and
Recognition and enforcement of arbitral awards.
India overtakes US to become 2nd largest smartphone market behind China
According to research firm Canalys, India has overtaken US to become second largest smartphone market
in the July-September 2018 quarter. India saw shipment of 40.4 million units during third quarter and was
second to China where 100.6 million smartphones were shipped. Smartphone shipment in US was at 40
million units.
Key Highlights
Worldwide smartphone shipments fell by 7.2% year-on-year to 348.9 million units during July-September
2018, a fourth consecutive quarter of decline. This was also worst third quarter performance since 2015.

© 2018 GKToday | All Rights Reserved | https://www.gktoday.in 31


https://t.me/APPSC
https://t.me/UpscMaterials https://t.me/FreeUpscMaterials https://t.me/MaterialForExams
Current Affairs [PDF] -November 1-15, 2018

Seven of the top ten markets recorded year-on-year (y-o-y) declines, mainly caused by lengthening
smartphone replacement cycles, worsening international trading conditions and competition from major
Chinese vendors.

Manjul Tripathi | tripathimanjul18@gmail.com |

The three markets (among top 10) that registered growth were Indonesia (13.2% increase y-o-y to
8.9 million units), Russia (11.5% jump y-o-y to 8.8 million units) and Germany (2.4% rise y-o-y to 5.5
million units). China’s smartphone shipment declined 15.2% y-o-y, India by 1.1% and US by 0.4% in
September 2018 quarter.
Samsung led vendor tally with 20.4% share in said quarter, followed by Huawei (14.9%), Apple
(13.4%), Xiaomi (9.6%), and Oppo (8.9%). The worldwide smartphone market faces unprecedented
challenge, while its dynamics are changing rapidly at both vendor and country level. This is
providing growth opportunities for aggressive vendors with speed to respond quickly to market
changes.
Defence Ministry enhances financial powers of Vice Chiefs of three Services
Union Defence Ministry has enhanced financial powers of three vice chiefs of the armed forces from Rs
100 crore to Rs 500 crore i.e. five times. This comes after vice chiefs were granted additional powers to
carry out specific procurement to ensure operational preparedness in March 2018.
Key Facts
The decision to enhance financial powers of three vice chiefs of the armed forces was taken to give fillip to
procurement of arms and ammunition and upgrade of defence preparedness. With new delegation, Vice
Chiefs will be now able to exercise financial powers upto five times more than existing powers with
enhanced ceiling.
Cabinet approves setting up of Central Tribal University in Andhra Pradesh
Union Cabinet has approved setting up of Central Tribal University in Andhra Pradesh. The Union Cabinet
meeting was chaired by Prime Minister Narendra Modi in New Delhi. The proposed university will come
up after necessary amendment in the Central Universities Act, 2009.
Central Tribal University
It will be set up in Relli village of Vizianagaram District of Andhra Pradesh. It will be established under
Thirteenth Schedule to Andhra Pradesh Reorganisation Act, 2014 (No. 6 of 2014). Cabinet has also
approved provision of funds of Rs. 420 crore for first phase expenditure towards establishment of Central

© 2018 GKToday | All Rights Reserved | https://www.gktoday.in 32


https://t.me/APPSC
https://t.me/UpscMaterials https://t.me/FreeUpscMaterials https://t.me/MaterialForExams
Current Affairs [PDF] -November 1-15, 2018

Tribal University.
Cabinet approves filling of Padur Strategic Petroleum Reserves in Karnataka
The Union Cabinet chaired by Prime Minister Narendra Modi has approved the filling of Padur Strategic
Petroleum Reserves (SPR) in Karnataka by overseas National Oil Companies (NOCs). The filling of the SPR
will be under PPP model and is being undertaken to reduce budgetary support of Union Government. The
SPR facility at Padur is underground rock cavern with total capacity of 2.5 million metric tonnes (MMT)
having four compartments of 0.625 MMT each.
Background
Indian Strategic Petroleum Reserves Ltd. (ISPRL) has constructed and commissioned underground rock
caverns for storage of total 5.33 MMT of crude oil at three locations namely Mangalore (1.5 MMT),
Vishakhapatnam (1.33 MMT), and Padur (2.5 MMT). The total 5.33 MMT capacity under Phase-I of the SPR
programme is currently estimated to supply approximately 9.5 days of India’s crude requirement.
Government also has given ‘in principle’ approval for establishing additional 6.5 MMT SPR facilities at
Chandikhol in Odisha and Padur in Karnataka in phase II which is expected to augment India’s energy
security by 11.5 days according the consumption data for FY 2017-18.
Cabinet approves leasing out six airports through PPP
Union Cabinet chaired by the Prime Minister Narendra Modi has approved leasing out six airports of
Airports Authority of India (AAI) for operation, management
Manjul Tripathi | tripathimanjul18@gmail.com | and development under Public Private
Partnership (PPP) in the first phase. This will be done through Public Private Partnership Appraisal
Committee (PPPAC).
Union Cabinet also approved constitution of Empowered Group of Secretaries headed by CEO, NITI Aayog
with Secretaries of Ministry of Civil Aviation, Department of Economic Affairs and Department of
Expenditure to decide on any issue falling beyond scope of PPPAC.
Benefits
PPP in infrastructure projects brings efficiency in service delivery, enterprise, expertise and
professionalism apart from harnessing needed investments in public sector. The PPP in airport
infrastructure projects has brought world class infrastructure at airports, delivery of efficient and timely
services to airport passengers, augmenting revenue stream to AAI without making any investment, etc. of
these, for development of Greenfield Airports.
Presently, Delhi, Mumbai, Bangalore, Hyderabad and Cochin airports are being managed under PPP
model.PPP airports in India have been ranked among top 5 in their respective categories by Airports
Council International (ACI) in terms of Airport Service Quality (ASQ). These PPP experiments have helped
to create world class airports and also helped AAI in enhancing its revenues and focusing on developing
airports and air navigation infrastructure in rest of the country.
CCEA approves Strategic Sale of Government Equity in Dredging Corporation Of India
Cabinet Committee on Economic Affairs (CCEA) has approved strategic disinvestment of 100%
Government of India’s shares in Dredging Corporation of India Limited (DCIL) to consortium of four ports.
Presently, Central Government holds 73.44% shares in DCIL. The consortium of four ports consists of
Vishakhapatnam Port Trust (Andhra Pradesh), Paradeep Port Trust (Odisha), Jawahar Lal Nehru Port Trust
(Maharashtra) and Kandla Port Trust (Gujarat). The government’s divestment target for fiscal 2018-19 was
Rs 80,000 crore and so far it has garnered over Rs 15,000 crore from PSU stake sales.
Significance
Strategic sale of DCIL will further facilitate linkage of dredging activities with ports, keeping in view the
role of DCIL in expansion of dredging activity in the country as well as potential scope for diversification
of ports into third party dredging. The co-sharing of facilities between company as well as ports shall lead
to savings for ports. This will also further provide opportunities for larger investment in DCIL as
integration with ports shall help ineffective vertical linkage in value chain.
Dredging Corporation of India Limited (DCIL)
It is miniratna public sector unit (PSU) engaged in the business of dredging. It does dredging for Indian
seaports exclusively. It is involved in capital dredging, beach nourishment, and land reclamation. It was
established in March 1976 and is headquartered in Visakhapatnam, Andhra Pradesh. It reports to the

© 2018 GKToday | All Rights Reserved | https://www.gktoday.in 33


https://t.me/APPSC
https://t.me/UpscMaterials https://t.me/FreeUpscMaterials https://t.me/MaterialForExams
Current Affairs [PDF] -November 1-15, 2018

Ministry of Shipping. Almost all maintenance dredging in Indian seaports is carried out by DCI. It also
occasionally dredges at foreign seaports in countries such as Sri Lanka, Taiwan and Dubai.
November 10, 2018
Jawahar Lal Sarin presented with France’s highest civilian honour
Renowned French scholar Jawahar Lal Sarin was presented with France’s highest civilian honour
Chevalier de la Lgion d’Honneur. He was presented with this award in recognition of his outstanding
contribution to enhancing Indo-French cultural cooperation and promoting French language. Jawahar Lal
Sarin is president of Alliance franaise de Delhi’s governing body. He had studied French for five years at
the Punjab University, Chandigarh, and later at the Sorbonne University, Paris.
Legion d’Honneur (Legion of Honour)
It is the highest civilian award given by the French government to person for his outstanding service to
France, regardless of the nationality. It was instituted in 1802 by Napoleon Bonaparte. The order (merit for
both military and civil merits) is divided into five degrees of increasing distinction: Chevalier (Knight),
Officier (Officer), Commandeur (Commander), Grand officier (Grand Officer), and Grand-croix (Grand Cross).
Actors Amitabh Bachchan and Shahrukh Khan were also awarded in 2007 and 2014. Other Indians who
were also awarded are Amartya Sen, Ravi Shankar, Zubin Mehta, Lata Mangeshkar, JRD Tata and Ratan
Tata among others.
National Legal Services
Manjul Tripathi Day: November
| tripathimanjul18@gmail.com | 9
National Legal Services Day (NLSD) is observed every year in India on 9th November to spread awareness
for ensuring reasonable fair and justice procedure for all citizens.
The aim of celebrating this day is to offer free of charge, proficient and legal services to the people
belonging to weaker sections of society. It also seeks to make sure availability of free services to weaker
section people as well as making them conscious about their rights.
On this day, Lok Adalats are organized across the country to make safe the legal system operations and
encourages righteousness of people on the equality basis. Besides, variety of legal literacy camps and
functions are also organized to increase awareness of free legal aid towards every weak citizen of the
country.
National Legal Services Day (NLSD)
It was started by Supreme Court of India in 1995 to provide help and support to poor and weaker sections
of the society. It was established with mandate to offer help and support to weaker and poor group of
sections of people which may be women, disabled persons, Scheduled Tribes (STs), children, Scheduled
Castes (SCs), human trafficking victims as well as natural calamities victims.
CIMAP signs agreement with US-based RIFM for improving the quality of fragrant oils
Central Institute of Medicinal and Aromatic Plants (CIMAP) has signed MoU with US-based Research
Institute for Fragrant Materials (RIFM) for improving the quality of fragrant oils. It was signed at
conclusion of two-day annual conference of International Fragrance Association (IFA) in Paris, capital city
of France. Team of RIFM had visited the Lucknow headquarter of CIMAP earlier in 2018 to increase the
acceptability and demand of Indian fragrances on a global level. The deliberations during the visit resulted
in signing of the MoU.
Benefits of MoU
It will go long way in standardizing fragrant material produced in India and bring it to an international
level. It will give a big boost to fragrance quality in India as RIFM internationally standardizes the quality
of fragrant materials. It will give impetus to aroma mission of Council of Scientific & Industrial Research
(CSIR) which aims at enhancing production of fragrant materials.
Central Institute of Medicinal and Aromatic Plants (CIMAP)
Central Institute of Medicinal and Aromatic Plants (CIMAP) is a frontier plant research laboratory of CSIR.
It was originally established as Central Indian Medicinal Plants Organisation (CIMPO) in 1959. It is
headquartered in Lucknow, Uttar Pradesh. It has four research centers situated in Bangalore, Hyderabad,
Pantnagar and Purara (near Bageshwar, Uttarakhand). It is engaged in the field of science and business of
medicinal and aromatic plants (MAPs). It steers multidisciplinary high quality research in biological and
chemical sciences and extending technologies and services to farmers and entrepreneurs of MAPs.

© 2018 GKToday | All Rights Reserved | https://www.gktoday.in 34


https://t.me/APPSC
https://t.me/UpscMaterials https://t.me/FreeUpscMaterials https://t.me/MaterialForExams
Current Affairs [PDF] -November 1-15, 2018

Poor-Quality diet greater public health threat than malaria, tuberculosis or measles: FAO
According to recently released report “Preventing nutrient loss and waste across the food system: Policy
actions for high-quality diets”, one-in-five deaths is associated with poor-quality diets. It shows that
regularly eating poor-quality food has become greater public health threat than malaria, tuberculosis or
measles. The report was published by United Nations Food and Agriculture Organization (FAO).
As per FAO Report
More than half of all globally-produced fruits and vegetables are lost or wasted annually. Around 25% of all
meat produced, equivalent to 75 million cows, goes uneaten. About one-third of food produced for human
consumption never reaches consumer’s plate. Global food loss or waste annually is estimated to be around
$1 trillion.
Nutrient-rich foods, like fruits, vegetables, seafood, meats are highly perishable, so susceptible to losses
throughout increasingly complex food production systems. Globally, agriculture produces 22% more
vitamin A required for human consumption, but after loss and waste, amount available is merely 11% per
cent less than required.
In Low-income nations, food is mostly lost during harvesting, storage, processing and transportation
while in high-income nations problem lies in retail and consumer level waste. Together, they directly
impact number of calories and nutrients actually available for consumption. Reducing food loss and waste,
mainly high-nutrient foods, has nutritional benefits and also contributes to Sustainable Development
Manjul Tripathi | tripathimanjul18@gmail.com |

Goals (SDGs).
FAO recommended solutions
Policymakers need to reduce food loss and waste in order to improve access to nutritious and healthy
food. Food systems that increase availability, affordability and consumption of fresh, nutrient-rich food for
everyone must be put in place to tackle all forms of malnutrition and to promote healthy diets.
Series of policy actions should be taken across entire food system, including educating all concerned,
focusing on perishable foods, improving public and private infrastructure and closing data gaps on food
losses and waste.
Reducing loss and waste of nutritious foods could yield substantial health benefits, given direct impact on
wellbeing, learning capacity and productivity. Cutting down on food waste will yield major economic
benefits. Besides, eating more of food already produced, will avoid wasting water, land and energy that
went into its production.
Food and Agriculture Organization (FAO)
It is specialised agency of UN that leads international efforts to defeat hunger. Its parent organization is
UN Economic and Social Council (UNESC). It was established on 16 October 1945 and its headquarters are
in Rome, It has 197 member states, along with European Union (member organization).
PK Dash, Akhilesh Ranjan and Neena Kumar inducted as members of CBDT
Appointments Committee of the Cabinet (ACC) has appointed three senior IRS officers K Dash, Akhilesh
Ranjan and Neena Kumar as new members of Central Board of Direct Taxes (CBDT). All three members
belong to 1982-batch of the Indian Revenue Service (IRS). Prior to this appointment, Dash was serving as
principal chief commissioner of I-T (PCCIT) in Bhopal, Ranjan and Kumar were serving as PCCsIT in Delhi.
With these three new joinings, CBDT now has full house strength of five members and chairman. The two
other members of CBDT are IRS officers Aditya Vikram and P C Mody while the borad’s Chairman is
Sushil Chandra.
Central Board of Direct Taxes (CBDT)
CBDT is nodal policy-making body of Income Tax (IT) department under Finance Ministry. It is a statutory
authority established under The Central Board of Revenue Act, 1963. It is supreme body in India for
framing policies related to direct taxes and is also tasked to enforce direct tax laws in the country. The
composition of CBDT includes Chairman and six members.
Ashok Kumar Gupta appointed as Chairperson of Competition Commission of India
Appointments Committee of the Cabinet (ACC) headed by Prime Minister Narendra Modi has appointed
former IAS officer Ashok Kumar Gupta as Chairperson of Competition Commission of India (CCI). He
replaces acting chairperson Sudhir Mital. He will be in office till October 25, 2022 i.e. till he attains age of

© 2018 GKToday | All Rights Reserved | https://www.gktoday.in 35


https://t.me/APPSC
https://t.me/UpscMaterials https://t.me/FreeUpscMaterials https://t.me/MaterialForExams
Current Affairs [PDF] -November 1-15, 2018

65 years or until further orders, whichever is the earliest. Gupta is former IAS officer of the Tamil Nadu
cadre.
Competition Commission of India (CCI)
CCI is quasi-judicial statutory body established under The Competition Act, 2002. It was established in
October 2003 and became fully functional in May 2009. Section 8(1) of the Act provides CCI shall consist of
Chairperson and not less than two and not more than six Members. Presently, Chairperson and four
Members are in position. It has been functioning as collegium right from its inception. It functions under
Ministry of Cooperate Affairs.
CCI aims to eliminate practices that adversely affect competition in different industries and protect
interests of consumers and ensure freedom of trade. Its seeks to eliminate practices having adverse effect
on competition, promote and sustain competition, protect interests of consumers and ensure freedom of
trade in markets of India. It also gives opinion on competition issues on reference received from statutory
authority established under any law and undertakes competition advocacy, create public awareness and
impart training on competition issues.
SIMBEX: Singapore-India Maritime Bilateral Exercise to be held off Andaman Sea and Bay of Bengal
India and Singapore will hold 25th joint naval exercise SIMBEX, acronym for Singapore-India Maritime
Bilateral Exercise from November 10 to 21 in India off Andaman Sea and Bay of Bengal. SIMBEX 2018 will
mark silver jubilee of exercise and will be largest edition since 1994 in terms of scale and complexity.
Manjul Tripathi | tripathimanjul18@gmail.com |

SIMBEX 2018
It will witness diverse range of exercises at sea such as live weapon drills including multiple missile
firings, heavy weight torpedo (HWT) and medium range gun and anti-submarine rocket firings; advanced
anti-submarine warfare (ASW) exercises, submarine rescue demonstrations; integrated surface and anti-
air warfare (AAW); unmanned aerial vehicle (UAV) operations and cross deck helicopter flying among
others.
The initial first harbour phase of the exercise will be held at Port Blair followed by Sea Phase in Andaman
Sea. Second harbour phase will be held at Visakhapatnam. Final sea phase will be held in Bay of Bengal.
The number of missiles and torpedo firings in this exercise will be largest ever undertaken by Indian Navy
with any foreign Navy till date. It will also witness highest numbers and variety of platforms fielded by
both sides.
Indian Navy for this exercise will be deploying Ranvir class destroyer — INS Ranvijay, two Project 17
multirole stealth frigates — INS Satpura and INS Sahyadri, Project 28 ASW corvette — INS Kadmatt,
Project 25A missile corvettes, INS Kirch, OPVs INS Sumedha and INS Sukanya, Fleet Support Ship, INS
Shakti, Sindhughosh Class submarine, INS Sindhukirti, P8I Long Range Maritime Patrol and ASW aircraft
from INAS 312, Dornier 228 Maritime Patrol aircraft from INAS 311, Mk 132 Hawk AJTs from INAS 551 and
integral helicopters such as the UH3H, Seaking 42 B, Seaking 42C and Chetak utility helicopters.
Singapore Navy will be deploying represented by two Formidable Class stealth frigates — RSS Formidable
and RSS Steadfast, two Missile Corvettes, RSS Vigour and RSS Valiant, one Littoral Mission Vessel — RSS
Unity, Archer class Submarine, RSS Swordsman, ‘Swift Rescue’ Deep Sea Rescue Vehicle (DSRV), Fokker
F50 maritime reconnaissance aircraft, embarked S70B helicopters and Scan Eagle unmanned aerial
systems.
Long cohabitation is presumed marriage: SC
Supreme Court has upheld presumption that couple who live together as husband and wife are legally
married and woman can claim maintenance under Section 125 of Code of Criminal Procedure (CrPC ). The
apex court held that law presumes in favour of marriage and against concubinage when man and woman
have cohabited continuously for a number of years.
Court Ruling
Man who lived with woman for long time and even though they may not have undergone legal necessities
of valid marriage is liable to pay the woman maintenance if he deserts her. In this case, man should not be
allowed to benefit from legal loopholes by enjoying advantages of de facto marriage without undertaking
duties and obligations. Any other interpretation in this case will lead woman to vagrancy and destitution,
which provision of maintenance in Section 125 of CrPC is meant to prevent.

© 2018 GKToday | All Rights Reserved | https://www.gktoday.in 36


https://t.me/APPSC
https://t.me/UpscMaterials https://t.me/FreeUpscMaterials https://t.me/MaterialForExams
Current Affairs [PDF] -November 1-15, 2018

Background
This judgment of SC was based on appeal filed by woman against Karnataka High Court decision of June
2009. The High Court had set aside family court order, directing man she lived with since 1998 and had
two children by to pay maintenance. Their relationship in this case was solemnised in a temple and
husband of woman had later abandoned the family. The family court had ordered him to pay maintenance
to woman and their children on monthly basis. This court had held the couple is accepted as husband and
wife by society. The man had however had moved appeal in High Court, which pronounced that there was
no proof that she was his legally-wedded wife.
Indian Army inducts 3 artillery gun system
Indian Army inducted three major globally combat proven artillery gun system viz. M777 A2 ultra light
howitzers (ULH), K-9 Vajra-tracked self-propelled guns and ‘Composite Gun Towing Vehicle’ at Deolali Field
Firing Range in Maharashtra. These are first artillery gun systems to be inducted by Indian Army in three
decades. Last inducted was Bofors artillery gun system procured from Sweden in early 1980s.
Key Facts
M777 A2 Ultra Light Howitzers : It is 155mm, 39 Calibre ULHs towed artillery gun, with maximum range
of 24-30km procured from US. It being versatile and light weight can be heli-lifted, thus providing
employment flexibility in different terrains.
It will be procured under India-US deal signed in
Manjul Tripathi November 2016
| tripathimanjul18@gmail.com | for 145 M777 Ultra-Light Howitzers.
Under it, 25 guns will be imported and rest will be assembled in India by BAE Systems in partnership with
Mahindra Defense.
Deliveries will be completed by mid-2021. The gun systems induction gives a major impetus to Make In
India initiative. It will bring new level of capability to artillery unit by offering rapid deployment and
extreme accuracy.
K9 Vajra guns: It is a 155-mm, 52-calibre self-propelled gun with maximum range of 40 km. It will be
imported from South Korea. It is customized version of original K9 Thunder artillery gun for desert
conditions. Total 100 units will be procured by Army, and first 10 will be imported in semi knocked down
form from South Korea and assembled by L&T in India and rest 90 will be manufactured in India. The first
regiment of K9 will be equipped by July 2019 and all 100 guns will be delivered by November 2020.
Common Gun Tower: It is 6×6 field artillery tractor is indigenously developed by Ashok Leyland. It with
has cross country capability and will replace ageing fleet of artillery gun towing vehicles.
Ladakh restoration project wins 2018 UNESCO Asia-Pacific award for Cultural Heritage conservation
The restoration of an aristocratic house project from partial ruin state in Ladakh undertaken by LAMO
(Ladakh Arts and Media Organisation) Center has won 2018 UNESCO Asia-Pacific award for Cultural
Heritage conservation in Award of Distinction category. LAMO is public charitable trust established to
articulate an alternative vision for the arts and media in Ladakh.
Key Facts
The restored structure is strategically located at the foot of Leh palace, contributes to continuity of
historic townscape of 17th century. Its recovery establishes precedent for conserving non-monumental
urban fabric of Leh Old Town. Under restoration project, LAMO had used salvaged and local building
materials and indigenous construction techniques and skillfully introduced modern amenities to assure
its ongoing use.
Other Category awardees
Award of Excellence: It was awarded to restoration project of early 20th-century Shijo-cho Ofune-
hoko Float Machiya in Japan. It celebrates Kyoto culture by safeguarding city’s endangered wooden
townhouse typology as well as iconic annual Gion Festival.
Award of Distinction: LAMO Center, Ladakh, India
Award of Merit: 5 Martin Place, Sydney (Australia); Aijing Zhuang, Fujian (China); Commercial
Bank of Honjo Warehouse, Saitama (Japan).
Honourable Mention: Hengdaohezi Town, Heilongjiang (China), Rajabai Clock Tower & University of
Mumbai Library Building, Mumbai (India), Ruttonsee Muljee Jetha Fountain, Mumbai (India).
New Design in Heritage Contexts: Kaomai Estate 1955, Chiang Mai (Thailand) and Harts Mill, Port

© 2018 GKToday | All Rights Reserved | https://www.gktoday.in 37


https://t.me/APPSC
https://t.me/UpscMaterials https://t.me/FreeUpscMaterials https://t.me/MaterialForExams
Current Affairs [PDF] -November 1-15, 2018

Adelaide, (Australia).
UNESCO Asia-Pacific Awards for Cultural Heritage Conservation
It was established in 2000. It recognizes efforts of private individuals and organizations that have
successfully conserved structures and buildings of heritage value in the region. It aims to encourage other
property owners to undertake conservation projects within their communities, either independently or by
seeking PPP (public-private partnerships).
November 11-12, 2018
7 people including late singer Elvis Presley awarded Presidential Medal of Freedom
United States President Donald Trump will present 7 distinguished individuals including late singer Elvis
Presley 2018 Presidential Medal of Freedom on November 16, 2018.
These seven awardees includes three posthumous recipients- Music legend Elvis Presley, baseball great
Babe Ruth and late Supreme Court justice Antonin Scalia. Other four are philanthropist and heavyweight
political donor Miriam Adelson and two football Hall of Famers- quarterback Roger Staubach and
defensive tackle Alan Page, who later became first African American to serve on Minnesota Supreme
Court.
About Presidential Medal of Freedom
It is United States’ highest civilian award. It is awarded to individuals with exceptional meritorious service
to security or national interests of US, Manjul
world
Tripathi peace, cultural
| tripathimanjul18@gmail.com | or other significant public or private
endeavors. It was established in 1945 by President Truman, to recognize notable service in war. In 1963,
President Kennedy reintroduced it as an honor for distinguished civilian service in peacetime. Majority of
awardees are US citizens, but individuals from other countries are also eligible.
Sri Lankan PM leaves Presidents Party, joins newly-formed Sri Lanka People’s Party
Newly appointed Sri Lankan Prime Minister Mahinda Rajapaksa ended his five-decade-long association
with Sri Lanka Freedom Party (SLFP) and joined newly-formed Sri Lanka People’s Party (SLPP). SLFP was
founded by his father Don Alwin Rajapaksa in 1951. Rajapaksa’s joining SLPP signals that he will contest
snap polls, under his own party banner and not that of President Maithripala Sirisena’s Sri Lanka Freedom
Party (SLFP). SLPP was formed in 2017 by Rajapaksa’s supporters as platform for his re-entry into politics.
In February 2018 local council elections SLPP won two-third of total 340 seats.
Sri Lankan Political and Constitutional Crisis
In January 2015 Presidential election, then President Mahinda Rajapaksa (was in office for decade from
2005) was unexpectedly defeated by his then deputy Sirisena with support from Ranil Wickremesinghe’s
United National Party (UNP). But, power-sharing arrangement between President Sirisena and PM
Wickremesinghe in later times became increasingly uncertain on several policy matters, mainly on
economy and security.
Further rift had developed between them over policy towards China and India, with Wickremesinghe
favouring Indian investment as counter to Chinese inroads in Sri Lankan infrastructure projects and
Sirisena’s intention to contest 2020 presidential election under Mr Wickremesinghe’s party.
In October 2018 Sirisena had abruptly removed Wickremesinghe and replaced with Rajapaksa, a pro-China
strongman as new Prime Minister. This move had plunged Sri Lanka into political and constitutional
crisis. Sirisena had suspended parliamentary proceedings and later dissolved it, after it became evident
that there is not enough support in House to prove Rajapaksa’s premiership.
Global IT Challenge for Youth with Disabilities held in New Delhi
The Global IT Challenge for Youth with Disabilities (GITC) 2018 held in New Delhi from November 9 to 11,
2018. It was co-hosted by Department of Empowerment of Persons with Disabilities (DEPwD), Ministry of
Social Justice, Ministry of Health and Family Welfare, Government of Korea and its organising partners
namely Rehabilitation International Korea and LG Group.
Key Facts
GITC is being held every year in Asia- Pacific region and such events were held in past in countries like
South Korea, China, Thailand, Vietnam etc. The objective of GICT was to leverage IT skills among youth
with disabilities and also to spread awareness about application of Information and Computer Technology
in enhancing the quality of life of persons with disabilities. The event saw participation around 300

© 2018 GKToday | All Rights Reserved | https://www.gktoday.in 38


https://t.me/APPSC
https://t.me/UpscMaterials https://t.me/FreeUpscMaterials https://t.me/MaterialForExams
Current Affairs [PDF] -November 1-15, 2018

participants including 120 Youth with disabilities from 24 Asia Pacific countries. It also had IT Forum in
which countries will present their work and achievement in ensuring accessibility to persons with
disabilities in IT sector.
National Education Day: 11 November
National Education Day of India is celebrated every year on 11 November to commemorate the birth
anniversary of Maulana Abul Kalam Azad, the first education minister of independent India. This year it
was 130th birth anniversary of Maulana Abul Kalam Azad. As first education minister of India, Azad had
oversaw establishment of national education system with free primary education and modern institutions
of higher education. He is also credited with establishment of Indian Institutes of Technology (IITs) and
laid foundation of University Grants Commission, an important institution to supervise and advance
higher education throughout the country.
Background
National Education Day was instituted by Union Ministry of Human Resource Development in 2008
announced to commemorate the birthday Maulana Abul Kalam Azad for recalling his contribution to
cause of education in India. Its objective is to strengthen educational institutions and to raise quality of
education to greater heights. It also seeks to remember Maulana Azad’s contribution in laying the
foundations of education system in an independent India as well as to evaluate our current performance in
this field. Manjul Tripathi | tripathimanjul18@gmail.com |

Maulana Abul Kalam Azad


Born on 11 November 1888 and died on 22 February 1958. He was senior Muslim leader of Indian National
Congress during Indian independence movement. He is commonly remembered as Maulana Azad (word
Maulana is honorific meaning ‘Our Master’) and he had adopted Azad (Free) as his pen name. He was first
Minister of Education of Independent India. He had played important role in foundation of Jamia Millia
Islamia at Aligarh in Uttar Pradesh. He assisted in shifting campus of university from Aligarh to New
Delhi in 1934.
Freedom Movement
During his young age, he rose to prominence through his work as journalist, publishing works critical of
British Raj and espousing causes of Indian nationalism. He became leader of Khilafat Movement, during
which he came into close contact with Mahatma Gandhi. He later became enthusiastic supporter of
Gandhi’s ideas of non-violent civil disobedience and worked to organise the non-co-operation movement
in protest of the 1919 Rowlatt Acts.
He was committed to Gandhi’s ideals, including promoting Swadeshi (indigenous) products and cause of
Swaraj (Self-rule) for India. In 1923, at age of 35, he became youngest person to serve as President of Indian
National Congress. He was one of main organizers of the Dharasana Satyagraha in 1931 and emerged as
one of the most important national leaders of the time, prominently leading causes of Hindu-Muslim unity
as well as espousing secularism and socialism. He served as Congress president from 1940 to 1945, during
which Quit India rebellion was launched. He also worked for Hindu-Muslim unity through the Al-Hilal
newspaper.
INSPIRE 2018 organized in New Delhi by EESL and World Bank
The second edition of was International Symposium to Promote Innovation & Research in Energy
Efficiency (INSPIRE 2018) launched in New Delhi. It was jointly organised by Energy Efficiency Services
Limited (EESL) and World Bank. The three-day symposium focused on enhancing grid management, e-
Mobility, financial instruments and technologies for energy efficiency in India. Moreover awards to four
pathbreaking innovations in clean energy and energy efficiency were also given as part of
#InnovateToINSPIRE, a first-of-its-kind energy innovation challenge.
Key Facts
#InnovateToINSPIRE challenge was organized by EESL and World resources Institute (WRI) in August-
October 2018 in run-up to INSPIRE 2018. The challenge had invited participants to submit sustainable and
scalable solutions to seven specific challenges spanning grid management, e-Mobility, energy efficient
technologies and financial instruments.
Four winners were selected from 94 entries by eminent jury comprised of Indian and international
© 2018 GKToday | All Rights Reserved | https://www.gktoday.in 39
https://t.me/APPSC
https://t.me/UpscMaterials https://t.me/FreeUpscMaterials https://t.me/MaterialForExams
Current Affairs [PDF] -November 1-15, 2018

experts in the field of energy. The winning entries received award of Rs. 5 lakhs, each, along with
mentoring and guidance from EESL to help them bring their solutions to market.
Energy Efficiency Services Limited (EESL)
EESL was set up under Union Ministry of Power to facilitate implementation of energy efficiency projects.
It is a joint venture of NTPC Limited, Power Finance Corporation (PFC), Rural Electrification Corporation
(REC) and POWERGRID. It leads market-related actions of National Mission for Enhanced Energy
Efficiency (NMEEE). It also acts as the resource centre for capacity building of State DISCOMs. It is also
implementing the world’s largest energy efficiency portfolio in the country.
ADB to provide US $13 million loan to EESL to promote efficient energy usage in India
Asian Development Bank (ADB) it will provide US $13 million loan to Energy Efficiency Services (EESL) to
promote efficient energy usage in India. It will be provided by Global Environment Facility (GEF) as part of
US $200 million ADB-financed Demand-Side Energy Efficiency Sector Project.
Key Facts
ADB funding is helping EESL finance energy service utilities for installing established energy-efficient
technologies such as light-emitting diode (LED) street-lights with municipalities, while exploring newer
technologies and their self-sustaining business models.
The additional financing for project through GEF grant will be used by EESL for making investments in
new and emerging technologies, including trigeneration,
Manjul efficient
Tripathi | tripathimanjul18@gmail.com | motors and air-conditioners, smart
meters/grids and associated new business models to expand use of such innovations as well as to set up v
Asian Development Bank (ADB)
ADB is a regional development bank based out of Asia. It aims to promote social and economic
development in Asia by achieving a prosperous, inclusive, resilient, and sustainable Asia and the Pacific,
while sustaining its efforts to eradicate extreme poverty. It was established in December 1966. It is
headquartered at Ortigas Centre in Manila, Philippines. It has total 67 members, of which 48 are from
within Asia and the Pacific and 19 outside.
IIT-Madras develops portable cold storage device
Indian Institute of Technology (IIT) Madras has developed portable solar-powered cold storage device with
500 kg capacity for storing vegetables and fruits, among other items. The device will help farmers store
their produce for sufficient time so that it does not get spoilt before it is sold, thereby preventing wastage
of agricultural produce. The first unit of device was deployed at farm in Madurantakam in Kancheepuram
district in Tamil Nadu.
Solar-powered cold storage device
It has been developed under project funded by Department of Science and Technology (Ministry of Science
and Technology) and IIT-M. It provides storage facility with temperature ranging between four and 10
degree Celsius. Using it, vegetables, leaves, flowers and fruits can be stored for 24 hours in the device.
Each device is enough for 5 or 6 acres of land or for group of three to four farmers. The unit uses thermal
energy storage. The photovoltaic-powered refrigeration system is used to charge indoor thermal battery
in form of latent heat during day time and used to maintain temperature (10 C) for 20-24 hours. Each unit
costs between Rs 5.5 to Rs 6 lakh and storage capacity could be scaled down based on requirement.
SpiNNaker: Largest brain-mimicking supercomputer switched on
The world’s largest supercomputer Spiking Neural Network Architecture (SpiNNaker) machine designed
to work in same way as human brain was switched on for the first time.
SpiNNaker
It designed and built at University of Manchester in UK. It can model more biological neurons in real time
than any other machine on planet. Biological neurons are basic brain cells present in nervous system that
communicate by emitting ‘spikes’ of pure electro-chemical energy. It is neuromorphic computing that
uses large scale computer systems containing electronic circuits to mimic these spikes in a machine.
It is capable of completing more than 200 million million actions per second, with each of its chips having
100 million transistors. It is unique as it mimics the massively parallel communication architecture of
brain, sending billions of small amounts of information simultaneously to thousands of different
destinations. It also unique because, unlike traditional computers, it does not communicate by sending
large amounts of information from point A to B via standard network.
© 2018 GKToday | All Rights Reserved | https://www.gktoday.in 40
https://t.me/APPSC
https://t.me/UpscMaterials https://t.me/FreeUpscMaterials https://t.me/MaterialForExams
Current Affairs [PDF] -November 1-15, 2018

Potential Applications
It has massive potential for neurological breakthroughs in science such as pharmaceutical testing. It can
be used to harnessed to control robot which can interpret real-time visual information and navigate
towards certain objects while ignoring others.
It can also help to unlock some of secrets of how human brain works by running unprecedentedly large
scale simulations. It will allow roboticists to design large scale neural networks into mobile robots so they
can walk, talk and move with flexibility and low power.
November 13, 2018
World Pneumonia Day: November 12
World Pneumonia Day is observed globally every year on November 12 to generate awareness about
‘pneumonia’ disease. The observance of day aims to highlight the severity of pneumonia and bring
together people from all over world to promote the prevention and treatment of the disease.
Significance of Day
Raise awareness about pneumonia, the world’s leading killer of children under age of five.
Promote interventions to protect against, prevent and treat pneumonia and
Generate action to combat pneumonia.
Background
World Pneumonia Day was first observed in 2009
Manjul after more than
Tripathi | tripathimanjul18@gmail.com | 100 organisations joined together form
Global Coalition against Child Pneumonia. At present, global coalition comprises of over 140 NGOs,
academic institutions, government agencies and foundations.
Pneumonia
Pneumonia is form of acute respiratory infection that affects the lungs. It is caused by inflammation of the
airspaces in the lungs. Lungs are made up of small sacs called alveoli, which fill with air when healthy
person breathes. When an individual has pneumonia, alveoli are filled with pus and fluid, which makes
breathing painful and limits oxygen intake.
Causes: It is caused by number of infectious agents, including viruses, bacteria and fungi. The most
common are Streptococcus pneumoniae (most common cause of bacterial pneumonia in children),
Haemophilus influenzae type b (Hib) (second most common cause of bacterial pneumonia), Respiratory
syncytial virus is most common viral cause of pneumonia.
Treatment: Vaccinations are available against several common organisms that are known to cause
pneumonia. Besides, antibiotics can also treat pneumonia by controlling the bacterial or fungal infection.
Public Service Broadcasting Day: November 12
Public Service Broadcasting Day is observed every year across India on November 12 year to
commemorate first and last visit of Mahatma Gandhi (Father of the Nation) of All India Radio Studio (AIR),
New Delhi in 1947. On this occasion, artistes of Akashvani Bhawan, Delhi performed Vaishnav Jan Toh
and other bhajans and school children performed Rabindra sangeet.
Key Facts
Gandhiji had visited AIR studio to address displaced people from Pakistan who were temporarily settled in
Kurukshetra, Haryana after partition of Indian subcontinent. It was his first and last live broadcast. In this
broadcast, Gandhiji had decided to convey his message through radio, as he couldn’t visit refugees of
Partition stationed at Kurukshetra in Haryana. This day was instituted in 2000 as Jan Prasaran Diwas
(Public Service Broadcasting Day) after it was conceptualised by Suhas Borker, Convenor of Jan Prasar.
This day seeks to spread Gandhiji belief that aim of media was public service. Accordingly, he used to run
several newspapers and understood power of media. He was completely against commercial ads and
believed that only those non-commercial ads should be accepted that serve some public purpose.
India and Morocco sign Agreement on Mutual Legal Assistance in Criminal Matters
India and Morocco sign an agreement on mutual legal assistance in criminal matters, mainly aimed at
containing menace of terrorists funding. It was signed in New Delhi by Union Minister of State (MoS) for
Home Affairs Kiren Rijiju from the Indian side while Minister of Justice Mohamed Aujjar signed on behalf
of Morocco government.
Key Facts
The agreement will help to strengthen bilateral cooperation between India and Morocco. It will enhance
© 2018 GKToday | All Rights Reserved | https://www.gktoday.in 41
https://t.me/APPSC
https://t.me/UpscMaterials https://t.me/FreeUpscMaterials https://t.me/MaterialForExams
Current Affairs [PDF] -November 1-15, 2018

effectiveness and provide broad legal framework for prevention, investigation and prosecution of crimes
as well as in tracing, restraint and confiscation of funds meant to finance terrorist acts. It will help in
countering threats posed by organized crime and terrorism.
Indian Health Fund: 4 innovators selected for early detection of TB, malaria by Tata Trust and The Global
Fund
Tata Trust and The Global Fund backed Indian Health Fund (IHF) has chosen four innovators for early
detection of tuberculosis (TB), malaria to effectively tackle these eradicate diseases. Three selected
innovators are related to TB and one to malaria. They were selected from proposals received for its second
Request for Proposals.
Key Facts
The aim for selecting these innovators aims at leveraging technology for improving early detection and
prompt treatment along with promoting diagnostics feasible for primary health care facilities. It also aims
to supporting national efforts to effectively tackle eradicate these diseases which is crucial health
challenges in India. Government has set a target of eliminating TB by 2025 and malaria by 2030. India
contributes to 27% of the global TB burden and 68% of all malaria cases in the Southeast Asia region
Indian Health Fund (IHF)
IHF was launched in 2016 by Tata Trusts in collaboration with The Global Fund, to drive innovations
towards key infectious disease challenges in India. It supports individuals and organisations with already
Manjul Tripathi | tripathimanjul18@gmail.com |

germinated innovative strategies, services, products, such that they become sustainable and scalable
solutions in addressing TB and malaria. The initiative is not a fellowship programme to do research from
scratch. It is long-term exercise aligned with country’s goal of eliminating TB by 2025 and malaria by
2030. It will promote innovative solutions such that they are widely accessible and are affordable.
Twitter launches #PowerOf18 campaign to boost youth engagement for 2019 polls
Twitter India has launched #PowerOf18 campaign aimed at encouraging youth to contribute in public
debates and participate in civic engagement for the 2019 general elections. The initiative was launched by
Twitter CEO Jack Dorsey and Maya Hari, Vice-President and Managing Director, Asia Pacific, Twitter
during townhall chat with the students of IIT Delhi.
#PowerOf18 campaign
The campaign will serve as resource for young Indians to find more information about elections, support
social causes they are passionate about and join public conversation. It is supported by Twitter’s mission
to serve the public conversation and work with more like-minded partners and personalities to create
awareness of importance of voting.
Background
In a nationwide survey conducted with 3,622 youth respondents from India, Twitter found rising
importance of social media for information consumption. It found that overwhelming 94% of respondents
indicating that they would vote in the upcoming elections. Nearly 60% of youth surveyed would turn to
social media to find out what’s happening in India and around world, a percentage higher than all other
mediums including newspapers.
Global Cooling Innovation Summit held in New Delhi
The first Global Cooling Innovation Summit was held in New Delhi. It was inaugurated by Union
Environment Minister Harsh Vardhan. It was jointly organised by Department of Science and Technology
along with Alliance for An Energy-Efficient Economy (AEEE), Rocky Mountain Institute, Conservation X
Labs and CEPT University.
Global Cooling Innovation Summit
The two-day summit was first-of-its-kind solutions-focused event organised to explore concrete means
and pathways to address climate threat that comes from the growing demand from room air conditioners.
It witnessed participation from distinguished speakers from around world, including innovators,
philanthropists, venture capitalists, and other industry leaders. The launch Global Cooling Prize was also
announced on the sidelines of this summit. It will be international competition to incentivise development
of residential cooling technology that will have five times less climate impact in comparison to standard
Room Air Conditioning (RAC) units sold currently.

© 2018 GKToday | All Rights Reserved | https://www.gktoday.in 42


https://t.me/APPSC
https://t.me/UpscMaterials https://t.me/FreeUpscMaterials https://t.me/MaterialForExams
Current Affairs [PDF] -November 1-15, 2018

India-Indonesia naval exercise ‘Samudra Shakti’ begins in Indonesia


The inaugural edition India, Indonesia Bilateral Naval Exercise ‘Samudra Shakti’ was held at the port of
Surabaya, Indonesia. The aim of the exercise is to strengthen bilateral relations, expand maritime co-
operation, enhance interoperability and exchange best practices.
Samudra Shakti
The exercise started with Harbour Phase which encompassed planning and briefing on various activities,
professional interactions, cross deck visits, sports fixtures and social interactions. It was progressed by
Sea Phase which included operations such as joint manoeuvres, helicopter operations, surface warfare
exercise, ASW exercise and anti-piracy exercises. Indian Navy was represented by INS Rana of Eastern
Fleet based at Visakhapatnam under Eastern Naval Command, which is deployed in South East Asia.
ISPRL signs MoU with ADNOC to explore storage of crude oil at Padur underground facility in Karnataka
Indian Strategic Petroleum Reserves Ltd. (ISPRL) has signed Memorandum of Understanding (MoU) with
Abu Dhabi National Oil Company (ADNOC) to explore storage of crude oil at Padur underground facility in
Karnataka. This follows arrival of final shipment of initial delivery of ADNOC crude to be stored in another
ISPRL underground facility in Mangaluru, which will store 5.86 million barrels.
ISPRL is Special Purpose Vehicle (SPV), which is a wholly owned subsidiary of Oil Industry Development Board
(OIDB) under Ministry of Petroleum & Natural Gas.
Manjul Tripathi | tripathimanjul18@gmail.com |

Key Facts
The MoU mandates ADNOC to store crude oil at Padur Strategic Petroleum Reserve which has storage
capacity 2.5 million-tonne (~17 million barrels) capacity for emergency needs. ADNOC will store crude in
two compartments at Padur. ADNOC is only foreign oil and gas company, so far to invest, by way of crude
oil in India’s strategic petroleum reserves program. This will help to ensure energy security to India and
enable ADNOC to efficiently and competitively meet Indian market demand.
Background
Under the Phase I of Strategic Petroleum Reserves Program, SPRL has already built 5.33 million tons of
underground storage capacity at three locations, Visakhapatnam (1.33 million tons), Mangalore (1.5 million
tons) and Padur (2.5 million tons), that can meet around 10 days of the country’s oil needs. In June 2018,
Government announced phase II of program for creation of two new reserves, a 4-million-tons storage
facility at Chandikhol in Odisha, and an additional 2.5-million-tons facility at Padur. The combined
strategic reserve capacity of existing and newly announced facilities will be 15.33 million tons and will
provide around 22 days of emergency coverage for India’s crude oil requirements.
PM inaugurates country’s first multi-modal terminal constructed on river Ganga in Varanasi
Prime Minister Narendra Modi inaugurated India’s first Inland Multi-Modal Terminal Port on river Ganga
at Ramnagar in Varanasi and received first container cargo belonging to PepsiCo. It is the first of four
Multi-Modal Terminals being constructed on National Waterways-I (NW1) on River Ganga as part of
World Bank-aided Jal Marg Vikas project of Inland Waterways Authority of India (IWAI). The other three
terminals are under construction at Sahibganj, Haldia and Gazipur. It enable commercial navigation of
vessels with capacity of 1500-2,000 DWT on river Ganga.
Background
The Jal Marg Vikas Project (JMVP) was approved in January 2017 for capacity augmentation of navigation
on NW-1. It was approved at a cost of Rs 5369.18 crore with technical assistance and investment support of
World Bank. Following this, IWAI in February 2018 had signed project agreement with World Bank for Jal
Marg Vikas Project. World Bank provided $375 million loan for the project.
Jal Marg Vikas Project (JMVP)
The project aims to enable commercial navigation of vessels with capacity of 1500 – 2000 tons on NW1. It
includes development of fairway, multi-modal terminals at Varanasi, Haldia and Sahibganj, modern River
Information System (RIS), strong river navigation system, Digital Global Positioning System (DGPS), night
navigation facilities, construction of navigational lock at Farakka. It will result in environment-friendly,
fuel-efficient and cost-effective alternative mode of transportation, especially for bulk, hazardous goods
and over-dimensional cargo. Its development and operations will provide direct employment generation
to 46000 people and indirect employment of 84000 in states of Bihar, Uttar Pradesh, Jharkhand, West
© 2018 GKToday | All Rights Reserved | https://www.gktoday.in 43
https://t.me/APPSC
https://t.me/UpscMaterials https://t.me/FreeUpscMaterials https://t.me/MaterialForExams
Current Affairs [PDF] -November 1-15, 2018

Bengal.
November 14, 2018
UP cabinet approves renaming of Faizabad as Ayodhya, Allahabad as Prayagraj
Uttar Pradesh Government has approved renaming of Faizabad and Allahabad divisions as Ayodhya and
Prayagraj respectively. The decision was taken at meeting of state cabinet presided over by Chief Minister
Yogi Adityanath in state capital Lucknow. Now, the proposal will now go to Central before the city is
officially renamed. Prayagraj division will comprise Prayagraj, Kaushambi, Fatehpur and Pratapgarh
districts. Ayodhya division will include Ayodhya, Ambedkarnagar, Sultanpur, Amethi and Barabanki
districts, Khanna said.
Background
Government has justified the name changes across Uttar Pradesh, saying they are merely restoring older
names and correcting historical distortions. Allahabad’s ancient name was ‘Prayag’, but was changed after
16th-century Mughal emperor Akbar built fort near ‘Sangam’, the holy confluence of three rivers viz.
Ganga, Yamuna and Saraswati. Akbar named the fort and its neighbourhood ‘Ilahabad’. Later, Akbar’s
grandson Shah Jahan renamed the entire city as ‘Allahabad’. But the area near the ‘Sangam’, the site of the
Kumbh Mela, continues to be known as ‘Prayag’. The reason cited for renaming Faizabad is that Ayodhya
was the capital of ‘Ikshvaku dynasty’, to which the Hindu god Ram belonged, and that it is popular known
as his birthplace. Ayodhya is has been capital
Manjulof many
Tripathi estates and
| tripathimanjul18@gmail.com | dynasties in different time periods.
6th Indian Social Work Congress in New Delhi
The 6th Indian Social Work Congress was held in New Delhi. It was joinly organized by National
Association of Professional Social Workers in India (NAPSWI) and Department of Social Work (Delhi
School of Social Work), University of Delhi. Dr. Martha Farrell posthumously honoured with Lifetime
Achievement Award during this edition of congress for her lifelong work towards gender equality,
women’s empowerment and prevention of sexual harassment at workplace.
Key Facts
The theme of 6th ISWC 2018 was “Human Development and Social Inclusion: Imperatives for Social Work
Education and Practice”. This theme takes into consideration contemporary concerns of local to
international community within present context and in consonance with the social work value
framework.
6th ISWC2016 aimed to
Provide platform to social work professionals to discuss and deliberate on issues related to human
development.
Engage with issues related to Social Justice and Inclusion, within context of social work education,
research and practice.
Address diverse issues and challenges pertaining to human development and social justice vis-à-vis
social work profession in rapidly transforming socio-political and economic landscape.
Exchange experiences of social work educators, researchers and development practitioners to
augment networks and coalitions of social workers for invigorating social work profession.
38th India International Trade Fair (IITF) held at Pragati Maidan in New Delhi
The 38th India International Trade Fair (IITF) was held at Pragati Maidan in New Delhi. The 14-day annual
event was inaugurated by inaugurated by Union Minister of State for Culture, Mahesh Sharma. It is
organised by India Trade Promotion Organisation (ITPO).
India Trade Promotion Organisation (ITPO)
It is the nodal agency of the Government of India under aegis of Ministry of Commerce and Industry for promoting
country’s external trade. It is Mini-Ratna Category-1 Central Public Sector Enterprise (CPSE) with 100% shareholding
of Government. It is headquartered at Pragati Maidan.

38th IITF
The theme of this edition of fair was “Rural Enterprises in India”. Afghanistan is partner country, while
Nepal is focus country for this edition of fair. Jharkhand is participating as partner state in the event.
The fair is aimed providing platform for all kind of businesses to showcase their excellence & wide

© 2018 GKToday | All Rights Reserved | https://www.gktoday.in 44


https://t.me/APPSC
https://t.me/UpscMaterials https://t.me/FreeUpscMaterials https://t.me/MaterialForExams
Current Affairs [PDF] -November 1-15, 2018

spectrum of services to trade, industry & common men.


Around 800 participants from different States, government departments and domestic and
international companies are taking part in the fair to showcase their product.
In this fair, exhibition of various types of products such as automobile, jute products, garments,
household utensils, processed foods, beauty products, electronic products etc. are being organized.
India International Cherry Blossom Festival in Shillong from Nov 14 to 17
India International Cherry Blossom festival is scheduled to be held from November 14 to 17 in Shillong,
Meghalaya. This festival is the world’s only autumn cherry blossom festival. The festival is organised by
Government of Meghalaya’s Forest & Environment Department as well as by the Institute of Bioresources
& Sustainable Development (IBSD) in collaboration with Indian Council for Cultural Relations (ICCR).
Aim
The aim of the festival is to celebrate the unique autumn flowering of Himalayan Cherry Blossoms. The
Cherry Blossom tree flowers only for a short period in a year. The festival will showcase full pink and
white cherry blossoms found all along the roadside of the famous Ward Lake. Alongside, the festival will
also showcase the region’s food, wine and crafts in addition with several cultural events.
Strategic Importance – Japan’s Participation
The main highlight of the festival will be the exposition of Japanese cultural events and cuisine. Embassy
of Japan is a strategic partner for the festival. There
Manjul Tripathi will be several
| tripathimanjul18@gmail.com | cultural events showcasing Japanese
cultural and cuisine. The festival in a way will act as a channel to deepen and strengthen the connections
with the Government of Japan.
The tradition of planting cherry blossoms, also called Sakura, was first begun by Japan. In fact, Japan has
been celebrating the Sakura Festival from the 3rd century and it continues till date.
Economic Importance
The cherry blossom festivals are being celebrated by around 28 countries across the world including India.
The other countries that celebrate this festival include Holland, United States of America, and South
Korea. This festival generates huge employment opportunities and generates a lot of revenue as it attracts
lakhs of visitors from across the world. Last year, the festival boosted the local economy of Meghalaya by
300%. The festival is also likely to promote sustainable tourism in the northeast.
Meghalaya-Biodiversity Hub
Meghalaya is the home to 5,538 species of fauna and about 3,128 species of flowering plants. It accounts for
18% of India’s total floral wealth. Also, Meghalaya is the home to the country’s only two species of true
apes.
World Diabetes Day: 14 November
World Diabetes Day was observed every year across the world on 14 November to raise awareness about
diabetes. Observance of day aims to pursue multilateral efforts to promote and improve human health and
provide access to health-care treatment and education.
The theme for World Diabetes Day 2018 and 2019 is “The Family and Diabetes”. The two-year theme
timeframe has been chosen to aise awareness of the impact that diabetes has on the family and support
network of those affected and promote role of family in management, care, prevention and education of
diabetes.
Background
The World Diabetes Day was instituted by United Nations General Assembly (UNGA) in 2007 by passing
resolution A/RES/61/225. The resolution had encouraged Member States to develop national policies for
prevention, treatment and care of diabetes.
Diabetes
Diabetes is a chronic, progressive non-communicable disease (NCD) characterized by elevated levels of
blood sugar (blood glucose). It occurs when (i) the pancreas does not produce enough of the insulin
hormone, which regulates blood sugar (ii) the body cannot effectively use the insulin it produces. Type 1
diabetes occurs when the body doesn’t make enough insulin, and type 2 diabetes occurs when the body
doesn’t make or use insulin very well, causing glucose to remain in the blood, which can lead to serious
problems. The main symptom of diabetes is excess fatigue, frequent urination, dysentery, excessive thirst,
etc. Diabetes can also be genetic, but its main cause is obesity, lack of adequate nutrition, etc. Insulin
© 2018 GKToday | All Rights Reserved | https://www.gktoday.in 45
https://t.me/APPSC
https://t.me/UpscMaterials https://t.me/FreeUpscMaterials https://t.me/MaterialForExams
Current Affairs [PDF] -November 1-15, 2018

injection is given to the patient suffering from type 1 diabetes. Prevention of type 2 diabetes is quite
possible.
Infosys Prize 2018: 6 professors awarded for science and research
The Infosys Science Foundation (ISF) occasion of its 10th anniversary announced six winners of the
Infosys Prize 2018 in the six different categories of the award. They were selected from 244 nominations
by six-member jury of renowned scientists and professors.
2018 Recipients are
Engineering and Computer Science: Navakanta Bhat. He is Professor at Indian Institute of Science (IISc),
Bangalore. He was awarded for his work on design of novel biosensors based on his research in
biochemistry and gaseous sensors that push the performance limits of existing metal-oxide sensors.
Humanities: Kavita Singh. She is Dean, School of Arts & Aesthetics, Jawaharlal Nehru University. She was
awarded for her illuminating study of Mughal, Rajput and Deccan art.
Life Sciences: Roop Mallik. He is associate Professor, Department of Biological Sciences, Tata Institute of
Fundamental Research (TIFR). He was awarded for his work on molecular motor proteins, which are
crucial for the functioning of living cells.
Mathematical Sciences: Nalini Anantharaman. She is Professor and Chairman of Mathematics, Institute
for Advanced Study, University of Strasbourg, France. She was awarded the prize in Mathematical
Sciences for her work related to ‘Quantum Manjul
Chaos.’
Tripathi | tripathimanjul18@gmail.com |

Physical Sciences: SK Satheesh. He is Professor, Centre for Atmospheric & Oceanic Sciences, IISc
Bangalore. He was awarded for his pioneering scientific work in the field of climate change.
Social Sciences: Sendhil Mullainathan. He is professor of Computation and Behavioral Science, the
University of Chicago in the US. He was awarded for his path-breaking work in behavioural economics.
Infosys Prize
Infosys Prize is an annual award bestowed upon researchers, scientists, social scientists and engineers in
India by the Infosys Science Foundation (ISF). It is given to six categories Life Sciences, Mathematical
Sciences, Engineering and Computer Science, Social Sciences, Physical Sciences and Humanities. The
award intends to celebrate success in research and stand as a marker of excellence in these fields. Each
category awardee receives 22-carat gold medallion, a citation certificate and prize money of US $100,000.
It is one of the highest monetary awards in India that recognizes the scientific research.
Government to release Rs 75 commemorative coin to mark 75th anniversary of Tricolour hoisting by Bose
Union Ministry of Finance has announced to release Rs 75 commemorative coin on occasion of 75th
anniversary of hoisting of Tricolour for first time by Netaji Subhash Chandra Bose at Cellular Jail, Port
Blair on December 30, 1943. Earlier on October 21, 2018, Prime Minister Narendra Modi had hoisted
National Flag at Red Fort and unveiled plaque to commemorate 75th Anniversary of formation of Azad
Hind Government, formed by Bose.
Features of coin
The 35-gram coin will be composed of 50% silver, 40% copper, and 5% each of nickel and zinc. It will bear
the portrait of Netaji Subhash Chandra Bose saluting flag on background of cellular jail. It will have
numeral 75th along with inscription “anniversary” will be depicted below portrait. It will also have
inscription in both Devanagari script and English saying ‘First Flag Hoisting Day’.
First Flag Hoisting by Netaji Subhash Chandra Bose
Subhash Chandra Bose on December 30, 1943 had hoisted Tricolor for first time on free Indian soil at
Cellular Jail of Port Blair during his visit to Andaman and declaring the island as the free territory from
the British rule. It was much before India attained Independence in 1947. On same occasion, he had
announced freedom of Andaman & Nicobar Islands (making it first Indian Territory) from the British rule
and renamed them as “Shaheed-dweep” (Martyr Island) and “Swaraj-dweep” (Self-rule Island). The island
at that time was liberated from British rule by Japan which captured it during World War – II. Major
General Arcot Doraiswamy Loganadan of Indian National Army (INA) was made the Governor of the
Andaman and Nicobar Islands.
Indra 2018: India, Russia joint military exercise held in Jhansi, UP
The joint military exercise between Indian and Russia EXERCISE INDRA 2018 was conducted at Babina

© 2018 GKToday | All Rights Reserved | https://www.gktoday.in 46


https://t.me/APPSC
https://t.me/UpscMaterials https://t.me/FreeUpscMaterials https://t.me/MaterialForExams
Current Affairs [PDF] -November 1-15, 2018

Field Firing Ranges, Babina Military Station Jhansi, Uttar Pradesh. It was tenth exercise in the series of
Exercise INDRA aimed at increasing interoperability between Indian and Russian Armies.
EXERCISE INDRA 2018
The aim of the exercise was to practice joint planning and conduct to enhance interoperability of between
two armies in peace keeping/ enforcement environment under aegis of United Nations. For this exercise,
India was represented by Mechanised Infantry Battalion of Indian Army, while Russia was represented by
company sized contingents of 5th Army.
It focused upon training on enhancing team building, special tactical level operations such as Cordon and
Search, house intervention, handling and neutralisation of Improvised Explosive Devices and integrated
employment of force multipliers. The primary focus of tactical field exercise was to share best practices
amongst militaries and hone existing skills of troops.
Government launches LEAP and ARPIT programmes for higher education faculty
Union Ministry off Human Resource Development launched two new initiatives viz. Leadership for
Academicians Programme (LEAP) and Annual Refresher Programme In Teaching (ARPIT) for higher
education faculty. ARPIT is unique initiative of online professional development of 15 lakh higher
education faculty. LEAP is three weeks flagship leadership development training programme for second
level academic functionaries in higher education institutions,
Leadership for Academicians Programme (LEAP) Manjul Tripathi | tripathimanjul18@gmail.com |

It is three weeks (2 weeks domestic and 1 week foreign training) flagship leadership development training
programme for second level academic functionaries in public funded higher education institutions. Its
main objective is to prepare second tier academic heads who are potentially likely to assume leadership
roles in the future.
It will make Higher Education Institutions (HEIs) perform their roles of developing better students who
will be future torch bearers. It will provide senior faculty required leadership and managerial skills
including skills of problem-solving, team building work, handling stress, communication, conflict
management, understanding complexity and challenges of governance in HEIs and general
administration.
It will be implemented through 15 National Institute Ranking Framework (NIRF) top ranked Indian
Institutions namely, IIT Bombay, IIT Kanpur, IIT Kharagpur, IIT Roorkee, IISER Kolkata, NIT Trichy, JNU;
IIT (BHU), Delhi University, TISS Mumbai, University of Hyderabad, NIEPA, Jamia Milia Islamia, BHU and
AMU.
The foreign universities identified for training collaboration are also within top 100 in world global
rankings. It includes University of Michigan, Harvard University, Oxford University, University of
Cambridge, Purdue University, University of Chicago, NTU Singapore, Monash University, University of
Pennsylvania and University of Virginia.
Annual Refresher Programme in Teaching (ARPIT)
It is a major and unique initiative of online professional development of 15 lakh higher education faculty
using Massive Open Online Courses (MOOCs) platform SWAYAM. Under it 75 discipline-specific
institutions are identified and notified as National Resource Centres (NRCs) in phase I.
These institutions will be tasked to prepare online training material with focus on latest developments in
discipline, methodologies for transacting revised curriculum and pedagogical improvements. The courses
will cover diverse range of topics.
Each course will be 40 hour module with 20 hours of video content and 20 hours of non-video content. It
will also have built-in assessment exercises and activities as part of the academic progression. There will
be a terminal assessment either online or written examination at end of the course.
All faculties after successfully completing the online refresher course will be certified. All in-service
teachers, irrespective of their subject and seniority will register and complete these refresher courses
which will help them in career advancement.
ARPIT will be every year exercise and NRCs will continuously develop new refresher module in their
earmarked discipline each year. The training materials will be uploaded and made available through
SWAYAM. In inaugural year 2018 itself, NRCs are expected to cater to around 13 lakh faculty of total 15
© 2018 GKToday | All Rights Reserved | https://www.gktoday.in 47
https://t.me/APPSC
https://t.me/UpscMaterials https://t.me/FreeUpscMaterials https://t.me/MaterialForExams
Current Affairs [PDF] -November 1-15, 2018

lakh faculty in higher education.


November 15, 2018
Nexxt Credit Card: IndusInd Bank launches first interactive Credit Card in India with buttons
IndusInd Bank has launched first interactive Credit Card in India with buttons called IndusInd Bank Nexxt
Credit Card. It will give customer multiple options on how to make a payment using his or her Credit
Card. This Card has been created in partnership with Pittsburgh USA headquartered Dynamics Inc., which
designs and manufactures intelligent, battery powered payment cards
Key Facts
This interactive Credit Card provides customers with flexibility of three payment options at Point of Sale
(POS) terminal – Credit, Converting Transactions into EMIs with 4 tenure options (6, 12, 18 & 24 months)
or using accumulated Reward Points, by simply pushing a button on the card.
It incorporates technology that indicates customer’s desired payment choice using LED lights associated
with three options. Using it customer does not need to fill any paperwork, or call their bank or log in to
any banking channel to convert their POS transactions into EMIs or to redeem their Rewards Points.
Athlete Hima Das appointed first Youth Ambassador of UNICEF India
India’s ace sprinter and Asian Games gold-medallist sprinter Hima Das was appointed as first ever youth
ambassador of United Nations Children’s Fund (UNICEF) India. UNICEF India works with central
government to ensure that each child bornManjul
in Tripathi
country gets best| start in life, thrives and develops to his or
| tripathimanjul18@gmail.com

her full potential.


Hima Das
She was born 9 January 2000 at Kandhulimari village near Dhing Town of Nagaon district, Assam. Her
parents are rice farmers. She was the first Indian athlete to win gold medal in track event at IAAF World
U20 Championships held in Tampere, Finland. In 2018 Asian Games (Jakarta), she had won silver medal in
400 m final to win a silver medal. She is nicknamed ‘Dhing Express’ after her village in Assam. In
September 2018, she was appointed as Sports Ambassador of Assam to inspire budding players of
northeastern state to aim big and young generation to take up sports seriously. She is also working with
Indian Oil Corporation in Guwahati as HR officer.
Medals won in 2018 Asian Games
Gold medal: Women’s 4 × 400 metres relay along with M. R. Poovamma, Sarita Gayakwad and V. K.
Vismaya.
Silver medal: Mixed4 × 400 metres relay along with Muhammed Anas, M. R. Poovamma and Arokia
Rajiv.
Silver medal: Women’s 400 m individual race (time: 50.79). She had set national record by clocking
51.00 to qualify for the 400 metre final.
Winter Session of Parliament to be held from Dec 11, 2018 to Jan 08, 2019
The Cabinet Committee on Parliamentary Affairs (CCPA) has recommended convening of winter session
of Parliament from December 11, 2018 to January 8, 2019. The CCPA was headed by Union Home Minister
Rajnath Singh and met in New Delhi and deliberated on the dates of the session. The winter session of
Parliament usually starts every year after Monsoon Session in November. However, from past two years
in a row, it is beginning in December. The session has been delayed this year due to assembly elections in
five states.
Cabinet Committee on Parliamentary Affairs (CCPA)
CCPA is one of the six cabinet committees in function (as of 2016). It looks after progress after of
government business in the Parliament. It is chaired by Union Home Minister. Others five cabinet
committees are Cabinet Committee on Political Affairs, Cabinet Committee on Economic (CCEA), Cabinet
Committee on Security (CCS), Cabinet Committee on Accommodation (CCA) and Appointments
Committee of Cabinet (ACC). These five cabinet committees are chaired by Prime Minister.
ISRO successfully launches communication satellite GSAT-29
Communication satellite GSAT-29 was launched successfully by the second developmental flight of
Geosynchronous Satellite Launch Vehicle Mark III (GSLV MKIII-D2) from the Satish Dhawan Space Centre
(SDSC) SHAR, Sriharikota.

© 2018 GKToday | All Rights Reserved | https://www.gktoday.in 48


https://t.me/APPSC
https://t.me/UpscMaterials https://t.me/FreeUpscMaterials https://t.me/MaterialForExams
Current Affairs [PDF] -November 1-15, 2018

GSAT-29
Communication satellite GSTA-29 weighing 3,423 kg is a multiband and multi-beam satellite. The mission
life will be 10 years.
The satellite’s Ku-band and Ka-band payloads will cater to the communication needs of people residing in
remote parts of the country especially those from Jammu & Kashmir and North-Eastern regions.
Similarly, the satellite’s Q/V-Band communication payload will help in demonstrating the future high
throughput satellite system technologies. Additionally, the onboard Geo High Resolution Camera will help
in carrying out high resolution imaging. The onboard Optical Communication Payload will showcase a
very high rate data transmission through optical communication link. This unique laser based optical
communication is being tested for the first time by the ISRO.
GSLV MKIII
GSLV MKIII is ISRO’s fifth generation three stage heavy lift launch vehicle. The launch vehicle is capable of
placing satellites weighing 4000 kg in GTO. The first stage constitutes two massive boosters with solid
propellants. The core with liquid propellant and the cryogenic engine constitutes the second and third
stages respectively.
The weight of GSLV Mark III is 641 tons roughly equals the weight of five fully loaded passenger
aircrafts. GSLV MarkIII, which took 15 years to make, is the heaviest among India’s operational
launch vehicles. Also, with only 43 meters tall,
Manjul Tripathi it is also the| shortest launch vehicles.
| tripathimanjul18@gmail.com

GSLV MKIII will be used to launch Chandrayaan-2 and Gaganyaan missions.


Significance
With the success of this flight, the Indian Space programme has achieved a significant milestone as the
heaviest launcher (GSLV MKIII) lifted off the heaviest satellite (GSAT-29). The success of this mission is
also an important milestone for the Indian space programme as it showcases the self-reliance of ISRO in
launching heavier satellites. Also, the launch also signifies the completion of the experimental phase of
GSLV MarkIII. The first successful mission of GSLV MarkIII was in 2014. In 2017, GSLV Mark III-D1 placed
3150 kg GSAT-19, a high throughput communication satellite into Geostationary Transfer orbit (GTO).
The launch of GSAT-29 communication satellite will pave for bridging the digital divide in the country.
The launch of the satellite is also expected to aid Digital India Programme. The satellite is also intended to
serve as a test bed for several new and critical next generation payload technologies. GSAT-29 is the 33rd
communication satellite of India.
The successful flight of GSAT-29 will constitute a series of experimental flights that will help in
developing the heaviest rocket for the manned space missions of the ISRO. ISRO’s manned mission
Gaganyaan is expected to lift of three Indian astronauts into space for a seven days trip to the Low-Earth
Orbit. The mission is slated for 2022.
Government launches Yuva Sahakar-Cooperative Enterprise Support and Innovation Scheme
Union Minister of Agriculture and Farmers’ Welfare Radha Mohan Singh launched Yuva Sahakar-
Cooperative Enterprise Support and Innovation Scheme of National Cooperative Development
Corporation (NCDC). It isyouth-friendly scheme aimed at attracting youth to cooperative business
ventures.
Yuva Sahakar-Cooperative Enterprise Support and Innovation Scheme
The scheme is linked to Rs 1000 crore ‘Cooperative Start-up and Innovation Fund (CSIF)’ created by the
NCDC. It aims to encourage cooperatives to venture into new and innovative areas. It envisages 2% less
than applicable rate of interest on term loan for the project cost up to Rs 3 crore including 2 years
moratorium on payment of principal. All types of cooperatives in operation for at least one year are
eligible to avail of the scheme.
Under it more incentives will be provided for cooperatives of North Eastern region, Aspirational Districts
and cooperatives with women or SC or ST members. The funding for the project under this scheme will be
up to 80% of the project cost for these special categories against 70% for others
NIPUN: Delhi Police launces e-learning portal for training its personnel
Delhi Police has launched an e-learning portal ‘Nipun’ to impart training and provide up to date
information to police officers. The initiative is aimed at imparting in-service online training and

© 2018 GKToday | All Rights Reserved | https://www.gktoday.in 49


https://t.me/APPSC
https://t.me/UpscMaterials https://t.me/FreeUpscMaterials https://t.me/MaterialForExams
Current Affairs [PDF] -November 1-15, 2018

information to police personnel through specialised courses available online which have been designed by
experts in various fields.
NIPUN
The online courses on this portal have been designed in collaboration with University Grant Commission
(UGC), FICCI, NHRC, NCPCR and Janaki Devi Memorial College, Delhi University under project CLAP “The
Collaborative Learning and Partnership”. Delhi Legal Service Authority has also consented to be partner in
developing specific courses for the Delhi Police.
It will have online courses and resources on law, standing orders, investigation checklists, forms for case
files, latest High Court and Supreme Court rulings. It is envisioned that Investigating Officers (IOS) will
be able to take benefit of this information during course of their duty or while handling case.
Through this portal Delhi Police officers can now access information by just logging on the portal. They
can take course from anywhere at any time. It will make easier for the police officer to upgrade their skills
and re-enforce their learning while balancing training time along with their daily routine work.
India and UK sign MoU for collaboration in cancer research
India and United Kingdom have signed Memorandum of Understanding (MoU) for collaboration in cancer
research. It is follow-up to joint statement issued by Prime Ministers of India and UK during visit of PM
Narendra Modi to the UK in April 2018.
Key Facts Manjul Tripathi | tripathimanjul18@gmail.com |

The MoU was signed between Department of Biotechnology (DBT) and Cancer Research UK (CRUK) for
India-UK Cancer Research Initiative for collaboration in cancer research for the next five years. Under it,
both CRUK and DBT will invest 5 million pounds each in this five-year pilot project and seek further
investment from other potential funding partners.
India-UK Cancer Research Initiative will identify core set of research challenges that address issues of
affordability, prevention and care of cancer patients by bringing together leading Indian and UK experts
in clinical research, demographic research, new technologies and physical sciences.
The initiative will provide funding to develop new research alliances and undertake impactful research to
enable significant progress against cancer outcomes. It will also provide catalysing platform for scientists
and researchers in India and UK to co-create solutions for affordable cancer care that improve cancer
outcomes around the globe.
Regional meeting of World Customs Organisation being held in Jaipur
Four-day regional meeting of World Customs Organisation (WCO) was held in Jaipur, capital city of
Rajasthan. It was attended by representatives of 33 member countries of Asia. The meeting was jointly
chaired by WCO Deputy Secretary General Ricardo Travino and Central Board of Indirect Taxes and
Customs Chairman S Ramesh.
Key Facts
The meeting deliberated upon various issues including the steps required for the capacity building and
reforms in customs. It also discussed and deliberated upon amended Kyoto protocol, digital customs, e-
commerce among other issues. Summary of various efforts made by National Trade Facilitation
Committee (NTFC) for trade facilitation was also tabled in the meeting.
World Customs Organisation (WCO)
WCO is independent intergovernmental body whose mission is to enhance effectiveness and efficiency of
Customs administrations. It was established in 1952 as Customs Co-operation Council (CCC). It is the only
international organization with competence in Customs matters and is considered as voice of
international Customs community.
It represents 182 Customs administrations across the globe that collectively process approximately 98% of
world trade. It is headquartered in Brussels, Belgium. It plays leading role in discussion, development,
promotion and implementation of modern customs systems and procedures.
Objectives: It aims to enhance efficiency and effectiveness of member customs administrations and assist
them to contribute successfully to national development goals, particularly revenue collection, national
security, trade facilitation, community protection, and collection of trade statistics.
Organistaional Structure: WCO’s governing body – the Council relies on competence and skills of WCO

© 2018 GKToday | All Rights Reserved | https://www.gktoday.in 50


https://t.me/APPSC
https://t.me/UpscMaterials https://t.me/FreeUpscMaterials https://t.me/MaterialForExams
Current Affairs [PDF] -November 1-15, 2018

Secretariat and a range of technical and advisory committees to accomplish its mission. The Secretariat
comprises over 100 international officials, technical experts and support staff of some nationalities. WCO
has divided its membership into six regions whereby each region is represented by elected vice-
chairperson to WCO Council.
Functions: It offers range of Conventions and other international instruments, as well as technical
assistance and training services to its members. It also actively supports its members in their endeavours
to modernize and build capacity within their national Customs administrations.
It also plays vital role in stimulating growth of legitimate international trade and combats fraudulent
activities. It also promotes emergence of honest, transparent and predictable Customs environment, thus
directly contributing to economic and social well-being of its members.
Multiple Choice Questions
1. Which Nobel Prize winner will be honoured by Harvard University for her work promoting girls’
education?
[A] Nadia Murad
[B] Malala Yousafzai
[C] Kailash Satyarthi
[D] Ellen Johnson Sirleaf Manjul Tripathi | tripathimanjul18@gmail.com |

Correct Answer: B [Malala Yousafzai ]


Notes:
Nobel Prize Winner Malala Yousafzai will be honoured by Harvard University for her work promoting
girls’ education. The award will be conferred with the 2018 Gleitsman Award at a ceremony on 6th
December. Currently, Yousafzai is a student at Oxford University in England. The Gleitsman Award
provides $125,000 for activism that has improved quality of life around the world. Yousafzai became the
youngest person to win the Nobel Peace Prize in 2014 when she was recognized for her global work
supporting schooling for all children. As a teen in Pakistan, she survived an assassination attempt by the
Taliban. She later founded the nonprofit Malala Fund to support her work.
2. Yeshwant Deo, who passed away recently, was the veteran personality of which field?
[A] Painting
[B] Sports
[C] Music
[D] Dance
Correct Answer: C [Music]
Notes:
Yeshwant Deo (91), the veteran Marathi musician, has passed away recently in Dadar, Maharashtra on
October 31, 2018. He had composed music for many films including Shabana Azmi-starrer “Saaz”. Born
on 1st November 1926, Deo was a revered figure in the Marathi music circle and industry where he
composed music for more than 40 Marathi films and plays. He worked in All India Radio (AIR) as a sitar
player. A recipient of the Lata Mangeshkar award, Yeshwant was also known for his composition in Bhav
Geet. He won several awards such as Maharashtra state award for music direction for the play Amrapali
(1974). He also published books on devotional music.
3. Who has been sworn-in as the new Chief Justice of the Bombay High Court?
[A] A. S. Bopanna
[B] R. Subhash Reddy
[C] Rajendra Menon
[D] Naresh Harishchandra Patil
Correct Answer: D [Naresh Harishchandra Patil]
Notes:
Justice Naresh Harishchandra Patil has been sworn-in as the new Chief Justice of the Bombay High
Court. Maharashtra Governor C Vidyasagar Rao administered the oath of office to Justice Patil at a brief
ceremony held at Raj Bhavan. The Bombay High Court is one of the oldest High Courts of India and its
principle seat is at Mumbai. Its jurisdiction covers the states of Maharashtra and Goa, and the Union
© 2018 GKToday | All Rights Reserved | https://www.gktoday.in 51
https://t.me/APPSC
https://t.me/UpscMaterials https://t.me/FreeUpscMaterials https://t.me/MaterialForExams
Current Affairs [PDF] -November 1-15, 2018

Territories of Daman and Diu and Dadra and Nagar Haveli.


4. The world’s largest airport ‘Under One Roof’ has officially opened in which city?
[A] Kochi
[B] Istanbul
[C] Beijing
[D] Tokyo
Correct Answer: B [Istanbul ]
Notes:
The world’s largest airport terminal “under one roof” has officially been opened in Istanbul on October
29, which marks the Republic Day of Turkey. It is dubbed as phase-1A of the project – Istanbul New
Airport – will have a capacity to handle 90 million passenger per year. The first phase has costed 7.5
billion Euro. The airport has been envisioned to have a total capacity of up to 200 million passengers,
once all four phases are completed in the next 10 years. The new terminal once fully functional by
December-end will allow passengers to use its five pier blocks as well as 77 boarding gates and 143 aero-
bridges. The new airport is located near the Black Sea coast on the Europe side of Istanbul. It features
two runways, while 114 narrow body aircraft can dock with the terminal building at the same time. This
new airport is also slated to become Turkish Airlines’ new hub.
5. India’s first Justice City will come up Manjul
in which of the following
Tripathi | tripathimanjul18@gmail.com | cities?
[A] Amaravati
[B] Allahabad
[C] Patna
[D] Chennai
Correct Answer: A [Amaravati ]
Notes:
The Andhra Pradesh government is building a world-class and India’s first ‘Justice City’ within its new
capital Amaravati with the vision to provide a state-of-the-art ecosystem built on latest technology for
supporting the judicial system in the state. The Justice City will comprise integrated judicial hub spread
across 3,309 acres with an estimated population of 2.5 lacs. It is expected to generate employment for
more than 1.3 lac people. The Andhra Pradesh Capital Region Development Authority (APCRDA). is the
nodal agency which is developing Amaravati.
6. Louis Cha, who passed away recently, was the noted journalist and martial arts novelist of
which country?
[A] South Korea
[B] Taiwan
[C] Hong Kong
[D] Japan
Correct Answer: C [Hong Kong]
Notes:
Louis Cha Jing-yong (94), a Hong Kong journalist and best-selling Chinese martial arts novelist, has
passed away in Happy Valley, Hong Kong on October 30, 2018. Cha’s first novel, “The Book and the
Sword,” was published in 1955 and became an instant hit. He went on to write 14 martial arts novels,
often under the pen name Jin Yong. His works have the unusual ability to transcend geographical and
ideological barriers separating Chinese communities of the world, achieving a greater success than any
other contemporary writer. His works have been translated into many languages including English,
French, Korean, Japanese, Vietnamese, Thai, Burmese, Malay and Indonesian. According to ‘The Oxford
Guide to Contemporary World Literature’, Jin’s novels are considered to be of very high quality and are
able to appeal to both highbrow and lowbrow tastes.
7. The scientists of which Indian organisation has recently developed less polluting firecrackers
named – SWAS, SAFAL & STAR?
[A] CSIR
[B] DRDO

© 2018 GKToday | All Rights Reserved | https://www.gktoday.in 52


https://t.me/APPSC
https://t.me/UpscMaterials https://t.me/FreeUpscMaterials https://t.me/MaterialForExams
Current Affairs [PDF] -November 1-15, 2018

[C] ISRO
[D] BARC
Correct Answer: A [CSIR ]
Notes:
The scientists at CSIR have developed Less Polluting ‘Green’ Firecrackers which are not only
environment friendly but 15-20 % cheaper than the conventional ones. These crackers have been named
as Safe Water Releaser (SWAS), Safe Minimal Aluminium (SAFAL) and Safe Thermite Cracker (STAR).
The commonly used pollution-causing chemicals – aluminium, barium, potassium nitrate and carbon –
have either been removed or sharply reduced in the green crackers to reduce emissions by 15-30% .
These eco-friendly firecrackers have the unique property of releasing water vapour and /or air as dust
suppressant and diluent for gaseous emissions and matching performance in sound with conventional
crackers. SWAS eliminates usage of (KNO3) Potassium nitrate and Sulphur, while STAR eliminates usage
of KNO3 and S. On the other side, SAFAL has minimal usage of aluminium (only in flash powder for
initiation) as compared to commercial crackers.The main point to be noted here is that these crackers
match the same sound intensity with that of commercial crackers in the range of 105-110 decibel. The
new crackers have been developed by scientists of two national labs of CSIR- Central Electrochemical
Research Institute (CECRI) based in Karaikudi, Tamil Nadu and National Environmental Engineering
Research Institute (NEERI) in Nagpur.
8. Which union ministry has organized Gandhian Thought and Swachhata seminar organized in
Manjul Tripathi | tripathimanjul18@gmail.com |
Wardha district of Maharashtra?
[A] Ministry of Women and Child Development
[B] Ministry of Rural Development
[C] Ministry of Health and Family Welfare
[D] Ministry of Drinking Water and Sanitation
Correct Answer: D [Ministry of Drinking Water and Sanitation]
Notes:
The Ministry of Drinking Water and Sanitation (MDWS) organized a seminar on Gandhian Thought and
Swachhata in Wardha district of Maharashtra on October 29, 2018. It was part of 150th birth year
celebrations of Father of the Nation. The seminar was centered on the Gandhian ideology on sanitation
and the implementation of the same through the Swachh Bharat Mission (SBM). It addressed on topics
such as appropriate technologies in rural sanitation, organic waste management, and preventive
sanitation. The main focus of the seminar was the Gandhi ji’s concept of Swachh evam Swawalambhi
Sankul. The SBM campaign is a massive people’s movement, which has shown significant progress with
the rural sanitation coverage having increased from 39%, at the launch of the SBM in 2014, to over 95%
today. With 8.7 crore toilets constructed, 5.15 lakh villages, 530 districts and 25 States and Union
Territories have been declared ODF. The campaign was officially launched on 2nd October 2014 at
Rajghat, New Delhi by Prime Minister Narendra Modi.
9. Which country will be the Partner Country at the 25th DST – CII Technology Summit 2019?
[A] Italy
[B] France
[C] Netherlands
[D] Japan
Correct Answer: C [Netherlands ]
Notes:
The Kingdom of Netherlands will be the Partner Country at the 25th DST – CII Technology Summit
2019. It was recently announced by Dr. Harsh Vardhan, Union Minister for Science & Technology, Earth
Sciences, Environment,Forest and Climate Change. The Minister highlighted the successful bilateral
research cooperation between India and Netherlands in sectors such as affordable medical devices,
urban water systems, big data and Internet of Things (IoT). The Summit will be organised by
Department of Science and Technology (DST) and the Confederation of Indian Industry (CII) in
collaboration with the Netherlands government.
10. Who is the chairperson of the Financial Stability and Development Council (FSDC)?
[A] Finance Minister

© 2018 GKToday | All Rights Reserved | https://www.gktoday.in 53


https://t.me/APPSC
https://t.me/UpscMaterials https://t.me/FreeUpscMaterials https://t.me/MaterialForExams
Current Affairs [PDF] -November 1-15, 2018

[B] RBI Governor


[C] Finance Secretary
[D] SEBI Chairman
Correct Answer: A [Finance Minister]
Notes:
Recently, the 19th meeting of the Financial Stability and Development Council (FSDC) held under the
Chairmanship of the Union Finance Minister Arun Jaitley reviewed the current global and domestic
economic situation and financial sector performance. Apart from liquidity in NBFCs (segmental liquidity
position and in mutual fund space), the FSDC discussed on various economic issues ranging from oil
prices, rupee, inflation to improving cyber security in the financial sector including “progress made
towards setting up of a Computer Emergency Response Team in the Financial Sector (CERT-Fin) under a
Statutory Framework”. The FSDC is an apex-level body to strengthen and institutionalise the
mechanism of maintaining financial stability, financial sector development, inter-regulatory
coordination along with monitoring macro-prudential regulation of economy.
11. Who has been selected for 2018 Ezhuthachan Puraskaram, the highest literacy prize of the
Kerala government?
[A] Uma Shankar Singh
[B] K Satchidanandan Manjul Tripathi | tripathimanjul18@gmail.com |

[C] M Mukundan
[D] Ram V Kumar
Correct Answer: C [M Mukundan]
Notes:
Noted Malayalam writer M Mukundan has been selected for 2018 Ezhuthachan Puraskaram, the highest
literacy prize of the Kerala government. The award, which carries a cash prize of Rs five lakh and
citation, is being given for his overall contributions to the language and literature. His popular works
inlcude ‘Mayyazhippuzhayude Theerangalil’, ‘Kesavante Vilapangal’, ‘Delhi Gadhakal’, ‘Appam Chudunna
Kunkiyamma’ and ‘Lesli Achante Kadangal’.
12. Who has been appointed as the new Director General (DG) of Indo-Tibetan Border Police
(ITBP)?
[A] RK Pachnanda
[B] S S Deswal
[C] Rajiv Rai Bhatnagar
[D] Sudeep Lakhtakia
Correct Answer: B [S S Deswal]
Notes:
S S Deswal, a 1984 batch IPS officer of Haryana cadre, has been appointed as the new Director General
(DG) of Indo-Tibetan Border Police (ITBP) for a period up to his superannuation on August 31, 2021. He
succeeded R K Pachnanda. Prior to this post, Deswal was the DG of the Sashastra Seema Bal (SSB). The
ITBP was raised on 24 October 1962 under the CRPF Act, in the wake of the Sino-Indian War of 1962. It is
one of the five Central Armed Police Forces of India. Apart from its primary task of guarding the 3,488-
km India-China border, where ITBP troops are deployed at freezing heights of up to 18,700 feet, the
paramilitary force is tasked with rendering a variety of duties in the internal security domain, including
undertaking anti-Naxal operations in Chhattisgarh.
13. Which Indian cricketer has officially been inducted into the ICC Cricket Hall of Fame?
[A] Sachin Tendulkar
[B] Saurav Ganguly
[C] Rahul Dravid
[D] VVS Laxman
Correct Answer: C [Rahul Dravid]
Notes:
On November 1, former Indian cricket captain Rahul Dravid has officially been inducted into the ICC
© 2018 GKToday | All Rights Reserved | https://www.gktoday.in 54
https://t.me/APPSC
https://t.me/UpscMaterials https://t.me/FreeUpscMaterials https://t.me/MaterialForExams
Current Affairs [PDF] -November 1-15, 2018

Cricket Hall of Fame for his stupendous achievements in international cricket. In a ceremony held ahead
of the fifth and final ODI between India and West Indies in Thiruvananthapuram, Dravid was presented
with the HoF cap by Sunil Gavaskar. With this, Dravid became only the fifth player from India to be
named in the elite list. The other four Indians are former skippers Bishan Singh Bedi, Sunil Gavaskar,
Kapil Dev and Anil Kumble. Dravid, also popularly known as ‘the Wall’ had been a long-standing rock of
Indian cricket for over 15 years. In all, he aggregated 13,288 runs in 164 Tests with 36 centuries and
10,889 runs in 344 ODIs with 12 centuries. He was also a brilliant slip fielder and finished his Test career
in 2012 with 210 catches, which is a world record. He is currently the head coach of India A and India
U-19 sides.
14. Which state government has recently launched “Soura Jalanidhi” scheme?
[A] Jharkhand
[B] Odisha
[C] Bihar
[D] Uttar Pradesh
Correct Answer: B [Odisha]
Notes:
Naveen Patnaik, the Chief Minister of Odisha, has recently launched ‘Soura Jalanidhi’ scheme which
aims at increasing use of solar energy for helping farmers in irrigation. ‘Soura Jalanidhi’ is a dug well
based solar pump irrigation system in convergence mode. Under the scheme, 5,000 solar pumps will be
Manjul Tripathi | tripathimanjul18@gmail.com |

provided to farmers at a subsidy of 90 per cent to irrigate 2,500 acres of land. The chief minister also
launched the web portal of this scheme on the occasion. The event was attended by 1,000 farmers from
30 districts through video conference. In the first phase, the facility will be available for farmers where
electicity is not avaialable for operating pump sets. The solar pumps will be given to the beneficiary
farmers at a subsidy of 90%. A total of Rs 27 crore will be spent for the programme. Farmers having valid
farmer identification, belonging to small and marginal categories having minimum 0.5 acre cultivable
land and a dug well will be covered under the programme.
15. What is the India’s rank in the World Bank’s Ease of Doing Business Index 2018?
[A] 77th
[B] 98th
[C] 87th
[D] 64th
Correct Answer: A [77th ]
Notes:
India has been ranked 77th in the World Bank’s Ease of Doing Business Index 2018 . With this, India has
jumped 23 places from 2017 in which it was ranked at 100th. In its annual ‘Doing Business’ 2019 report,
World Bank stated that India improved its rank on six out of the 10 parameters relating to starting and
doing business in a country.The report also recognises India as one of the top 10 improvers in this year’s
assessment, for the 2nd successive time. The Index ranks 190 countries across 10 indicators ranged
across the lifecycle of a business from ‘starting a business’ to ‘resolving insolvency’. These parameters
include ease of starting a business, construction permits, getting electricity, getting credit, paying taxes,
trade across borders, enforcing contracts and resolving insolvency. New Zealand tops the list, followed
by Singapore, Denmark, and Hong Kong.
16. Who has become the first Indian to win an Asian Snooker Tour event?
[A] Pankaj Advani
[B] Lucky Vatnani
[C] Varun Madan
[D] Manan Chandra
Correct Answer: A [Pankaj Advani]
Notes:
19-times world champion Pankaj Advani has won the second leg of Asian Snooker Tour title by
delivering a power-packed performance to outclass China’s Ju Reti 6-1 in the final at Jinan, China.With
this, Advani has become the first Indian to win an Asian Snooker Tour event. The IBSF World Billiards
and Snooker Championships is less than a fortnight away and the Indian star cueist is the defending

© 2018 GKToday | All Rights Reserved | https://www.gktoday.in 55


https://t.me/APPSC
https://t.me/UpscMaterials https://t.me/FreeUpscMaterials https://t.me/MaterialForExams
Current Affairs [PDF] -November 1-15, 2018

champion in both sports. The new 10-red format of the Tour was well-received by all players as some
fine performances were on display throughout the second leg. Pankaj Advani will now have to
immediately switch to Billiards as Myanmar hosts the prestigious world championships from November
12 to 27.
17. Which country has launched the world’s first Sovereign Blue Bond?
[A] United Kingdom
[B] Mauritius
[C] Japan
[D] Seychelles
Correct Answer: D [Seychelles ]
Notes:
The Republic of Seychelles has launched the world’s first sovereign blue bond – a financial instrument
designed to support sustainable marine and fisheries projects. The bond developed with the assistance
of the World Bank & was placed to 3 investors: Calvert Impact Capital, Nuveen and Prudential. The blue
bond has a US$5m guarantee from the World Bank, and is also supported by a US$5m concessional loan
from the Global Environment Facility (GEF) which will partially cover interest payments.The blue bond
demonstrates the potential for countries to harness capital markets for financing the sustainable use of
marine resources. It will include support for the expansion of marine protected areas, improved
governance of priority fisheries and the development of the Seychelles’ blue economy.
Manjul Tripathi | tripathimanjul18@gmail.com |

18. Which union territory falls under jurisdiction of Calcutta High Court?
[A] Puducherry
[B] Lakshadweep
[C] Andaman and Nicobar Islands
[D] West Bengal
Correct Answer: C [Andaman and Nicobar Islands]
Notes:
Justice Debasish Kar Gupta has recently been sworn-in as the new chief justice of the Calcutta High
Court. It is the oldest High Court in India and has jurisdiction over the state of West Bengal and the
Union Territory of the Andaman and Nicobar Islands. The seat of the High Court is Kolkata, capital of
West Bengal. The High Court building’s design is based on the Cloth Hall, Ypres, in Belgium.
19. Which is the only Indian film has been included in BBC’s 100 best foreign language films list of
the 21st century?
[A] Pather Panchali
[B] Aparajito
[C] The World of Apu
[D] The Big City
Correct Answer: A [Pather Panchali]
Notes:
Satyajit Ray’s epic masterpiece ‘Pather Panchali’ has been included in BBC’s 100 best foreign language
films list of the 21st century. Ray’s film, which was released in 1955 and the first in his ‘Apu’ trilogy, is the
only Indian film that find its spot in the BBC list and is placed at number 15. The list is topped by Akira
Kurosawa’s “Seven Samurai”. It was culled out from a poll where over 200 critics from 43 countries
picked their best foreign language film. Some of the notable films that featured in the BBC list are —
Kurosawa’s ‘Rashomon ‘, Wong Kar-wai’s ‘In the Mood for Love’, Andrei Tarkovsky’s ‘The Mirror’, Asghar
Farhadi’s ‘A Separation’, Guillermo del Toro’s ‘Pan’s Labyrinth’, Ingmar Bergman’s ‘The Seventh Seal’ and
Alfonso Cuaron’s ‘Y Tu Mama Tambien’, among others. According to BBC, 27 of the highest-rated films
were in French, followed by 12 in Mandarin, and 11 each in Italian and Japanese. The BBC said that only
four of the 100 films have been directed by women. However, it said that 45% of the critics who took part
in the poll were women.
20. Which Indian Sculptor designed the world’s ‘tallest statue’Statue of Unity’?
[A] Ram V. Sutar

© 2018 GKToday | All Rights Reserved | https://www.gktoday.in 56


https://t.me/APPSC
https://t.me/UpscMaterials https://t.me/FreeUpscMaterials https://t.me/MaterialForExams
Current Affairs [PDF] -November 1-15, 2018

[B] Vivan Sundaram


[C] Jitish Kallat
[D] N. N. Rimzon
Correct Answer: A [Ram V. Sutar]
Notes:
On October 31, Prime Minister Narendra Modi unveiled the ‘Statue of Unity’ in Narmada district of
Gujarat on the birth anniversary of Sardar Vallabhbhai Patel. Sardar Patel – who served as the first
Home Minister of independent India – is credited with uniting all 562 princely states in pre-independent
India to build the Republic of India. That is why it is called the ‘Statue Of Unity’. It is the world’s tallest
statue, with a height of 182m. It located around 3.5 km downstream from the Sardar Sarovar Dam, on
islet Sadhu Bet on the bed of the river Narmada. The statue was designed by 93-year-old acclaimed
sculptor Ram V. Sutar, who is Padma Bhushan awardee. It is built by Larsen & Toubro Ltd at a cost of Rs.
2,989 crore. The Statue of Unity is made up of 70,000 tonnes of cement, 18,500 tonnes of reinforcement
steel, 6,000 tonnes of structural steel and 1,700 metric tonnes of bronze, which was used for the outer
cladding of the structure. The viewing gallery at the height of 135 metres can accommodate up to 200
people at a time.
21. Which decommissioned aircraft carrier will be converted into India’s first-ever moored
maritime museum-cum-marine adventure centre?
[A] INS Chakra
Manjul Tripathi | tripathimanjul18@gmail.com |

[B] INS Vagsheer


[C] INS Vikrant
[D] INS Viraat
Correct Answer: D [INS Viraat]
Notes:
The Maharashtra Cabinet has recently approved the conversion of decommissioned aircraft carrier INS
Viraat into a floating museum. It will be India’s first-ever moored maritime museum-cum-marine
adventure centre. As per primary estimates, the project worth Rs 852 crore will be implemented through
Public-Private-Partnership (PPP). It will be grouted (grounded, sealed to the seabed with concrete and
moored) seven nautical miles off the Malvan coast at Nivati rocks in Sindhudurg. This will enable the
tourists to see the diverse aquatic life and experience sky-diving, sailing and other thrilling sporting
activities. Besides, it will make the new generation familiar with the glorious history of INS Viraat. INS
Viraat, which holds a place of pride in Indian defence history, was decommissioned in March 2017 and
since then has been laid up at the Naval Dockyard. INS Viraat was commissioned by Navy in 1987 and
had taken part in all major military operations such as Operation Jupiter (1988, Sri Lanka peacekeeping
mission), Operation Vijay (1999, post-Kargil), and Operation Parakram (2001, after Parliament attack). It
was made in 1959 for the United Kingdom’s Royal Navy as ‘HMS Hermes’ and was instrumental in the
Falklands War victory. It is 743-foot long and 160-foot wide with a height of 29 feet.
22. Which of the following is the global satellite navigation system of European Union?
[A] BeiDou
[B] GPS
[C] Galileo
[D] GLONASS
Correct Answer: C [Galileo]
Notes:
Recently, China has successfully launched its first geostationary BeiDou-3 satellite ‘G1Q’ into high orbit
(about 36,000 km above the Earth) to boost its home-grown BeiDou global satellite navigation system. It
was launched from the Xichang Satellite Launch Centre in the southwestern Sichuan Province. In a
geostationary orbit, following the Earth’s rotation, it will view the same point on Earth continuously.
The satellite is meant to serve countries in the China-proposed Belt and Road Initiative (BRI). Apart from
radio navigation system, the satellite is equipped with an improved radio determination satellite service
that can provide short message services to 10 million subscribers each hour. With an advanced satellite-
based augmentation system, the satellite can also provide low-cost and reliable navigation services to
civil aviation clients at home and abroad. The satellite is carrying hydrogen and rubidium atomic clocks,
which will play a key role in positioning and timing accuracy. It will be the fourth global satellite

© 2018 GKToday | All Rights Reserved | https://www.gktoday.in 57


https://t.me/APPSC
https://t.me/UpscMaterials https://t.me/FreeUpscMaterials https://t.me/MaterialForExams
Current Affairs [PDF] -November 1-15, 2018

navigation system after the United States (US) Global Position System (GPS), Russia’s GLONASS and the
European Union’s Galileo. By around 2020, when the BeiDou system goes global, it will have more than
30 satellites.
23. India has officially signed a $950 million contract with which country for the purchase of two
upgraded Krivak III-class stealth frigates?
[A] France
[B] Russia
[C] Japan
[D] United States
Correct Answer: B [Russia ]
Notes:
India has officially signed a $950 million contract with Russia for the purchase of two upgraded Krivak
III-class stealth frigates for the Indian Navy. The two frigates of the Project 11356 class will be bought
directly from Russia and a contract to build two more at an Indian yard is likely to be signed at a later
stage. The frigates are built by Yantar Shipyard of Russia and will be powered by Ukrainian gas turbine
engines. They will be equipped with Indian-built radar, sonar, communication suites and anti-ship
BrahMos missile systems. The ships are expected to be delivered by the end of 2022. The procurement of
the frigates will involve a rupee-rouble Manjul
transaction instead of using U.S. dollars to avoid American
Tripathi | tripathimanjul18@gmail.com |
sanctions on Russian defense companies under CAATSA (Countering America’s Adversaries Through
Sanctions Act). The Project 11356 is a derivative of the Talwar-class frigates that Russia has built for the
Indian Navy.
24. Which Indian company has been named an Official Partner of FIH Men’s Hockey World Cup
2018?
[A] Tata Steel
[B] Reliance
[C] Infosys
[D] Wipro
Correct Answer: A [Tata Steel]
Notes:
Steel manufacturing giant Tata Steel has been named as an official partner of the 14th edition of the FIH
men’s hockey World Cup 2018 to continue their efforts of promoting development through sports across
the country. The tournament is scheduled to be held at the Kalinga Stadium in Bhubaneswar, Odisha
from November 28 to December 16. India will host the upcoming prestigious 2018 FIH World Cup for the
3rd time after having played hosts in 1982 in Mumbai and in 2010 in New Delhi. A total of 16 nations will
take part in the tournament which will see each team play three matches during the league stage over a
span of 12 days before four cross-over matches take place on December 10 and 11.
25. Ntozake Shange, who passed away recently, was the acclaimed playwright and poet of which
country?
[A] New Zealand
[B] Brazil
[C] United States
[D] South Africa
Correct Answer: C [United States]
Notes:
Ntozake Shange (70), the noted African-American Playwright, poet and author, has passed away in
Maryland, United States on October 27, 2018. Her most acclaimed theatre piece is the 1975 Tony Award-
nominated play “For Colored Girls Who Have Considered Suicide/When the Rainbow is Enuf”, which
describe the racism, sexism, violence and rape experienced by seven black women. She also penned
several novels including Sassafrass, Cypress & Indigo , Liliane, and Betsey Brown.
26. The first-ever Indo-Japan joint military exercise ‘Dharma Guardian-2018’ has started in which
of the following states?

© 2018 GKToday | All Rights Reserved | https://www.gktoday.in 58


https://t.me/APPSC
https://t.me/UpscMaterials https://t.me/FreeUpscMaterials https://t.me/MaterialForExams
Current Affairs [PDF] -November 1-15, 2018

[A] Mizoram
[B] Nagaland
[C] West Bengal
[D] Odisha
Correct Answer: A [Mizoram]
Notes:
The first-ever Indo-Japan joint military exercise “Dharma Guardian- 2018” has started at a jungle
warfare school in Mizoram’s Vairengte with a focus on strengthening counter-terror cooperation. The
focus of the two-week-long exercise will be to enhance tactical skills against global terrorism and
increase inter-operability between the two forces. The Japanese contingent is represented by 32nd
Infantry Battalion, while the Indian side by 6/1 Gorkha Rifles. The purpose of the exercise is to build and
promote Army to Army relation in addition to exchange skills and experiences of both the armies. The
exercise will comprise a balance of indoor classes and outdoor training activities. Both the armies will
jointly train, plan and execute a series of well developed tactical drills for neutralisation of likely threats
that may be encountered in urban warfare scenario.
27. Which country to host the 23rd edition of the World Corporate Games (WCG-2019)?
[A] Qatar
[B] Saudi Arabia Manjul Tripathi | tripathimanjul18@gmail.com |

[C] India
[D] Bangladesh
Correct Answer: A [Qatar ]
Notes:
First time in Middle east, Qatar will host the 23rd edition of the World Corporate Games (WCG) in Doha
in 2019, which will provide an opportunity for corporations around the world to showcase their
competitive prowess through sports. It will be the first time that the WCG incorporate eSports and
mindsports. The event features both individual and team competitions, open equally to women and men
of all ages and abilities with team events for skilled, semi-skilled or novice participants.It is an all-
inclusive event with paragames integrated for various categories as well. The World Corportate Games
(WCG) aims to bring together, the local, regional and international business sporting community. The
sporting event takes place every one to two years and the 2018 edition was held in Houston in the US.
28. On which date, the 2018 World Vegan Day (WVD) was observed recently?
[A] November 2
[B] November 3
[C] November 1
[D] November 4
Correct Answer: C [November 1]
Notes:
The World Vegan Day (WVD) is observed every year on November 1 to commemorate the holistic
benefits of switching to veganism. Veganism is the practice of abstaining from consuming any animal
products, including milk, curd, eggs, cheese, honey, and meat amongst other derived products.
29. The Jharsuguda Airport of Odisha will be renamed after which eminent freedom fighter of the
state?
[A] Jagabandu Hota
[B] Surendra Sai
[C] Salegram Bariha
[D] Trilochan Panigrahi
Correct Answer: B [Surendra Sai]
Notes:
On November 1, the Union Cabinet has approved a proposal to rename the Jharsuguda airport of Odisha
after eminent freedom fighter Veer Surendra Sai. Renaming of the Jharsuguda airport in his name will
fulfill long-pending demand of the Odisha Government, which reflects the sentiments of the local public
© 2018 GKToday | All Rights Reserved | https://www.gktoday.in 59
https://t.me/APPSC
https://t.me/UpscMaterials https://t.me/FreeUpscMaterials https://t.me/MaterialForExams
Current Affairs [PDF] -November 1-15, 2018

of the respective area. It will also be a befitting tribute to the contribution of the revered personality
associated with the State. Veer Surendra Sai, who was born on 22nd January 1809 in Sambalpur district
of the State, had sacrificed his life fighting against British East India company. He and his associates
resisted the British and successfully protected most parts of Western Odisha region for some time from
the British rule. Many of them were hanged by the British & a few died in the Cellular Jail in the
Andamans. Surendra Sai died in Asirgarh Jail on 23rd May 1884.
30. Simone Biles, who has become the first ever gymnast to win 13 world championship gold
medals, is from which country?
[A] United States
[B] Brazil
[C] South Africa
[D] Chile
Correct Answer: A [United States]
Notes:
Simone Biles, an American artistic gymnast, has become the first ever gymnast to win 13 world
championship gold medals with victory in the individual vault competition in Doha, Qatar on November
2, 2018. She broke the all-time record set by Belarusian male gymnast Vitaly Scherbo in 1996.
31. RBI has recently permitted banks to Manjul
provide PCE to NBFC bonds. What does ‘PCE’ stands for?
Tripathi | tripathimanjul18@gmail.com |

[A] Premium Credit Enhancement


[B] Personal Credit Enhancement
[C] Prone Credit Enhancement
[D] Partial Credit Enhancement
Correct Answer: D [Partial Credit Enhancement ]
Notes:
The Reserve Bank of India (RBI) has recently permitted banks to grant Partial Credit Enhancement
(PCE), or a partial guarantee, to bonds issued by systematically important non-deposit taking Non-
Banking Financial Companies (NBFC-ND-SIs) registered with the Reserve Bank of India and Housing
Finance Companies (HFCs) registered with National Housing Bank. The move is aimed at enhancing the
credit rating of the bonds and enabling these NBFCs to access funds from the bond market on better
terms. PCE is expected to help NBFCs and HFCs raise money from insurance and provident or pension
funds who invest only in highly-rated instruments. The proceeds from the bonds, whose tenor cannot be
less than three years, backed by PCE from banks can only be utilised for refinancing the existing debt.
The central bank has restricted the exposure of a bank through PCEs to bonds issued by each such NBFC
or HFC to 1% of capital funds of the bank within the current single and group borrower exposure limits.
32. What is the theme of the United Nations for 2018 World Cities Day (WCD)?
[A] Promoting a better urban future
[B] Building Sustainable and Resilient Cities
[C] Better City, Better Life
[D] Innovative Governance, Open Cities
Correct Answer: B [Building Sustainable and Resilient Cities]
Notes:
The United Nations (UN) has designated every 31st of October as World Cities Day (WCD) to promote
the international community’s interest in global urbanization, enhance cooperation among countries
and cities in meeting opportunities and addressing challenges of urbanization, and contribute to
sustainable urban development. This year, the United Nations has selected the 2018 theme ‘Building
Sustainable and Resilient Cities’ because cities need support to become resilient and develop their
capacity to absorb the impact of hazards, protect and preserve human life and limit damage to and
destruction of public and private assets while continuing to provide infrastructure and services after a
crisis. The main event was celebrated in Liverpool, UK.
33. India and which country have signed a loan pact of Rs 18.17 crore for the construction of the
Turga Pumped Storage in West Bengal?

© 2018 GKToday | All Rights Reserved | https://www.gktoday.in 60


https://t.me/APPSC
https://t.me/UpscMaterials https://t.me/FreeUpscMaterials https://t.me/MaterialForExams
Current Affairs [PDF] -November 1-15, 2018

[A] France
[B] Brazil
[C] South Africa
[D] Japan
Correct Answer: D [Japan]
Notes:
India and Japan has recently a Loan Agreement worth Rs. 1817 crore for the ‘Project for the Construction
of Turga Pumped Storage (I)’ in the State of West Bengal. The purpose of the Project is to strengthen the
capability to respond to fluctuation in supply and demand of power and to improve stability of the power
supply by constructing the pumped storage facilities, thereby contributing to the Industrial
Development and Living Standard Improvement in the State of West Bengal.
34. Recently, the first-ever India-US Dialogue On Intellectual Property (IP) was launched in which
of the following cities?
[A] New York
[B] New Delhi
[C] Washington DC
[D] Pune Manjul Tripathi | tripathimanjul18@gmail.com |

Correct Answer: B [New Delhi]


Notes:
Recently, the first-ever India-US Dialogue On Intellectual Property (IP) was launched in New Delhi to
deepen bilateral strategic cooperation on IP policy. The dialogue was launched by US Chamber of
Commerce’s Global Innovation Policy Center (GIPC) and FICCI in partnership with the US-India
Business Council. It will be convened annually, alternating between New Delhi and Washington DC. The
dialogue aims to identify solutions, and also share technical knowledge and expertise. The US-India
Business Council, which was formed in 1975, is a business advocacy organisation comprised of 350 top-
tier US and Indian companies advancing US-India commercial ties.
35. Who has been named the Brand Ambassador of 2018 Women’s World Boxing Championships
(WWBC)?
[A] Sarjubala Devi
[B] Yogeshwar Dutt
[C] Mary Kom
[D] Pinki Jangra
Correct Answer: C [Mary Kom]
Notes:
Mary Kom, a five-time World Champion, has been named the Brand Ambassador of the 10th edition of
theWomen’s World Boxing Championships (WWBC-2018). The World Championships is scheduled to be
held at the IG Stadium’s K.D. Jhadav Hall in New Delhi from November 15 and 24. It will see close to 300
pugilists from 70 countries across 10 weight categories, the tournament’s highest-ever participation,
including Olympic medalists, World and European Champions.
36. Which of the following cities is the venue of the 2018 India International Cherry Blossom (IICB)
festival?
[A] Agartala
[B] Gangtok
[C] Srinagar
[D] Shillong
Correct Answer: D [Shillong]
Notes:
The 2nd edition of the India International Cherry Blossom (IICB) festival in Shillong, which is is
scheduled to be held from November 14 and 17. It is the world’s only autumn cherry blossom festival. The
fest will celebrate the unique mass flowering of autumn Himalayan cherry blossoms with several

© 2018 GKToday | All Rights Reserved | https://www.gktoday.in 61


https://t.me/APPSC
https://t.me/UpscMaterials https://t.me/FreeUpscMaterials https://t.me/MaterialForExams
Current Affairs [PDF] -November 1-15, 2018

cultural events in the Meghalaya capital. The visitors can enjoy fashion shows, rock concerts, beauty
pageants and also compete in an amateur golf tournament. Alongside, there will be stalls showcasing the
food, arts and crafts of the region, and several cultural events and a food pavilion of Japan in partnership
with the Embassy of Japan in India. It is expected that the fest will attract more than one lakh visitors
from across the world.
37. Which city has topped the Ola Mobility Institute (OMI)’s ‘Ease of Moving Index 2018’ in public
transport?
[A] Chennai
[B] Hyderabad
[C] Kolkata
[D] New Delhi
Correct Answer: C [Kolkata]
Notes:
According to India’s first-ever ‘Ease of Moving Index 2018’ – India Report 2018, Kolkata has topped the
index in terms of cleanliness and affordability of public transport, followed by New Delhi and Chennai.
The index offers insights on mobility preferences & aspirations of Indian commuters across 20 cities
with a population of over 92 million. It was released by Ola Mobility Institute (OMI), the research and
outreach arm of Ola ride-hailing platform. The 2018 report, which was released by the Union Minister
Manjul Tripathi | tripathimanjul18@gmail.com |
for Surface Transport Nitin Gadkari, seeks to offer a comprehensive action plan for cities to further
strengthen their public transportation systems and improve the overall state of mobility. Over 43,000
respondents from 20 cities across the country were surveyed. Cities were selected on five parameters —
scale, character, culture, economy and geography. The report also assessed the accessibility, efficiency
and safety of current transportation facilities. More than 95% of denizens prefer sustainable mobility
and 80% observed transport ecosystem has improved over the last five years with 60% reporting using
public modes of commute.
38. Which of the following films was the India’s lone entry at the Rome Film Festival 2018?
[A] Sui Dhaaga
[B] Parmanu: The Story of Pokhran
[C] Raazi
[D] Mare Pyare Prime Minister
Correct Answer: D [Mare Pyare Prime Minister]
Notes:
India was partner country at Videocitta 2018 in the Rome Film Festival 2018, which was held from 18th
to 28th October. The Indian delegation at the Film Festival was lead by Ashok Kumar Parmar, Joint
Secretary of Ministry of I&B. The agenda of the Indian delegation was to showcase Indian films’ heritage,
ease of film shooting in India, promoted film shooting locations in India, International Film Festival of
India, Indian Cinema from different regions, Audio Visual Co-Production between India and Italy. This
year, “Mare Pyare Prime Minister” was the India’s lone entry in Rome Film Festival. The film revolves
around four children living in a Mumbai slum. One of them wants to build a toilet for his single mother
and makes an appeal to the Prime Minister.
39. Under the provisions of which article of the constitution, the President appoint judges of
Supreme Court (SC)?
[A] Article 125
[B] Article 124
[C] Article 126
[D] Article 127
Correct Answer: B [Article 124]
Notes:
Under the provisions of Article 124 of the Constitution, the President has recently appointed four new
judges — Justices Hemant Gupta, R Subhash Reddy, M R Shah and Ajay Rastogi – of Supreme Court
(SC). Chief Justice Ranjan Gogoi administered the oath of office to the four judges. The President’s assent
was based on the recommendation of the Supreme Court Collegium consists of CJI Ranjan Gogoi, Justice
Madan B Lokur, Justice Kurian Joseph, Justice A K Sikri and Justice S A Bobde. The apex court has a
© 2018 GKToday | All Rights Reserved | https://www.gktoday.in 62
https://t.me/APPSC
https://t.me/UpscMaterials https://t.me/FreeUpscMaterials https://t.me/MaterialForExams
Current Affairs [PDF] -November 1-15, 2018

sanctioned strength of 31. With the elevation of these four new judges, the strength has risen from 24 to
28.
40. Which city is the venue of the National Seminar on Entrepreneurship and Business
Development in Ayurveda?
[A] Guwahati
[B] Chennai
[C] Pune
[D] New Delhi
Correct Answer: D [New Delhi]
Notes:
On November 4, the National Seminar on Entrepreneurship and Business Development in Ayurveda was
started at Dr. Ambedkar International centre in New Delhi. The 2-day seminar is being organised by
Ministry of AYUSH in association with NITI Aayog with the aim of encouraging entrepreneurs and
Ayurveda stakeholders towards businesses opportunities in the sector. The seminar is expected to
develop awareness among stakeholders about business opportunities, encouraging young entrepreneurs
to use modern technologies and highlight opportunities at global level. Experts in marketing,
innovation, policymakers, experts in start-up as well as experienced entrepreneurs in Ayurveda
pharmaceuticals and hospital industry will guide and share their experiences with the participants.
Manjul Tripathi | tripathimanjul18@gmail.com |
Nearly 800 participants from across the country are expected to take part in the seminar.
41. RBI has started process to set up PCR for capturing borrowers’ details. The acronym “PCR”
stands for?
[A] Public Credit Registry
[B] Personal Credit Registry
[C] Prone Credit Registry
[D] Premium Credit Registry
Correct Answer: A [Public Credit Registry ]
Notes:
The Reserve Bank of India (RBI) has initiated steps to set up a wide-based digital Public Credit Registry
(PCR) to capture details of all borrowers, including wilful defaulters and also the pending legal suits in
order to check financial delinquencies. The PCR will also include data from entities like market regulator
SEBI, the Ministry of Corporate Affairs, Goods and Service Tax Network (GSTN) and the Insolvency and
Bankruptcy Board of India (IBBI) to enable the banks and financial institutions to get 360 degree profile
of existing and prospective borrowers on a real-time basis. The central bank has invited expression of
interest for developing the registry from companies with a turnover of over 100 crore rupees in the last
three years. The PCR would be the single point of mandatory reporting for all material events for each
loan, notwithstanding any threshold in the loan amount or type of borrower. Currently, there are
multiple granular credit information repositories in India, with each having somewhat distinct
objectives and coverage.
42. IAMAI has constituted which AR/VR Committee to promote the technologies in India?
[A] Kumar Yadav committee
[B] Mahesh Prabhu committee
[C] Ajit Kumar committee
[D] Namrita Mahindro committee
Correct Answer: D [Namrita Mahindro committee ]
Notes:
The Internet and Mobile Association of India (IAMAI) has formed a new industry expert committee to
develop and promote an augmented reality/virtual reality (AR/VR) ecosystem in India. The committee is
chaired by Namrita Mahindro, Senior General Manager of digital transformation in Mahindra Group and
co-chaired by Satyajeet Singh, Head –Strategic Product Partnership, India and South Asia, Facebook.
The committee is a step in IAMAI’s efforts to evangelise the nascent technology’s ability to drive
economic growth, job creation, and skill development in the country. The agenda of the committee will
include nurturing India’s AR/VR technology and talent ecosystem, in particular by driving Skill
Development; engaging with business, industry and government to evangelize acceptance of the
© 2018 GKToday | All Rights Reserved | https://www.gktoday.in 63
https://t.me/APPSC
https://t.me/UpscMaterials https://t.me/FreeUpscMaterials https://t.me/MaterialForExams
Current Affairs [PDF] -November 1-15, 2018

technology and helping drive training workshops to familarise potential users with technology and build
use cases, in key sectors.
43. The All-India ‘Unreserved Mobile Ticketing’ facility (UTS on Mobile) has been developed by
Indian Railway- CRIS. Where is the headquarters of CRIS?
[A] Pune
[B] New Delhi
[C] Kolkata
[D] Chennai
Correct Answer: B [New Delhi]
Notes:
On November 1, the Ministry of Railways has officially rolled out All-India Unreserved Mobile Ticketing
facility (UTS on Mobile) with a view to promote three C’s- Cashless transactions (Digital payment),
Contact less ticketing (no need to physically visit the point of sale) and Customer convenience and
experience. Now, the facility of booking unreserved tickets, including season tickets and also platform
tickets is available through the ‘UTSONMOBILE’ app available for Android, IOS and Windows phones.
The booking of unreserved tickets on all non-suburban sections across all Zonal Railways has been made
available from 1st November to enable seamless booking of unreserved tickets all over Indian Railways.
The ‘UTS on Mobile’ application ‘in house’ has been developed by Centre for Railway Information
Manjul Tripathi | tripathimanjul18@gmail.com |
Systems (CRIS). CRIS was established in 1986 by the Ministry of Railways and its of headquarters CRIS is
located in New Delhi.
44. V. Rathinam, veteran freedom fighter and member of Indian National Army (INA) has passed
away. He belonged to which state?
[A] Karnataka
[B] Kerala
[C] Tamil Nadu
[D] Odisha
Correct Answer: C [Tamil Nadu]
Notes:
V. Rathinam (99), the veteran freedom fighter and member of Indian National Army (INA) led by Nethaji
Subash Chandra Bose, has passed away in South Kodikulam, Tamil Nadu on November 4, 2018. Born on
16th August 1919, Rathinam had moved to Burma when he was a bachelor and joined the INA during the
Second World War in the early 1940s. After completing the training programme, he was inducted into
the INA and fought against the British. Later, he was imprisoned by the British along with others and
served jail terms in different prisons before he was set free from a prison in Kolkata.
45. The first WHO Global Conference on Air Pollution and Health was held in which city?
[A] New York
[B] Geneva
[C] New Delhi
[D] Paris
Correct Answer: B [Geneva]
Notes:
The first Global Conference on Air Pollution and Health was held at World Health Organisation (WHO)
Headquarters in Geneva on October 30, 2018. The 2-day conference was organized by WHO in
collaboration with UN Environment, World Meteorological Organization (WMO), the Secretariat of the
UNFCCC, the Climate and Clean Air Coalition to Reduce Short-Lived Climate Pollutants (CCAC) and the
United Nations Economic Commission for Europe (UNECE). The theme of the conference was:
“Improving Air Quality, Combating Climate Change- Saving Lives”. Some 92% of people worldwide
breathe unhealthy air, resulting in about 6.5 million deaths annually. In it, household air pollution is a
leading killer in poor rural and urban homes. Thus, the event responds to a World Health Assembly
mandate to combat one of the world’s most significant causes of premature death. At the conclusion of
the Conference, participants agreed an aspirational goal of reducing the number of deaths from air
pollution by two thirds by 2030.

© 2018 GKToday | All Rights Reserved | https://www.gktoday.in 64


https://t.me/APPSC
https://t.me/UpscMaterials https://t.me/FreeUpscMaterials https://t.me/MaterialForExams
Current Affairs [PDF] -November 1-15, 2018

46. ‘Little India Gate’, which is in news recently, is inaugurated in which of the following cities?
[A] Medan
[B] Jakarta
[C] Banten
[D] Depok
Correct Answer: A [Medan]
Notes:
The ‘Little India Gate’, the first-of-its-kind structure in Indonesia, has recently inaugurated at Kampung
Madras or Madras Village area in Medan city, the 4th largest city in the country. It recognises the
contribution of Indian community in the development of Medan city. The new structure was jointly by
India’s ambassador to Indonesia and Timor Leste Pradeep Kumar Rawat and Mayor of Medan H T
Dzulmi Eldin S. About 7,000 people witnessed the historic moment with great enthusiasm. The
Kampung Madras area is one of the city’s significant ethnic enclaves comprising a large population of
people of Indian descent whose ancestors had settled down in Medan in mid-19th century. Every year,
Kampung Madras becomes a meeting point for Hindu and Tamil festivals such as Thaipusam or the
Tamil New Year, Pongal and Deepavali.
47. On which date, the 2018 World Tsunami Awareness Day (WTAD) is observed on which date?
[A] November 3 Manjul Tripathi | tripathimanjul18@gmail.com |

[B] November 4
[C] November 5
[D] November 6
Correct Answer: C [November 5]
Notes:
The World Tsunami Awareness Day (WTAD) is observed every year on November 5 to raise awareness
about precautionary measures against tsunami and share innovative approaches to risk reduction.
Along with risk of large loss of life, the day also highlight the economic losses tsunamis can inflict along
vulnerable, densely populated coastlines, shown in red on this map. Tsunamis are rare events, but can be
extremely deadly. In the past 100 years, 58 tsunamis have claimed more than 260,000 lives, which is the
highest number of deaths than in any other natural hazard. The day was the brainchild of Japan, which
due to its repeated, bitter experience has over the years built up major expertise in areas such as
tsunami early warning, public action and building back better after a disaster to reduce future impacts.
48. India’s first asymmetrical cable-stayed bridge “Signature Bridge” has recently inaugurated in
which of the following cities?
[A] Mumbai
[B] Kolkata
[C] Chennai
[D] Delhi
Correct Answer: D [Delhi]
Notes:
India’s first asymmetrical cable-stayed bridge “Signature Bridge” has recently inaugurated by the Chief
Minister of Delhi, Arvind Kejriwal across the Yamuna River. The bridge connects Outer Ring Road on the
western flank of the Yamuna river with the Wazirabad Road on the east. It shortens the travel time
between north and northeast Delhi. The 675-metre long bridge, originally conceptualised in 2004, has
finally available for public use from November 5. Just like the Eiffel Tower in France, Signature Bridge
will also have the facility to transport tourists on top of the main pillar, which is 154 m high, from where
distance view of North Delhi can be seen.The pylon of the Signature bridge is tallest structure in Delhi
and is double the height of Qutb Minar, which act as selfie points for people.
49. The Union government has awarded which state police for citizen services under an e-
governance project “CCTNS”?
[A] Haryana
[B] Assam

© 2018 GKToday | All Rights Reserved | https://www.gktoday.in 65


https://t.me/APPSC
https://t.me/UpscMaterials https://t.me/FreeUpscMaterials https://t.me/MaterialForExams
Current Affairs [PDF] -November 1-15, 2018

[C] Jharkhand
[D] Kerala
Correct Answer: B [Assam ]
Notes:
The Ministry of Home Affairs (MHA) has awarded the Assam Police for “good practices” under the Crime
and Criminal Tracking Network & Systems (CCTNS), an e-governance project. The award was given for
citizen centric service delivery, in which the state police has automated passport verification and
Permanent Residential Certificate (PRC) services using the online CCTNS. Under the CCTNS, the force
has recently launched a mobile application for ease of availability of services to the citizens, besides the
existing online system.
50. Which noted journalist has been chosen for the prestigious Raja Ram Mohan Roy Award?
[A] Ruby Sarkar
[B] Rajesh Parshuram Joshte
[C] VS Rajesh
[D] N. Ram
Correct Answer: D [N. Ram]
Notes: Manjul Tripathi | tripathimanjul18@gmail.com |
N. Ram, the veteran journalist and The Hindu publishing group chairman, has been chosen for the
prestigious Raja Ram Mohan Roy Award 2018 for his outstanding contribution towards journalism. The
award will be presented by the Press Council of India (PCI) on November 16 on the occasion of the
National Press Day (NPD).
51. Which of the following cities is the venue of the 18th IORA Council of (Foreign) Ministers
Meetings (COM)?
[A] Santiago
[B] Durban
[C] Brasília
[D] Harare
Correct Answer: B [Durban]
Notes:
On 2nd November 2018, South Africa hosted the 18th Indian Ocean Rim Association (IORA) Council of
(Foreign) Ministers Meetings (COM) in Durban under the theme of “IORA – Uniting the Peoples of
Africa, Asia, Australasia and the Middle East through Enhanced Co-operation for Peace, Stability and
Sustainable Development”. This meeting was held during the on-going celebrations of the 150th year of
the birth anniversary of Mahatma Gandhi and the 100th birth anniversary of Nelson Mandela. The
Indian delegation was led by Gen.(Dr.) VK Singh (Retd.), Minister of State for External Affairs. The IORA
is an international organisation consisting of coastal states bordering the Indian Ocean. The Association
comprises 21 member states and its headquarters is located at Ebene, Mauritius.
52. Which company has received India’s first Letter of Credit (LC) payment via blockchain?
[A] Tata Motors
[B] Wipro
[C] Reliance Industries
[D] Infosys
Correct Answer: C [Reliance Industries]
Notes:
HSBC Holdings Plc has executed India’s first trade financial transaction using blockchain for a deal
involving an export by Reliance Industries Ltd (RIL) to US-based Tricon Energy. The blockchain-enabled
letter of credit (LC) reduces transaction timeline from 7-10 days to less than a day. The Letter of Credit
(LC) was issued by ING Bank for Tricon Energy USA (importer) with HSBC India as the advisory and
negotiation bank for RIL. The blockchain platform was integrated with the electronic bill of lading (eBL)
platform to issue and manage an electronic bill of lading. This allows a digital transfer of the title of
goods from the seller to the buyer in the underlying trade. It is claimed that the blockchain solution is a
major improvement for any organisation involved in buying and selling goods internationally, as it
© 2018 GKToday | All Rights Reserved | https://www.gktoday.in 66
https://t.me/APPSC
https://t.me/UpscMaterials https://t.me/FreeUpscMaterials https://t.me/MaterialForExams
Current Affairs [PDF] -November 1-15, 2018

brings together all parties onto one platform. It is a distributed database that maintains a continuously-
growing list of ordered records called blocks and is deemed to be very transparent as all the stakeholders
can view progress real time.
53. The Government of India (GoI) has recently approved operationalisation strategy for
Operation Greens for which of the following crops?
[A] Tomato, Onion and Potato
[B] Onion, Carrot and Peas
[C] Potato, Wheat and Onion
[D] Tomato, Peas and Chilli
Correct Answer: A [Tomato, Onion and Potato]
Notes:
The Ministry of Food Processing Industries (MoFPI) has recently approved the operationalisation
strategy for Operation Greens to ensure availability of Tomato, Onion and Potato (TOP) throughout the
country round the year without price volatility. Operation Greens was announced in the Budget speech
of 2018-19 with an outlay of Rs. 500 crore to stabilize the supply of TOP crops. One of the objectives of
“Operation Greens” is the reduction in post-harvest losses by creation of farm gate infrastructure,
development of suitable agro-logistics, creation of appropriate storage capacity linking consumption
centres. Manjul Tripathi | tripathimanjul18@gmail.com |

54. Who has been appointed India’s new Ambassador to Iran?


[A] Saurabh Kumar
[B] Gaddam Dharmendra
[C] N K Saxena
[D] Priti Jindal
Correct Answer: B [Gaddam Dharmendra]
Notes:
Gaddam Dharmendra, a 1990-batch Indian Foreign Service officer, has been appointed India’s new
Ambassador to Iran. He will succeed Saurabh Kumar, who was appointed India’s ambassador to
Myanmar. Dharmendra’s appointment comes at a time when the global focus is on oil imports from Iran
as the US has recently imposed “the toughest ever” sanctions aimed at altering the Iranian regime’s
“behaviour”.
55. Who of the following lifted the 2018 Men’s Singles title at the Rolex Paris Masters?
[A] Milos Raonic
[B] Juan Martín del Potro
[C] Karen Khachanov
[D] Novak Djokovic
Correct Answer: C [Karen Khachanov]
Notes:
In Tennis, Karen Khachanov has lifted the Who of the following lifted the 2018 Men’s Singles title at the
Rolex Paris Masters with a stunning win over World No. 2 Novak Djokovic. In the title clash, Khachanov
defeated Djokovic in straight sets by 7-5, 6-4 to end the Serb’s 22-match unbeaten run. With the victory,
Khachanov has become the first Russian since Nikolay Davydenko in 2009 to win a Masters 1000 title.
56. Which sports journalist is the author of the book “The Fire Burns Blue: A History of Women’s
Cricket in India”?
[A] Jatin Sapru
[B] Karunya Keshav
[C] Mayanti Langer
[D] Boria Majumdar
Correct Answer: B [Karunya Keshav]
Notes:
The book, titled ‘The Fire Burns Blue: A History of Women’s Cricket in India’, is co-authored by sports
© 2018 GKToday | All Rights Reserved | https://www.gktoday.in 67
https://t.me/APPSC
https://t.me/UpscMaterials https://t.me/FreeUpscMaterials https://t.me/MaterialForExams
Current Affairs [PDF] -November 1-15, 2018

journalists Karunya Keshav and Sidhanta Pathak. This is the first comprehensive history of women’s
cricket that documents the entire gamut of the women’s game starting from its humble beginnings, its
spirited journey through its early years to the present-day when truly, the game has come of age. It talks
about the numerous “twist and turns” of women’s cricket, including the early days of pioneers like Diana
Eduljee and Shantha Rangaswamy in the early ’70s and the stars of today like Mithali Raj and
Harmanpreet Kaur. The book will be launched as part of the publishing house’s new imprint, Westland
Sport on November 30, 2018.
57. What is the theme of the 2018 National Ayurveda Day (NAD) of India?
[A] Ayurveda for Public Health
[B] Ayurveda for Prevention and Control of Diabetes
[C] Ayurveda: Cure and Protect Our Lives
[D] Live happily with Ayurveda
Correct Answer: A [Ayurveda for Public Health]
Notes:
The 2018 National Ayurveda Day (NAD) is celebrated on November 5 with the theme “Ayurveda for
Public Health” to create awareness on people’s health. The day focus on strengths of Ayurveda and its
unique treatment principles. It create a sense of awareness in today’s generation and promote Ayurvedic
principles of healing in society. The main aim of Ayurveda is the prevention of disease and promotion of
health. Lord Dhanvantari is considered Manjul
as divine propagator of Ayurveda. He is conferred with the
Tripathi | tripathimanjul18@gmail.com |

virtues of granting health and wealth. Hence, Dhanvantari Jayanti (Dhanteras) was preferred for
celebration of Ayurveda Day to nationalize this system of medicine which can prove to be a cornerstone
for its ultimate globalization.
58. The newly-built Ekana cricket stadium is located in which of the following cities?
[A] Udaipur
[B] Lucknow
[C] New Delhi
[D] Pune
Correct Answer: B [Lucknow]
Notes:
On November 6, Yogi Adityanath-led Uttar Pradesh government has renamed the newly-constructed
Ekana International Cricket Stadium after former prime minister Atal Bihari Vajpayee. The stadium will
now be known as ‘Bharat Ratna Atal Bihari Vajpayee International Cricket Stadium’ . It has a seating
capacity of around 50,000 and will host its first international match on November 6, when the 2nd
Twenty-20 match between India and the West Indies will be played in the stadium in Lucknow . Hence,
Lucknow will host the first international match after 24 years. The stadium was named after Vajpayee,
as the former prime minister was a Member of Parliament from Lucknow for five consecutive times
between 1991 and 2009 .
59. Which renowned personality has been honoured with ‘Distinguished Fellow Award’ by
IndiaGlobal in Boston?
[A] Sachin Tendulkar
[B] Sudha Murthy
[C] Anupam Kher
[D] Piyush Goyal
Correct Answer: C [Anupam Kher]
Notes:
In Boston, veteran actor Anupam Kher has recently been honoured with ‘Distinguished Fellow Award’ by
IndiaGlobal – a leading think-tank that works on issues concerning non-resident Indians. He was given a
fellowship at the 3rd IndiaGlobal Summit 2018. The event was held at the prestigious MIT Sloan School
of Management in Boston, and was facilitated by MIT Sloan India Business Club.
60. Supreme Court has recently given nod for a separate High Court for which of the following
states/UTs?

© 2018 GKToday | All Rights Reserved | https://www.gktoday.in 68


https://t.me/APPSC
https://t.me/UpscMaterials https://t.me/FreeUpscMaterials https://t.me/MaterialForExams
Current Affairs [PDF] -November 1-15, 2018

[A] Nagaland
[B] Andhra Pradesh
[C] Andaman and Nicobar Islands
[D] Mizoram
Correct Answer: B [Andhra Pradesh]
Notes:
The Supreme Court of India (SCI) has recently given nod for a separate High Court for Andhra Pradesh
and it will start functioning from 1st January 2019. It will be the 25th high court in the country and will
initially function from a temporary structure till the permanent building comes up in the ambitious
Justice City complex in the State’s capital Amaravati. Since the bifurcation of the State on June 2, 2014,
Andhra Pradesh and Telangana have had a common high court situated at Hyderabad. The existing
court will from next year be known as the Telangana High Court.
61. Who has been elected the new President of International Boxing Association (AIBA)?
[A] Thomas Bach
[B] P S Bopanna
[C] Gafur Rakhimov
[D] Will Smith Manjul Tripathi | tripathimanjul18@gmail.com |

Correct Answer: C [Gafur Rakhimov]


Notes:
Gafur Rakhimov, an Uzbek businessman and sports administrator, has been elected as the new President
of the International Boxing Association (AIBA), which is an amateur boxing’s global governing body.
AIBA is a sport organization that sanctions amateur (Olympic-style) boxing matches and awards world
and subordinate championships. Its headquarters is located at Lausanne, Switzerland.
62. Which state government has recently rolled out the technology-driven & citizen-friendly ‘Mo
Bus’ service?
[A] Andhra Pradesh
[B] Karnataka
[C] Odisha
[D] Kerala
Correct Answer: C [Odisha ]
Notes:
In Bhubaneswar, the Odisha government has recently rolled out the technology-driven and citizen-
friendly ‘Mo bus’ service featuring free wi-fi, CCTV surveillance, public information display system and
on board announcements in the city. The service was launched as a part of state government-owned
Capital Region Urban Transport’s (CRUT) ‘City Bus Modernisation’ programme.It will also provide the
Mo Bus mobile app through which passengers can book their tickets and get to know the locations of the
vehicle to minimise their waiting time at the Bus Queue Shelters (BQS). The BQS will have advanced
features like digital screens to display the bus route numbers and their arrival times for the waiting
passengers. The bus service has been launched for the residents of Bhubaneswar, Cuttack and Puri.
63. Which international organisation has recently inked $172 million loan pact to make farming
viable in Andhra Pradesh?
[A] AIIB
[B] New Development Bank
[C] UNICEF
[D] World Bank
Correct Answer: D [World Bank]
Notes:
The Central and Andhra Pradesh governments and the World Bank have signed the loan agreement for a
$172.20 million project to enhance agricultural productivity, profitability, and climate resilience of poor
and marginalised farmers in Andhra Pradesh. The projects seeks to ensure that farming continues to

© 2018 GKToday | All Rights Reserved | https://www.gktoday.in 69


https://t.me/APPSC
https://t.me/UpscMaterials https://t.me/FreeUpscMaterials https://t.me/MaterialForExams
Current Affairs [PDF] -November 1-15, 2018

remain a financially viable activity. The Andhra Pradesh Integrated Irrigation and Agriculture
Transformation Project (APIIATP) will be implemented in rural areas largely dependent upon rain-fed
agriculture. It will strengthen the resilience of poor and marginalised farmers against adverse climate
events by improving access to irrigation, drought seed varieties and post-harvest technology that are
aimed at improving soil health, water-use efficiency and crop productivity. The project will benefit over
200,000 families of poor and marginalised farmers, agro-entrepreneurs, women and other vulnerable
groups. The $172.20 million loan from the International Bank for Reconstruction and Development
(IBRD), has a 6-year grace period, and a maturity of 24 years.
64. For which term, India has been elected as a member of the International Telecommunications
Union (ITU) Council?
[A] 2020 to 2023
[B] 2019 to 2022
[C] 2021 to 2024
[D] 2018 to 2021
Correct Answer: B [2019 to 2022]
Notes:
India has been elected as a member of the International Telecommunications Union (ITU) Council for
another four-year term from 2019 to 2022. The elections to the Council were held during the ongoing
Manjul Tripathi | tripathimanjul18@gmail.com |
ITU Plenipotentiary Conference 2018 at Dubai. India secured 165 votes and ranked third among the 13
countries elected to the council from the Asia-Australasia region, and 8th among the 48 countries
elected to the council globally. The ITU has 193 member states who elect representatives to the Council.
India has been a regular member of the ITU Council since 1952, and has played an important role in
harmonising the contributions of member states from the region, always respecting the principles of
equality and consensus-building.
65. Which of the following countries have recently exempted by United States for buying Iranian
oil?
[A] China, Taiwan and Greece
[B] Japan, South Korea and India
[C] India, Italy and Turkey
[D] All of the above
Correct Answer: D [All of the above]
Notes:
On November 5, the United States imposed the toughest ever sanctions on Iran aimed at altering the
Iranian regime’s “behaviour”. The sanctions cover Iran’s banking and energy sectors and reinstate
penalties for countries and companies in Europe, Asia and elsewhere that do not halt Iranian oil imports.
However, 8 countries – India, China, Italy, Greece, Japan, South Korea, Taiwan and Turkey — were
temporarily allowed to continue buying Iranian oil as they showed “significant reduction” in oil purchase
from the Persian Gulf country. The United States has exempted India from imposition of certain
sanctions for the development of the strategically-located Chabahar Port in Iran, along with the
construction of the railway line connecting it with Afghanistan. The decision by the Trump
administration is being seen as a recognition by Washington of India’s major role in the development of
the port on the Gulf of Oman, which is of immense strategic importance for the reconstruction of war-
torn Afghanistan. The Chabahar Port is considered a gateway to golden opportunities for trade by India,
Iran and Afghanistan with central Asian countries besides ramping up trade among the three countries
after Pakistan denied transit access to India.
66. Who has been appointed the new Secretary General of Lok Sabha?
[A] Snehlata Shrivastava
[B] Anup Mishra
[C] Rama Devi
[D] Nischal Jain
Correct Answer: A [Snehlata Shrivastava]
Notes:
The tenure of the Lok Sabha Secretary General, Snehlata Shrivastava, has been extended by Speaker
© 2018 GKToday | All Rights Reserved | https://www.gktoday.in 70
https://t.me/APPSC
https://t.me/UpscMaterials https://t.me/FreeUpscMaterials https://t.me/MaterialForExams
Current Affairs [PDF] -November 1-15, 2018

Sumitra Mahajan for one year. Snehlata, who is 1982-batch retired IAS officer of the Madhya Pradesh
cadre, was appointed last year in rank and status of a Cabinet Secretary. She was scheduled to retire on
the 30th of this month. Her extension will be effective from the 1st of December 2018 to the 30th of
November 2019. She is the first woman Secretary General of the Lok Sabha.
67. Which Indian shooter has won the historic men’s skeet gold at the Asian Shotgun
Championship 2018 in Kuwait?
[A] Saurabh Chaudhary
[B] Arjun Babuta
[C] Angad Vir Singh Bajwa
[D] Hriday Hazarika
Correct Answer: C [Angad Vir Singh Bajwa]
Notes:
In the 8th Asian Shotgun Championship 2018 in Kuwait, Angad Vir Singh Bajwa has won the men’s skeet
gold with a perfect score of 60 — a World record — in the final on November 7. With this, he became the
first Indian skeet shooter to win a continental or world level event. Angad shot a perfect 60 out of 60 in
the final round to claim top spot ahead of China’s Di Jin who shot 58. UAE’s Saeed Al Maktoum won
bronze with a score of 46. In the concurrently held 11th Asian Air Gun championship, Indian team of
Elavenil Valarivan and Hriday Hazarika won the mixed air rifle gold with a world junior record of 502.1.
Manjul Tripathi | tripathimanjul18@gmail.com |
The other Indian team of Mehuli Ghosh and Arjun Babuta bagged the bronze.
68. Under which schedule of the Andhra Pradesh Reorganisation Act – 2014, the Central Tribal
University will be set up in the state?
[A] 10th
[B] 13th
[C] 11th
[D] 12th
Correct Answer: B [13th ]
Notes:
The Union Cabinet has recently approved Amendment to the Central Universities Act – 2009 for setting
up of a Central Tribal University in Andhra Pradesh. A fund of Rs 420 crore has been granted as the first
phase expenditure towards the establishment of the University. The ‘Central Tribal University of Andhra
Pradesh’ will be set up in Relli village of Vizianagaram district as provided under the 13th schedule to the
Andhra Pradesh Reorganisation Act, 2014.
69. Lalan Sarang, who passed away recently, was the noted theatre personality of which regional
cinema?
[A] Marathi
[B] Odia
[C] Tamil
[D] Telugu
Correct Answer: A [Marathi ]
Notes:
Lalan Sarang (79), the noted Marathi actor and theatre personality, has passed away recently in Pune,
Maharashtra on November 9, 2018. Born on 26th December 1938 in Mumbai, Sarang is known for her
roles in path-breaking plays, including “Sakharam Binder”, “Gidhade”, “Rathchakra” and “Kamla”. She
received several accolades for these roles. While playing key roles in films such as “Samna”, “Ha Khel
Sawalyancha” and “Mahek”, she also worked in several Hindi serials. Recipient of ‘Grahini Sachiv Award’
of Ga Di Ma Pratishthan, Sarang also penned a few books- “Natakanmagil Natya”, “Mi ani Majhya
Bhumika” and “Bahardar Kisse”.
70. The 5th edition of World Buddhist Forum (WBF -2018) was recently held in which country?
[A] India
[B] Nepal

© 2018 GKToday | All Rights Reserved | https://www.gktoday.in 71


https://t.me/APPSC
https://t.me/UpscMaterials https://t.me/FreeUpscMaterials https://t.me/MaterialForExams
Current Affairs [PDF] -November 1-15, 2018

[C] Sri Lanka


[D] China
Correct Answer: D [China]
Notes:
The 5th edition of World Buddhist Forum (WBF -2018) was recently held in Putian, the southeast Fujian
Province of China. Over 1,000 Buddhists, scholars and representatives from 55 countries and regions
attended the forum. The WBF-2018 was jointly hosted by the Buddhist Association of China and the
China Religious Culture Communication Association. The aim of the forum is to carry forward the
positive Buddhist cultural spirits, promote exchanges between Buddhism and other religions, and make
contributions to building a community with a shared future for humanity. The World Buddhist Forum
(WBF) was initiated by the Buddhist circles in the Chinese mainland, Taiwan, Hong Kong and Macao in
2005.
71. The 25th maritime exercise “SIMBEX-18” has started between India and which country?
[A] Saudi Arabia
[B] Seychelles
[C] Sri Lanka
[D] Singapore
Correct Answer: D [Singapore ] Manjul Tripathi | tripathimanjul18@gmail.com |

Notes:
The 25th edition of bilateral maritime exercise “SIMBEX-18” has started between the Navies of the
Republic of Singapore and India from 10th to 21st November 2018 off Andaman Sea and Bay of Bengal.
The SIMBEX 2018 will be the largest edition since 1994 in terms of scale and complexity. The initial
harbour phase will be held at Port Blair, the capital of Andaman and Nicobar Islands from 10 to 12
November followed by Sea Phase from 12 to 16 November in the Andaman Sea. The second harbour
phase from 16 to 19 November will be held at Visakhapatnam – ‘the City of Destiny’. The final sea phase
would be held in the Bay of Bengal from 19 to 21 November. The 2018 edition marks the Silver Jubilee of
Singapore-India Maritime Bilateral Exercise (SIMBEX). To mark the historical occasion, both navies are
undertaking exercises over an extended geography. The 25th edition will witness a diverse range of
exercises at sea ranging from live weapon drills including Heavy Weight Torpedo (HWT), advanced Anti-
Submarine Warfare (ASW) exercises, integrated Surface and Anti-Air Warfare (AAW); Unmanned Aerial
Vehicle (UAV) operations and Cross Deck Helicopter flying among others.
72. India is hosting the “Global IT Challenge for Youth with Disabilities 2018” in collaboration with
which country?
[A] South Korea
[B] Spain
[C] Japan
[D] France
Correct Answer: A [South Korea]
Notes:
In New Delhi, the Global IT Challenge for Youth with Disabilities 2018 is organised by the Department of
Empowerment of Persons with Disabilities (DEPwD) under Ministry of Social Justice and Empowerment
during 9-11 November, 2018. India is hosting the event in collaboration with Government of Korea and
Rehabilitation International (RI), and their associated partner LG Electronics. The purpose of the 3-day
event is to leverage IT skills among youth with disabilities and also to spread awareness about the
application of Information and Computer Technology (ICT) in enhancing the quality of life of persons
with disabilities especially in Asia-Pacific region. Around 100 youth with disabilities (visual disability,
hearing disability, locomotor disability and intellectual disability/developmental disorder) from 18
countries namely, India, Indonesia, China, Vietnam, Malaysia, Thailand, Sri Lanka, Bangladesh, Nepal,
Mongolia, Cambodia, Laos, Philippines, Korea, Kazakhstan, Kyrgyzstan, UK and UAE are participating in
this event.
73. The first-ever solutions-focused event “Global Cooling Innovation Summit” will be held in
which country?
[A] Brazil

© 2018 GKToday | All Rights Reserved | https://www.gktoday.in 72


https://t.me/APPSC
https://t.me/UpscMaterials https://t.me/FreeUpscMaterials https://t.me/MaterialForExams
Current Affairs [PDF] -November 1-15, 2018

[B] India
[C] South Africa
[D] United Kingdom
Correct Answer: B [India]
Notes:
The first-of-its-kind solutions-focused event “Global Cooling Innovation Summit” will be inaugurated by
Union Science & Technology Minister, Dr. Harsh Vardhan in New Delhi on 12th November 2018. The
Summit will bring together leaders from around the world to explore concrete means and pathways to
address the climate threat that comes from the growing demand from room air conditioners. The 2-day
event is jointly organized by the Department of Science and Technology (DST) along with Rocky
Mountain Institute, Alliance for An Energy Efficient Economy (AEEE), Conservation X Labs and CEPT
University. The summit will also see the launching of Global Cooling Prize— Mission Innovation
challenge that aims to spur development of a residential cooling solution that has at least five times (5x)
less climate impact than today’s standard.
74. Which news agency has recently launched the world’s first AI news anchor?
[A] Yonhap
[B] Kyodo News
[C] Xinhua Manjul Tripathi | tripathimanjul18@gmail.com |

[D] Lankapuvath
Correct Answer: C [Xinhua ]
Notes:
China’s state news agency Xinhua has recently launched the world’s first Artificial Intelligence (AI) news
anchor at the ongoing 5th World Internet Conference in Wuzhen. The anchor resembles a male human
anchor with the voice and facial expressions of a real person. “He” learns from live broadcasting videos
by himself and can read texts as naturally as a professional human anchor. Two AI news anchors that
can read news in English as well as Mandarin were jointly developed by Xinhua and the Chinese search
engine “Sogou”. The anchors were developed through Machine Learning (ML) programme to simulate
the voice, facial movements, and gestures of real-life broadcasters, to present a “a lifelike image instead
of a cold robot”.
75. The Union Cabinet has recently approved sale of Govt. of India’s equity in DCIL to consortium
of which of the following ports?
[A] Paradeep Port
[B] Vishakhapatnam Port
[C] Jawahar Lal Nehru Port
[D] All of the above
Correct Answer: D [All of the above]
Notes:
The Cabinet Committee on Economic Affairs (CCEA) has recently approved strategic disinvestment of
100% Govt. of India’s equity in Dredging Corporation of India Ltd. (DCIL) to consortium of four ports –
Vishakhapatnam Port Trust (VPT), Paradeep Port Trust (PPT), Jawahar Lal Nehru Port Trust (JLNPT)
and Kandla Port Trust (KPT). The Govt. of India (GoI) currently holds 73.44% in DCIL. The approval will
further facilitate the linkage of dredging activities with the ports, keeping in view the role of the DCIL in
expansion of dredging activity in the country as well as potential scope for diversification of ports into
third party dredging. The GoI has budgeted to raise Rs 80,000 crore from PSU disinvestment. So far this
fiscal, the government has mopped up over Rs 15,000 crore from PSU stake sale.
76. Who has been appointed the new chairperson of Competition Commission of India (CCI)?
[A] G P Mittal
[B] U C Nahta
[C] Ashok Kumar Gupta
[D] Smita Jhingran
Correct Answer: C [Ashok Kumar Gupta]

© 2018 GKToday | All Rights Reserved | https://www.gktoday.in 73


https://t.me/APPSC
https://t.me/UpscMaterials https://t.me/FreeUpscMaterials https://t.me/MaterialForExams
Current Affairs [PDF] -November 1-15, 2018

Notes:
Ashok Kumar Gupta, a 1981 Tamil Nadu cadre IAS officer, has been appointed as new chairperson of
Competition Commission of India (CCI). His tenure as the CCI Chairman will be till October 25, 2022, i.e.
when attains the age of 65 years, or until further orders, whichever is earlier. He will succeed acting
chairperson Sudhir Mittal. The CCI is a Quasi-judicial statutory body , established in October 2003 under
the Competition Act of 2002, that keeps a tab on unfair business practices across sectors. It is the duty of
the Commission to eliminate practices having adverse effect on competition, promote and sustain
competition, protect the interests of consumers and ensure freedom of trade in the markets of India.
77. Which Indian personality has recently bestowed with France’s highest civilian honour “Knight
of the Legion of Honour”?
[A] Kailash Satyarthi
[B] Amartya Sen
[C] Jawahar Lal Sarin
[D] Soumitra Chatterjee
Correct Answer: C [Jawahar Lal Sarin]
Notes:
Jawahar Lal Sarin, the president of the Alliance Francaise de Delhi’s governing body, has been bestowed
with France’s highest civilian honour “Knight of| tripathimanjul18@gmail.com
Manjul Tripathi the Legion of | Honour”. The award was presented to
Sarin by Ambassador of France Alexandre Ziegler in recognition of his outstanding contribution to
enhancing Indo-French cultural cooperation and promoting the French language. The Alliance francaise
de Delhi was started as a French association by Francophiles in 1956. The Legion d’Honneur (Legion of
Honour) was instituted in 1802 by Napoleon Bonaparte. It is the highest civilian award given by the
French Republic for outstanding service to France, regardless of the nationality of the recipient.
78. On which date, the National Cancer Awareness Day (NCAD), was observed recently in India?
[A] November 7
[B] November 9
[C] November 10
[D] November 11
Correct Answer: A [November 7]
Notes:
In India, the National Cancer Awareness Day (NCAD) is observed every year on 7th of November to
remove all stops to lower the national cancer burden by generating awareness about early detection and
avoiding leading cancer causing lifestyles. The main motto of the special day is to educate people on
early detection & cure of cancer. It is estimated that there are about 2.9 million cases of cancer in the
country with 1.1 million new cases being reported annually. Two-thirds of cancer cases are diagnosed at
the advanced stage.
79. Which IIT has developed a portable solar-powered cold storage device with a 500 kg capacity?
[A] IIT Indore
[B] IIT Bombay
[C] IIT Kanpur
[D] IIT Madras
Correct Answer: D [IIT Madras]
Notes:
The IIT Madras has recently developed a portable solar-powered cold storage device with a 500 kg
capacity for storing vegetables, and fruits, among other items. The device will help farmers store their
produce for sufficient time so that it does not get spoilt before it is sold, thereby preventing wastage of
agricultural produce.The first unit of the device, which provides storage facility with temperature
ranging between 4 to 10 degree celsius, was deployed at a farm in Madurantakam in Kancheepuram
district, around 80 km from Chennai, recently. Each unit would cost between Rs 5.5 to Rs 6 lakh and
storage capacity could be scaled down based on requirement. The project has been funded by
Department of Science and Technology (Ministry of Science and Technology) and IIT-M. A startup called
Tan90, initiated by IIT-M students, aims to commercialise the technology and take it to individual
farmers with even smaller devices at affordable prices.
© 2018 GKToday | All Rights Reserved | https://www.gktoday.in 74
https://t.me/APPSC
https://t.me/UpscMaterials https://t.me/FreeUpscMaterials https://t.me/MaterialForExams
Current Affairs [PDF] -November 1-15, 2018

80. Who has won the 2018 Fuzhou China Open Badminton tournament?
[A] Huang Yuxiang
[B] Lin Dan
[C] Kento Momota
[D] Chou Tien-chen
Correct Answer: C [Kento Momota]
Notes:
World champion Kento Momota from Japan has won the 23rd edition of Fuzhou China Open Badminton
tournament 2018 on November 11. He defeated the 4th seed Chou Tien-chen of Taiwan 21-13, 11-21, 21-16
in 66 minutes. Momota is known for his explosive movements on court and his unpredictable style of
play.
81. Which Indian wrestler has been ranked world No. 1 rank in 65 Kg category?
[A] Sushil Kumar
[B] Sanjeet Kumar
[C] Yogeshwar Dutt
[D] Bajrang Punia
Manjul Tripathi | tripathimanjul18@gmail.com |
Correct Answer: D [Bajrang Punia]
Notes:
Star Indian wrestler Bajrang Punia has recently touched a new high in his career by achieving the World
number one rank in the 65kg category. He was placed atop the ranking table with 96 points in the
United World Wrestling (UWW) list. The UWW is the international governing body for the sport of
amateur wrestling. Its duties include overseeing wrestling at the Olympics. Bajrang has won five medals
this season, including Common Wealth Games and the Asian Games gold and a silver at the World
Championship in Budapest. Bajrang was also the only Indian grappler to be given a seeding at the
Budapest World Championship.
82. M777 Howitzers, which are recently inducted into Indian Army, are procured from which
country?
[A] United States
[B] South Korea
[C] Russia
[D] Italy
Correct Answer: A [United States]
Notes:
The new artillery guns and equipment, including K9 Vajra, M777 howitzers, have recently inducted into
the Indian Army at the Deolali artillery centre in Nashik district of Maharashtra. Beside this, the third
gun system inducted is the ‘Composite Gun Towing Vehicle’ for towing some of the existing guns in
service with the country. The K9 VAJRA-T 155mm/ 52 is a tracked self-propelled artillery gun, which has
its roots in the K9 Thunder, the mainstay of the South Korean Army. The Vajra offers a high rate of fire
at a long range and is compatible with Indian and standard NATO ammunition. The M777 is the 155mm/
39 Calibre Ultra Light Howitzer (ULH) that have been procured from USA under Government to
Government Foreign Military Sales and will be assembled in India by BAE Systems in partnership with
Mahindra Defence. The M777s can be easily transported to high altitude areas such as borders with
China and Pakistan by helicopters.
83. Ananth Kumar, who passed away recently, was in charge of which of the following union
ministries?
[A] Ministry of Parliamentary Affairs
[B] Ministry of Overseas Indian Affairs
[C] Ministry of Heavy Industries and Public Enterprises
[D] Ministry of Labour and Employment
Correct Answer: A [Ministry of Parliamentary Affairs]

© 2018 GKToday | All Rights Reserved | https://www.gktoday.in 75


https://t.me/APPSC
https://t.me/UpscMaterials https://t.me/FreeUpscMaterials https://t.me/MaterialForExams
Current Affairs [PDF] -November 1-15, 2018

Notes:
Ananth Kumar (59), the senior BJP leader , has recently passed away in Bengaluru on November 12th,
2018. He was in charge of two key ministries – serving as Union Minister of Chemicals and Fertilizers
since May 2014 and as Minister of Parliamentary Affairs since July 2016 in the Narendra Modi
government until his death. He represented the Bengaluru South parliamentary constituency in the Lok
Sabha six times since 1996. Kumar joined the BJP in 1987 and never looked back, as he held posts of state
secretary, state president of the Yuva Morcha, general secretary and national secretary.
84. Which Indian mountaineer has recently been awarded an honorary doctorate by a prestigious
University of Strathclyde ?
[A] Bachendri Pal
[B] Premlata Agarwal
[C] Arunima Sinha
[D] Malavath Purna
Correct Answer: C [Arunima Sinha]
Notes:
Arunima Sinha, the Indian mountaineer who became the world’s first woman amputee to climb Mount
Everest in 2013, has been recently awarded an honorary doctorate by a prestigious UK university ‘
University of Strathclyde’ at a graduationManjul
ceremony in Glasgow
Tripathi | tripathimanjul18@gmail.com | for her inspirational achievements in
mountaineering. The award also recognises Ms Sinha’s charitable work through the Arunima
Foundation, which seeks to empower women, disabled people and generally improve the health and
social and economic situation for poorer communities.
85. What is the theme of the 2018 World Science Day for Peace and Development?
[A] Science for Global Understanding
[B] Science, a Human Right
[C] Celebrating Science Centres and Science Museums
[D] Quality Science Education
Correct Answer: B [Science, a Human Right]
Notes:
The World Science Day for Peace and Development is observed every year on 10th of November to
highlight the significant role of science in society and the need to engage the wider public in debates on
emerging scientific issues. It also underlines the importance and relevance of science in our daily lives.
The theme for 2018 is “Science, a Human Right”, in celebration of the 70th anniversary of the Universal
Declaration of Human Rights (art. 27), and of the Recommendation on Science and Scientific
Researchers. The purpose of the day is to strengthen public awareness on the role of science for peaceful
and sustainable societies.
86. Which city is the venue of the International Conference on Yoga for Public Health?
[A] Kolkata
[B] Panjim
[C] Agartala
[D] Bhopal
Correct Answer: B [Panjim]
Notes:
On November 12, the International Conference on Yoga for Public Health has inaugurated at Kala
Academy, Panjim. The two-day conference is being organised by the Ministry of Ayush with theme ‘Yoga
for Public Health’. The 1000 representatives from around 50 countries are participating in the
conference.
87. Which state government is celebrating the “Jashn-e-Virasat-e-Urdu” festival?
[A] Telangana
[B] Uttarakhand
[C] Delhi

© 2018 GKToday | All Rights Reserved | https://www.gktoday.in 76


https://t.me/APPSC
https://t.me/UpscMaterials https://t.me/FreeUpscMaterials https://t.me/MaterialForExams
Current Affairs [PDF] -November 1-15, 2018

[D] Jharkhand
Correct Answer: C [Delhi ]
Notes:
The Delhi government is celebrating the “Jashn-e-Virasat-e-Urdu”, a festival to promote & preserve the
Urdu culture and its heritage, at Central Park, Connaught Place. The aim of the 6-day festival is to keep
the language alive and ensure its growth through larger participation. The festival will witness a diverse
range of artistes and artforms practising Urdu including traditional forms like Chaar Bait – a rare
traditional style of singing in Urdu, Dastangoi and Kissagoi. Delhi’s culturally-rich heritage from the
walled city will also be showcased at the festival with various arts and crafts including composite
calligraphy styles. The Urdu festival will come to a close on November 15.
88. During INSPIRE 2018, EESL and ADB signed pact for $13 million Global Environment Facility
(GEF) grant to set up an EERF. What does EERF stands for?
[A] Environment Efficiency Revolving Fund
[B] Ecology Efficiency Revolving Fund
[C] Eco Efficiency Revolving Fund
[D] Energy Efficiency Revolving Fund
Correct Answer: D [Energy Efficiency Revolving Fund]
Manjul Tripathi | tripathimanjul18@gmail.com |
Notes:
In New Delhi, the 2nd edition of INSPIRE 2018 (International Symposium to Promote Innovation &
Research in Energy Efficiency) has been inagurated by R K Singh, the Union Power Minister. During
INSPIRE 2018, EESL and Asian Development Bank (ADB) signed pact for a Global Environment Facility
(GEF) grant of $13 million to establish an Energy Efficiency Revolving Fund (EERF) to support
investments in new, innovative and scalable business models in India. The aim of EERF is to expand and
sustain investments in the energy efficiency market in India, build market diversification, and scale up
existing technologies. INSPIRE 2018 is being organised by Energy Efficiency Services Limited (EESL) and
World Bank. The 3-day symposium will focus on enhancing grid management, e-Mobility, financial
instruments and technologies for energy efficiency in India. The Power Minister also gave away awards
to four pathbreaking innovations in clean energy and energy efficiency as part of #InnovateToINSPIRE,
a first-of-its-kind energy innovation challenge.
89. What is the theme of the 33rd ASEAN summit 2018?
[A] Partnering for change, engaging the world
[B] Resilient and Innovative Asean
[C] One ASEAN at the Heart of Dynamic Asia
[D] Strengthening ASEAN Maritime Cooperation
Correct Answer: B [Resilient and Innovative Asean]
Notes:
The 33rd edition of ASEAN summit 2018 has started in the city-state of Singapore on November 11 with
theme ‘Resilient and innovative Asean’. The main event is being held at Suntec Singapore Convention
Centre, wherein the ten ASEAN members will be discussing key issues on economic, security and
cultural cooperation. The summit is being chaired by Singapore’s Prime Minister Lee Hsien Loong.
Besides the main event, several related meetings like Regional Comprehensive Economic Partnership
(RCEP), East Asia Summit (EAS) and Fintech Festival are also taking place during the 5-day event. Prime
Minister Narendra Modi is scheduled to attend the East Asia Summit and Associated meetings of
ASEAN on 14th and 15th November in Singapore. In the biggest of its kind event more than 400
companies from over 100 countries are taking part as exhibitors.
90. Which of the following countries has become the world’s first nation to ban reef-killing toxic
sunscreen?
[A] Nauru
[B] Republic of Palau
[C] New Guinea
[D] Vanuatu
Correct Answer: B [Republic of Palau]
© 2018 GKToday | All Rights Reserved | https://www.gktoday.in 77
https://t.me/APPSC
https://t.me/UpscMaterials https://t.me/FreeUpscMaterials https://t.me/MaterialForExams
Current Affairs [PDF] -November 1-15, 2018

Notes:
The Republic of Palau, an archipelago found in the western Pacific Ocean, has become the world’s first
nation to ban the use of sunscreens that have been deemed harmful to coral reefs and ocean life. The ban,
which will come into force by 2020, has been signed into law by the Palau government. These
sunscreens are made from “reef-toxic” chemicals such as oxybenzone, octinoxate and triclosan. These
chemicals causes corals to bleach at lower temperatures, and it reduces their resilience to climate
change. Palau is home to just 21,000 people but has a booming tourism and fishing industry. It contains
approximately 340 islands, forming the western chain of the Caroline Islands in Micronesia, and has an
area of 466 square km.
91. Which Indian journalist has won the 2018 London Press Freedom Award for Courage?
[A] Ravish Kumar
[B] Arnab Goswami
[C] Swati Chaturvedi
[D] Dhruv Rathee
Correct Answer: C [Swati Chaturvedi]
Notes:
Swati Chaturvedi, an Indian freelance journalist, has won the 2018 London Press Freedom Award for
Courage for her work in exposing the “IT cell” within the ruling Bharatiya Janata Party (BJP) for keeping
Manjul Tripathi | tripathimanjul18@gmail.com |
an army of angry trolls. She beat shortlisted journalists from Italy, Turkey, and Morocco to wom the
award. Swati, who is famous for her investigative reports, is the author of the book title “I am a Troll:
Inside the Secret World of the BJP’s Digital Army”. The award was presented at the Getty Images Gallery
in London’s Fitzrovia on November 8. The Prize for Courage is given to journalists who show courage in
their work or in the defence and promotion of journalism.
92. As per latest US-based National Bureau of Economic Research (NBER) report, which of the
following cities is the India’s most congested city?
[A] Bengaluru
[B] Delhi
[C] Chennai
[D] Kolkata
Correct Answer: A [Bengaluru ]
Notes:
The US-based National Bureau of Economic Research (NBER) has recently published the report titled
‘Mobility and Congestion in Urban India’. In it, Bengaluru has been ranked India’s most congested city
followed by Mumbai, Delhi, Chennai, Kolkata, Hyderabad and Pune. The report further found that
Bengaluru was the second slowest city in terms of traffic speeds only behind Kolkata. The report also
notes that in terms of traffic speeds, the fastest Indian cities surveyed were slower than the American
cities surveyed. The researchers studied 154 Indian cities and measured trips based on the web-mapping
service ‘Google Maps’. They particularly studied pairs of trip instances taking place around the same
time on different days. The study was conducted by a team of researchers at theUniversity of California,
Berkeley and three other U.S. universities.
93. Which article of the Constitution provides free legal assistance to the poor and weaker
sections of the society?
[A] Article 39A
[B] Article 38A
[C] Article 37A
[D] Article 36A
Correct Answer: A [Article 39A]
Notes:
The National Legal Services Day (NLSD) is celebrated every year on 9th of November in India to ensure
reasonable fair and justice procedure for all citizens. It was first started by Supreme court of India (SCI)
in the year 1995 to provide help and support to poor and weaker sections of the society. The Article 39A
of Constitution provides free legal assistance to the poor and weaker sections of the society. It directs
the State to ensure that the operation of the legal system promotes justice on a basis of equal
© 2018 GKToday | All Rights Reserved | https://www.gktoday.in 78
https://t.me/APPSC
https://t.me/UpscMaterials https://t.me/FreeUpscMaterials https://t.me/MaterialForExams
Current Affairs [PDF] -November 1-15, 2018

opportunity and shall, in particular, provide free legal aid by suitable legislation or schemes or in any
other way, to ensure that opportunities for securing justice are not denied to any citizen by reason of
economic or other disabilities. The Right to free legal aid or free legal service is an essential fundamental
right guaranteed by the Constitution. It forms the basis of reasonable, fair and just liberty under Article
21 of the Constitution of India, which says, “No person shall be deprived of his life or personal liberty
except according to procedure established by law”.
94. Who is the author of the book “Notes of a Dream The Authorized Biography of A.R. Rahman”?
[A] Anuja Chauhan
[B] Krishna Trilok
[C] Mridula Behari
[D] Devapriya Roy
Correct Answer: B [Krishna Trilok]
Notes:
The book titled “Notes of a Dream The Authorized Biography of A.R. Rahman” has been authored by
Krishna Trilok. It is a magical journey of Oscar-winning musician A.R. Rahman. The book captures
Rahman’s extraordinary success story with all the rhythm and melody, the highs and lows, of a terrific
soundtrack by the man himself.
95. The Union HRD ministry has launched the| tripathimanjul18@gmail.com
Manjul Tripathi LEAP initiative | for higher education faculty. What
does ‘LEAP’ stands for?
[A] Legitimate for Academicians Programme
[B] Lasting for Academicians Programme
[C] Leadership for Academicians Programme
[D] Legible for Academicians Programme
Correct Answer: C [Leadership for Academicians Programme]
Notes:
Dr. Satya Pal Singh, Minister of State, Minister of State for Human Resource Development, launched the
Leadership for Academicians Programme (LEAP) and Annual Refresher Programme In Teaching (ARPIT)
for higher education faculty in New Delhi on November 13. ARPIT is a unique initiative of online
professional development of 15 lakh higher education faculty using the MOOCs platform SWAYAM. For
implementing ARPIT, 75 discipline-specific institutions have been identified and notified as National
Resource Centres (NRCs) in the first phase. LEAP is a 3 weeks flagship leadership development training
programme for second level academic functionaries in public-funded higher education institutions. The
main purpose of LEAP is to prepare the second tier academic heads who are potentially likely to assume
leadership roles in the future.
96. Which India’s restoration project has won the 2018 UNESCO Asia-Pacific award for
conservation in Award of Distinction category?
[A] Ruttonsee Muljee Jetha Fountain
[B] Rajabai Clock Tower
[C] Agra Fort
[D] LAMO center
Correct Answer: D [LAMO center]
Notes:
The restoration of an aristocratic house from a state of partial ruin in Ladakh undertaken by the LAMO
center has won the 2018 UNESCO Asia-Pacific award for conservation in the category of Award of
Distinction. The LAMO (Ladakh Arts and Media Organisation) Center in Jammu and Kashmir’s Ladakh
region was chosen for its systematic restoration project that used salvaged and local building materials,
and indigenous construction techniques while adroitly introducing modern amenities to assure its
ongoing use. Apart from this, the 2018 Award of Excellence has been won by the renewal of the early
20th-century Shijo-cho Ofune-hoko Float Machiya of Kyoto, Japan. Beside this, from Mumbai, the
restoration projects of the iconic Rajabai Clock Tower of Mumbai University & Ruttonsee Muljee Jetha
Fountain, both belonging to the colonial-era have received Honourable Mention.
97. The first-ever bilateral naval exercise ‘Samudra Shakti 2018’ has started between India and
© 2018 GKToday | All Rights Reserved | https://www.gktoday.in 79
https://t.me/APPSC
https://t.me/UpscMaterials https://t.me/FreeUpscMaterials https://t.me/MaterialForExams
Current Affairs [PDF] -November 1-15, 2018

which country?
[A] Indonesia
[B] Seychelles
[C] Vanuatu
[D] Philippines
Correct Answer: A [Indonesia]
Notes:
The first-ever bilateral naval exercise ‘Samudra Shakti 2018’ has started between Indian & Indonesian
Navy at the port of Surabaya in Indonesia from November 12 to 18. The aim of the exercise is to
strengthen bilateral relations, expand maritime co-operation, enhance interoperability and exchange
best practices. From Indian side, INS Rana of the Eastern Fleet based at Visakhapatnam under the
Eastern Naval Command is participating in the exercise.
98. Which social initiative has been launched by Twitter India to boost youth engagement for 2019
General Elections?
[A] #PowerOf18
[B] #PowerOfVoters
[C] #PowerOf19 Manjul Tripathi | tripathimanjul18@gmail.com |

[D] #PowerOfElection
Correct Answer: A [#PowerOf18]
Notes:
Twitter India has recently launched a new social initiative #PowerOf18, which aims to encourage youth
to contribute in public debates and participate in civic engagement for the 2019 general elections. The
#PowerOf18 will serve as a resource for young Indians to find more information about the elections,
support social causes they are passionate about and join the public conversation. The initiative was
launched by CEO Jack Dorsey and Maya Hari, Vice-President and Managing Director of Asia Pacific,
Twitter during a townhall chat with the students of IIT Delhi.
99. Stan Lee, who passed away recently, was the legendary comic book writer of which country?
[A] South Africa
[B] United States
[C] France
[D] Canada
Correct Answer: B [United States ]
Notes:
Stan Lee (95), the legendary comic book writer and former chief of Marvel Comics, has passed away in
Los Angeles, United States. Lee created the Fantastic Four for Marvel Comics in 1961 and went on to
create titles like Spider-Man and The Incredible Hulk. In collaboration with several artists—particularly
Jack Kirby and Steve Ditko—he co-created fictional characters including Spider-Man, the Hulk, Doctor
Strange, the Fantastic Four, Daredevil, Black Panther, the X-Men, and—with his brother, co-writer Larry
Lieber—the characters Ant-Man, Iron Man, and Thor.
100. Who has become the first Indian to climb Mt Giluwe in Papua New Guinea?
[A] Taka Tamut
[B] Sivangi Pathak
[C] Raj Singh Dharmshaktu
[D] Satyarup Siddhanta
Correct Answer: D [Satyarup Siddhanta]
Notes:
Indian mountaineer Satyarup Siddhanta from Bengal has become the first Indian to climb Mt Giluwe –
the second highest mountain in Papua New Guinea. He reached the summit of the mountain which is at
an elevation of 4,367m on November 11, 2018. With this, Satya has scaled 5 of the 7 volcanic summits. In a
few days, he will now set out to summit Mt Wilhelm – the highest mountain of Papua New Guinea. Once

© 2018 GKToday | All Rights Reserved | https://www.gktoday.in 80


https://t.me/APPSC
https://t.me/UpscMaterials https://t.me/FreeUpscMaterials https://t.me/MaterialForExams
Current Affairs [PDF] -November 1-15, 2018

successful, Satya will be the first from India to scale these two mountains. In September 2018, Satyarup
Siddhanta and Mousumi Khatua created history by climbing Asia’s highest volcanic peak Mount
Damavand and became the first Bengalis to do so. Mt Damavand is the highest peak in Iran and also a
potentially active volcano.
101. The 17th ASEAN Economic Community (AEC) Council Meeting was held in which country?
[A] Indonesia
[B] New Zealand
[C] Singapore
[D] Malaysia
Correct Answer: C [Singapore ]
Notes:
On November 12, The 17th ASEAN Economic Community (AEC) Council Meeting was held in Singapore
as part of the 33rd ASEAN Summit and Related Summit Meetings. Apart from taking stock of regional
economic integration efforts during the year, it also reiterated ASEAN’s commitment to free and open
trade by deepening the integration of its economies and realising the AEC Blueprint 2025. The blueprint
seeks sustained and long-term growth for the ASEAN region. The meeting was attended by the ASEAN
Economic Ministers and AEC Ministers. They signed the ASEAN Agreement on e-Commerce, Concluded
the ASEAN Trade in Services Agreement (ATISA) and Finalised the 4th Protocol to Amend the ASEAN
Comprehensive Investment Agreement (ACIA). Manjul Tripathi | tripathimanjul18@gmail.com |

102. Who has been honoured with the 2018 Munin Barkotoki Literary Award?
[A] Arupa Kalita Patangia
[B] Homen Borgohain
[C] Debabhuson Borah
[D] Dhrubajyoti Bora
Correct Answer: C [Debabhuson Borah]
Notes:
Young writer-critic Dr Debabhuson Borah has been honoured with the prestigious Munin Barkotoki
Literary Award 2018 in Assam for his book on literary criticism titled ‘Nirbochon’ The annual award,
which carries a cash prize of Rs 50,000 along with a citation and a plaque, was instituted in the memory
of renowned writer, critic and journalist Munin Barkotoki to encourage young and promising authors
writing in Assamese. The Awards for the years 2017 and 2018 will be given away at a function to be held
here in February 2019. It must be noted that Short-story writer Dalim Das has been selected for the 2017
award for his collection ‘Hari Jadu Madhu’.
103. The Abu Dhabi National Oil Company (ADNOC) has recently hired which of the following
India’s strategic crude oil storage facility?
[A] Padur
[B] Visakhapatnam
[C] Mangalore
[D] Rajkot
Correct Answer: A [Padur ]
Notes:
On November 12, Abu Dhabi National Oil Company (ADNOC) signed MoU with the Indian Strategic
Petroleum Reserves Ltd (ISPRL) to explore the possibility of storing ADNOC crude oil at Padur
Underground Facility in Udupi district of Karnataka, which has a 2.5 million tonne (~17 million barrels)
capacity. So far, ADNOC is the only foreign oil & gas company to invest by way of crude oil in India’s
strategic petroleum reserves program. ISPRL is an Indian government-owned company mandated to
store crude oil for emergency needs and its headquarters is located at Noida. ISPRL has already built 5.33
million tonnes of underground storage capacity at three locations – Visakhapatnam (1.33 million tonnes),
Mangalore (1.5 million tonnes) and Padur (2.5 million tonnes), that can meet around 9.5 days of the
country’s oil needs as per consumption data of last financial year.
104. Which of the following countries is the Partner Country at the 38th India International Trade
Fair (IITF-2018)?
© 2018 GKToday | All Rights Reserved | https://www.gktoday.in 81
https://t.me/APPSC
https://t.me/UpscMaterials https://t.me/FreeUpscMaterials https://t.me/MaterialForExams
Current Affairs [PDF] -November 1-15, 2018

[A] Iran
[B] Afghanistan
[C] Nepal
[D] Bangladesh
Correct Answer: B [Afghanistan ]
Notes:
On November 14, the 38th India International Trade Fair (IITF) has started at Pragati Maidan in New
Delhi with theme – Rural Enterprises in India. The 14-day annual event will continue till 27th of this
month. Afghanistan is the Partner Country, while Nepal will be the Focus Country. Jharkhand is
participating as a Partner State in the event. Around 800 participants from States and Government
Departments, domestic and International companies are taking part to showcase their product. For the
general public, the trade fair will open from 18th to 27th of November. During a public holiday, the price
of the ticket will be Rs.120 for adult and Rs. 60 for the child. While during working days, the ticket
charges will be Rs. 60 for adult and 40 rupees for the child.
105. What is the theme of the 2018 campaign of World Diabetes Day (WDD)?
[A] Go Blue for Breakfast
[B] Eyes on Diabetes
[C] The Family and Diabetes Manjul Tripathi | tripathimanjul18@gmail.com |

[D] Women and diabetes


Correct Answer: C [The Family and Diabetes ]
Notes:
The World Diabetes Day (WDD) is observed every year on November 14 to raise awareness among
families to discover, prevent and manage diabetes. It is the primary global awareness campaign focusing
on diabetes mellitus The theme for WDD 2018 – 2019 is “The Family and Diabetes – diabetes concerns
every family”. It focuses on the fact that families need to work together in adopting a healthy lifestyle in
order to prevent and manage diabetes.
106. India’s first multi-modal terminal has recently inaugurated on which of the following rivers?
[A] Yamuna
[B] Ganga
[C] Brahmaputra
[D] Godavari
Correct Answer: B [Ganga ]
Notes:
On November 11, Prime Minister Narendra Modi inaugurated India’s first multi-modal terminal on the
Ganga river in his parliamentary constituency in Varanasi and received the country’s first container
cargo transported on inland waterways from Kolkata. This is the first of the four multi-modal terminals
being constructed on the National Waterway-1 (River Ganga) as part of the World Bank-aided ‘Jal Marg
Vikas Project’ of the Inland Waterways Authority of India. The total estimated cost of the project is Rs
5,369.18 crore, which will be equally shared between the Government of India and the World Bank. The
aim of the Centre’s Jal Marg Vikas Project is to develope the stretch of the river between Varanasi and
Haldia for navigation of large vessels weighing up to 1,500 tonnes to 2,000 tonnes. Its objective is to
promote inland waterways as a cheap and an environment-friendly means of transportation, especially
for cargo movement. The Inland Waterways Authority of India (IWAI) is the project implementing
agency.
107. Which is the host nation for the conference on Global Digital Content Market (GDCM) 2018?
[A] India
[B] Nepal
[C] France
[D] Indonesia
Correct Answer: A [India]
Notes:

© 2018 GKToday | All Rights Reserved | https://www.gktoday.in 82


https://t.me/APPSC
https://t.me/UpscMaterials https://t.me/FreeUpscMaterials https://t.me/MaterialForExams
Current Affairs [PDF] -November 1-15, 2018

On November 14, the conference on Global Digital Content Market (GDCM) 2018 has started in New
Delhi with focus on ‘Asia Pacific region’. The 2-day conference will feature sessions on music, film,
broadcasting and publishing, as well as collective management, emerging models and the implications
for the market and policy makers. It is being organized by the Department of Industrial Policy and
Promotion (DIPP), Ministry of Commerce and Industry. India has been chosen as a host nation for the
conference byWorld Intellectual Property Organization(WIPO) due to the strong creative industry in the
country in films, music and media. GDCM 2018 is the second edition of the conference. The aim of
GDCM 2018 is to give industry stakeholders a platform to come together and discuss new avenues in the
films, music, gaming and creative industry along with points of challenge that is presented due to the
changing creative landscape because of the disruption brought about by digital technologies.
108. Which Indian city is the venue of the regional meeting of the World Customs Organisation
(WCO)?
[A] New Delhi
[B] Jaipur
[C] Pune
[D] Hyderabad
Correct Answer: B [Jaipur]
Notes: Manjul Tripathi | tripathimanjul18@gmail.com |
The regional meeting of the World Customs Organisation has started in Jaipur, Rajasthan with
representatives of 33 member countries of Asia attending it. The meeting is slated to deliberate upon
various issues including the steps required for the capacity building and reforms in customs.The
participants would also discuss and deliberate upon the amended Kyoto protocol, digital customs, e-
commerce among other issues. The meeting is being jointly chaired by WCO Deputy Secretary General
Ricardo Travino and Central Board of Indirect Taxes and Customs Chairman S Ramesh.
109. Which of the following countries is the Partner Country at the 9th Vibrant Gujarat Summit
2019?
[A] Oman
[B] Qatar
[C] UAE
[D] Israel
Correct Answer: C [UAE]
Notes:
The 9th edition of the Vibrant Gujarat Global Summit 2019 is scheduled to be held in Gandhinagar from
18th to 20th January with theme `Shaping of a New India’. The `Youth Connect Forums’. The summit
will provide a forum for discussion on global, national and state-level agendas with a sharp focus on all
round economic development for a ‘New India’. United Arab Emirates (UAE) will participate in the
summit as a partner country for the second consecutive year.

© 2018 GKToday | All Rights Reserved | https://www.gktoday.in 83


https://t.me/APPSC

You might also like